Jump to content

Wikipedia:Reference desk/Science

From Wikipedia, the free encyclopedia

This is an old revision of this page, as edited by 91.111.64.177 (talk) at 23:02, 28 February 2009 (→‎Stone-age axe-head factory.: new section). The present address (URL) is a permanent link to this revision, which may differ significantly from the current revision.

Welcome to the science section
of the Wikipedia reference desk.
Select a section:
Want a faster answer?

Main page: Help searching Wikipedia

   

How can I get my question answered?

  • Select the section of the desk that best fits the general topic of your question (see the navigation column to the right).
  • Post your question to only one section, providing a short header that gives the topic of your question.
  • Type '~~~~' (that is, four tilde characters) at the end – this signs and dates your contribution so we know who wrote what and when.
  • Don't post personal contact information – it will be removed. Any answers will be provided here.
  • Please be as specific as possible, and include all relevant context – the usefulness of answers may depend on the context.
  • Note:
    • We don't answer (and may remove) questions that require medical diagnosis or legal advice.
    • We don't answer requests for opinions, predictions or debate.
    • We don't do your homework for you, though we'll help you past the stuck point.
    • We don't conduct original research or provide a free source of ideas, but we'll help you find information you need.



How do I answer a question?

Main page: Wikipedia:Reference desk/Guidelines

  • The best answers address the question directly, and back up facts with wikilinks and links to sources. Do not edit others' comments and do not give any medical or legal advice.
See also:


February 21

moving charges

Do accelerating charges produce magnetic field ?? I thought any moving charge would do {biot-savart law} but a man tells me that only charges that move with uniform velocity produce magnetic field. Is it true ? --scoobydoo (talk) 05:43, 21 February 2009 (UTC)[reply]

No, it's not true. As our article on magnetic fields points out, all moving charges produce a magnetic field. A charge that is accelerating is moving and so produces a magnetic field. The gentleman you were speaking to may having been referring to the fact that the Biot-Savart law covers the case of a charge which is moving with a uniform velocity, i.e. one that is not accelerating. - EronTalk 05:58, 21 February 2009 (UTC)[reply]

Then,i suppose, the accelerating particle should produce time varying magnetic field?Do you know any such example ?? i've searched but can't find any example/application of a magnetic field produced by accelerating charges. Also i wanted to know whether biotSavart Law has any limitations,ie whether it can be applied in all situations.{like newton's laws of motion in inertial frames only}--scoobydoo (talk) 10:00, 21 February 2009 (UTC)[reply]

The article, Liénard–Wiechert potential, explains in horrible detail all the mathematics you need to calculate the the electric and magnetic potentials and fields due to a single moving charge with arbitrary velocity. These formulations are essentially representations of the conceptual bridge between Maxwell's equations and special relativity. For any charge q with vector velocity v(t), you can derive an analytic expression for electric and magnetic potentials at all points in space at all times t. The equations dictate the propagation of those fields through time, and implicitly contain a relative velocity with respect to the source particle. Noting that the speed of light is constant, the speed of propagation of the fields is also constant; and this must be seen from all inertial reference frames via the concept of retarded time. Nimur (talk) 13:46, 21 February 2009 (UTC)[reply]
I wondered why the last eight years seemed so long... the concept of retarded time explains it perfectly. arimareiji (talk) 04:23, 22 February 2009 (UTC)[reply]

thanks a lot--scoobydoo (talk) 04:45, 22 February 2009 (UTC)[reply]

by what year is the amazon rainforest expected to be completely logged/destroyed?

when's it gonna happen?Troyster87 (talk) 07:11, 21 February 2009 (UTC)[reply]

We cannot answer "When's it gonna happen?" because the Reference Desk is not a crystal ball. We do not foretell the future. Go find some gypsy for fortune teller willing to take your money and ask him or her.
As for when it is expected to happen... That depends entirely on who you ask. Most people do not expect it to happen. Others will tell you that it will be happen next week. For somewhat sane opinions on the matter, see Deforestation of the Amazon Rainforest. -- kainaw 07:46, 21 February 2009 (UTC)[reply]
It may not happen at all because some areas may be kept as nature reserves or parks, and it may also recover before it's destroyed. More likely for it's complete destruction would be by something like climate change: too much heat or a severe ice age. Looking at the figures from the source of the above article, the mean average annual loss in Brazil from 1988/2006 was 18 122 km2 (344 317 2 / 19). In 2006 there was about 340 0254 km2 left, so at the average rate it would take about 188 more years, or in the year 2194 - tho that's not a good way of estimating because the unknown variables are too many. -- Jeandré, 2009-02-21t08:43z

(collapse action discussed here) --Scray (talk) 16:56, 21 February 2009 (UTC)[reply]


A couple problems with just taking the current rate of deforestation and extrapolating that to determine when all the forest will be gone:
1) As forest becomes more rare, it becomes more valuable, say as a tourist attraction, and thus there will be more incentive to keep it.
2) A deforested are can become forested again. This can happen naturally to an area which is abandoned (let's say it was deforested during mining, then the mines all played out and everybody left). Trees can also be intentionally replanted for future logging, to make a place more attractive to tourists, to stop soil erosion, etc. StuRat (talk) 19:34, 21 February 2009 (UTC)[reply]
And from the opposite side:
  1. The "critical mass" aspect - surely no one would argue that a few square meters of "rainforest" can survive very long. The trouble is, no one knows where the dividing line lies between survivability and eventual extinction. See Population bottleneck.
  2. Technology - Villagers getting firewood don't do nearly as much damage as a farmer clearing land don't do nearly as much damage as an airplane spraying defoliants. Which will predominate is variable, especially in the age of genetically-engineered crops.
  3. Anthropogenic climate change, AKA global warming - a hundred years from now, plains may be coastland and rainforest may be plains. arimareiji (talk) 20:41, 21 February 2009 (UTC)[reply]
3) I'd expect global warming to actually help, as forests will be able to grow where none were possible before, due to permafrost, such as northern Canada and Siberia. We could eventually get forests in Greenland, and maybe even Antarctica, some day. StuRat (talk) 21:28, 21 February 2009 (UTC)[reply]
Not impossible, but rather implausible. And Canada will not grow Amazon rainforest, no matter how much the climate changes ;-). --Stephan Schulz (talk) 23:02, 21 February 2009 (UTC)[reply]
A computer model prediction of global warming and regional temperature rise, assuming a rise in global temperatures of 3C by about 2070 - 2100.
Actually, the Amazon rainforest is widely predicted by computer models on global warming to collapse if global temperatures rise more than 2C (3.6F), and such a temperature rise is likely before 2100. Temperature and vegetation models both predict this, and some of this extra temperature rise in maps is depicted in our article, global warming. In at least one book I've read on GW (and I've read about a dozen by now), the prediction is that once global temperature rise exceeds 2C, a "firestorm", literally a wall of fire, would sweep across the Amazon basin from the northeast inwards, turning the region into desert. In another book, it depicted three possible scenarios in the 21st century: a) the collapse of the Gulf Stream; b) the collapse of the Amazon rainforest; or c) the release of methane clathrates from the seabed. The book claimed that "c)" was the least likely to occur this century, but if peliminary results from the AGU are correct, then the clathrates are already being released. I don't think deforestation will be quick enough to destroy the Amazon rainforest before that potentially happens. ~AH1(TCU) 00:27, 22 February 2009 (UTC)[reply]
Dadgummit, why'd you have to not provide an explanatory link to AGU? :-) arimareiji (talk)
I believe this is it: American Geophysical Union. --Scray (talk) 04:08, 22 February 2009 (UTC)[reply]
"Widely predicted" is too strong. Some models show a catastrophic collapse of the rainforest this century (including the one pictured), but a majority of current models have it continuing to be present through 2100. Eventually climate change may make that biome unsustainable, but there isn't a lot of agreement on how much warming must occur before that happens. Dragons flight (talk) 19:35, 22 February 2009 (UTC)[reply]

Did Ross actually deserve his Nobel in 1902?

Grassi challenged the Nobel awarded to Ross,the India born doctor claiming he had a better claim!Does the wiki community agree?(Ramanathan) —Preceding unsigned comment added by 212.247.70.129 (talkcontribs) 06:57, 21 February 2009

Have you a source that we should look at? Cuddlyable3 (talk) 14:03, 21 February 2009 (UTC)[reply]
From the top of the page: "The reference desk does not answer requests for opinions..." There are other internet sites that are appropriate for discussing personal opinions. The Reference Desk is for answering factual questions. You may be interested in searching for internet forums where topics such as Nobel prizes or scientific claims are discussed. 152.16.59.190 (talk) 04:52, 22 February 2009 (UTC)[reply]
You might want to try your luck posting this question on the entertainment RD, here. Not only would you probably get a better answer, but the entertainment desk looks a little sparse; it could use an extra post or two. -Pete5x5 (talk) 06:41, 22 February 2009 (UTC)[reply]

Scientists with distinguished contribution to literature

Apart from Bertrand Russell who won teh Nobel in literature are ther any other scientists who have distinguished themselves in this manner?(Ramanathan) —Preceding unsigned comment added by 212.247.70.129 (talkcontribs) 07:01, 21 February 2009

In what manner? As far as I know, no other scientist has won the Nobel Prize in Literature. But many scientists have also been successful popular writers. Richard Feynman has written two widely read collections of autobiographic sketches. Stephen Hawking has popularized his research very successfully, and so has Richard Dawkins. Vernor Vinge, probably best known for his acclaimed science fiction, is a computer scientist. Isaac Asimov was a biochemist and held a professorship, and E. E. Smith was a practicing food scientist. John Norman is a professor of philosophy (if you call this a science, and if you consider his writings literature). These are just some examples from my bookshelf (and this may give away too much about my reading habits ;-). --Stephan Schulz (talk) 11:20, 21 February 2009 (UTC)[reply]
Well Bertrand Russell did.and he was a scientist that is why I asked the question.And my colleagues tell me Omar Khayyam was another(Ramanathan)
Robert L. Forward - Famous physicist and sci-fi writer. There are a LOT more. SteveBaker (talk) 14:08, 21 February 2009 (UTC)[reply]
Stephen Jay Gould, evolutionary biologist and paleontologist, perhaps just a famous as a writer of philosophy, c.f. Rocks of Ages. Not a writer, but Alexander Borodin was notable both as a chemist (his work on aromatic compounds was seminal, right up there with Kekule) and as a composer (he was one of the Mighty Handful of 5 great Russian composers. --Jayron32.talk.contribs 19:46, 21 February 2009 (UTC)[reply]
Carl Sagan -- The Demon-Haunted World, Cosmos (book), The Dragons of Eden, Contact (novel) and many more. --NorwegianBlue talk 22:17, 21 February 2009 (UTC)[reply]
And if mere physicians count - we have an entire article: Physician writer. Personal favourites: Oliver Wolf Sacks -- The Man Who Mistook His Wife for a Hat and P. C. Jersild: A living soul and The house of Babel (the latter two, regrettably redlinks, but available in English). --NorwegianBlue talk 22:44, 21 February 2009 (UTC)[reply]
Though I would hesitate to call much of the above "literature"... being a popular science writer, or able to write a witty autobiography, is not at all in the same class as a Nobel Prize in Literature... Ernst Mach, by contrast, was both a famous physicist, but perhaps an even more famous philosopher/author during his day, not writing up popular accounts of science, but making strong arguments about the nature of science and experience, to the point where none other than Lenin himself decided to denounce his work. Gould, Feynman, Dawkins—good writers, all, but their literary works are not exactly world-shaking... --98.217.14.211 (talk) 20:11, 22 February 2009 (UTC)[reply]
Chemist Primo Levi was a literary man. The Nobel prize in Lit isn't the only mark of distinction imo. Julia Rossi (talk) 10:53, 23 February 2009 (UTC)[reply]
C. P. Snow was well-regarded as a novelist, as well as being a physicist; he won the James Tait Black Memorial Prize, one of Britain's most distinguished literary awards and taught at Cambridge University. --Maltelauridsbrigge (talk) 11:48, 23 February 2009 (UTC)[reply]
Empedocles, Aristotle, Francis Bacon, Erasmus Darwin, Miroslav Holub. N p holmes (talk) 13:12, 23 February 2009 (UTC)[reply]

Hybrid eclipse: both annular and total at the same place?

"A hybrid eclipse (also called annular/total eclipse) transitions between a total and annular eclipse. At some points on the surface of the Earth it is visible as a total eclipse, whereas at others it is annular." -- Solar eclipse.

Is it possible to, from one specific place on Earth, see an annular eclipse become a total eclipse (or the other way) if standing at the correct place during a hybrid? -- Jeandré, 2009-02-21t11:14z

The only real option I can imagine is for an observer watching an eclipse (lunar or solar) at the moment of the solar supernova. I assume the pleasure to be rather short lived. --Cookatoo.ergo.ZooM (talk) 13:36, 21 February 2009 (UTC)[reply]
Our Sun is nowhere near big enough to go supernova, please read the articles you link to. --Tango (talk) 15:05, 22 February 2009 (UTC)[reply]
The answer is yes, and in situations much more common than that proposed by Cookatoo. There will be a very narrow region in the path of a hybrid eclipse where it transitions from total to annular (or vice versa). At the midpoint of that region, two minutes of totality would be one minute of an actual total eclipse and one minute of an annular. — Lomn 14:42, 21 February 2009 (UTC)[reply]
The answer is "No." During an eclipse, at any one point on Earth, the Moon covers up a certain percentage of the Sun. Let us suppose the Moon is covering 99% of it, thus presenting an annular eclipse. In order for the eclipse to become total, one of four things (or a combination of them) would have to happen: (1) the Moon suddenly grew larger, (2) the Sun got smaller, (3) the Moon got closer to the Earth, or (4) the Sun got farther from the Earth. Since all four are impossible to the extent needed in the few minutes of an eclipse, it can't happen. B00P (talk) 11:20, 22 February 2009 (UTC)[reply]
Total or annular? WHO KNOWS??? (mysterious echo)
The answer is "Maybe". In the region where a hybrid eclipse was transforming from annular to total, there has to be a gray area of some sort, where Baily's beads become more and more numerous around the sun, leading to ambiguity in defining it as annular or total. Not everything can be pigeonholed so easily :-D -RunningOnBrains 21:30, 22 February 2009 (UTC)[reply]
And by "annular to total", I of course meant the other way around. -RunningOnBrains 01:21, 23 February 2009 (UTC)[reply]
Well, yes, I did think of that knife-edge case - note that I specified 99%, not 99.9999% - but then where does one draw the line between annular and total? So, I'll grant that RunningOnBrains is correct, but still maintain that the answer is "no" in any meaningful way. B00P (talk) 06:03, 23 February 2009 (UTC)[reply]

Oh Jeez - what a lot of crappy answers! No, no, NO!! Think about the reason that some eclipses are annular and some are total. It is because the earth's orbit around the sun is not a circle. It's an ellipse. Hence at some times of year the earth/moon system is closer to the sun and at other times we're further away. Hence the sun appears to be slightly larger or smaller. Hence the moon sometimes completely covers the sun at the moment of 'totality' and at other times of the year doesn't completely cover it. It has absolutely nothing to do with where you are on the surface of the earth. It might seem that the moment just before or just after a total eclipse is "annular" - or if you are not quite in the path of 'totality' that it might seem to be 'annular' but in those cases, the ring of sunlight around the moon won't be a complete circle which is the defining feature of an annular eclipse. A true annular eclipse cannot precede or follow a total eclipse because the distance the earth travels around it's orbit is negligable over the few minutes of totality or annularity. QED. SteveBaker (talk) 14:08, 23 February 2009 (UTC)[reply]

Your FACE is a crappy answer. :-P -RunningOnBrains 19:00, 23 February 2009 (UTC)[reply]
WP:NPA please. SteveBaker (talk) 23:31, 23 February 2009 (UTC)[reply]
WP:NIHEITSIUHTPOWYHBUH please. 79.66.56.21 (talk) 16:54, 25 February 2009 (UTC)[reply]

derivation of e = mc squared

please visit this link : [1]

the e = mc2 has been derived in a very easy way. But my doubt is that the 'm' in the derived equation is actually the mass of the photon. It means that photons and hence, energy has mass. But how can you say that any mass has energy equal mc2? 1kg of gold costs a lot. But 1kg of any mass doesn't cost that much. A photon may have some mass equal to E/c2 . But anything with mass m may not have energy equal to mc2. Please correct me if i am wrong anywhere. I have understood the derivation but i got this doubt. I also know that matter reacting with antimatter produces energy equal to mc2. Please explain my doubt.Also I am just 14 years old and so, please explain in a nice way. --Harnithish (talk) 11:31, 21 February 2009 (UTC)[reply]


Actually, E=m*c^2 only holds for non-moving objects (that is, objects that aren't moving in the reference frame of the person who calculates E=m*c^2 ).

The more general formula is E=gamma*m*c^2, where gamma is the Lorentz factor gamma=1/sqrt(1-v^2/c^2).

Another formula, directly applicable to photons is

E^2=m^2*c^4+p^2*c^2

where p is the momentum of the particle

p=gamma*m*v

In other words, the energy of the photon is not due to its mass (m=0) but due to its momentum.

(Some physicists consider mass to be velocity-dependent and define the relativistic mass m=gamma*m0 where m0 is the rest mass, the mass an object has according to an observer in whose reference frame the mass is at rest. Photons then, have zero rest mass but not zero relativistic mass.)

Summary : no, you can't apply E=m*c^2 to a photon, because the photon is not at rest in your frame. —Preceding unsigned comment added by 81.11.173.78 (talkcontribs) 09:50, 21 February 2009

In answer to the other question (if it is a question), equal masses of substances can have different values despite having the same energy because we can't convert between them. For example, an apple's worth of energy could power vast parts of the world, and therefore be worth a lot. however, we can't convert the apple to energy (yet), so it's not worth that much. - Jarry1250 (t, c) 13:57, 21 February 2009 (UTC)[reply]
We talk about the photon as though it were a little ball that someone could catch in a butterfly net and look at. In fact it is only a conceptual model that has been constructed to satisfy the need to explain how electromagnetic energy propagates seemingly without any mass transfer. Cuddlyable3 (talk) 14:00, 21 February 2009 (UTC)[reply]

what is "Vital fluorescent staining" ?

Google give plenty of search results for "Vital fluorescent staining", but they're all "Application of a Vital Fluorescent Staining...", "Development of a vital fluorescent staining...".

I can more-or-less figure out what it should be from the context in which I read it, and from this wikipedia article: http://en.wikipedia.org/wiki/Staining

Presumably it's some kind of indicator that can be added to bacteria cultures and differentiates between living (vital) and dead bacteria? —Preceding unsigned comment added by 81.11.173.78 (talkcontribs) 09:41, 21 February 2009

Your sense is correct, but this is not limited to bacteria. You'll find a lot of useful information using Google (or other search engine, even Pubmed) with the example of Acridine orange and "vital". I must agree, though, that our content on WP is either hard to locate or lacking. --Scray (talk) 16:12, 21 February 2009 (UTC)[reply]
Vital staining (vital in this case referring to vita, Latin for life/living) of any type is that which is done on live (as opposed to fixed tissue. It can involve chemicals, like acridine orange mentioned above, or the addition of markers (like fluorescently labelled antibodies or tagged proteins, e.g. for FAC sorting) to the outside of the cell. -- Flyguy649 talk 20:59, 23 February 2009 (UTC)[reply]

Hypothetical lifeforms on earth

What might life be like on earth if earth was orbiting a blue giant star instead of the sun? (that is if the earth was far enough away from the blue giant to prevent it from being too hot for life)? Also what might the landscape (aside from the earth having no moon) and life be like on earth (and possibly thea) be like if they never collided 4.4 billion years ago? —Preceding unsigned comment added by 99.146.124.35 (talkcontribs) 10:14, 21 February 2009

Blue giant is just one short stage during the death of a star. Only very massive stars become blue giants and such large stars don't live as long as our Sun. There probably wouldn't be time for life to evolve, at least not complex life. There has been a lot of discussion about what the Earth would be like without the Moon - basically it would less stable (the axis of rotation would move around more) and there would be much weaker tides (there would still be some due to the Sun). It's impossible to say what Theia would be like, it would have to have been thrown out of Earth's orbit somehow and what would happen to it depends on where it ended up. I don't think the Earth would be much different otherwise, it would be slightly smaller, but probably not sufficiently smaller to make a great deal of difference (gravity wouldn't be very different, since its smaller mass would be partially offset by the surface being closer to the centre). The length of a day would be very different (it's difficult to guess what), and the axis might be in a very different place (although it would be moving quite a bit anyway, without the Moon). --Tango (talk) 14:37, 21 February 2009 (UTC)[reply]
Some external sources for the no-moon effect: [2] [3]Lomn 14:38, 21 February 2009 (UTC)[reply]
For any reader unfamiliar with Theia, see the giant impact hypothesis. StuRat (talk) 19:25, 21 February 2009 (UTC)[reply]
The last episode of The Future is Wild, The Tentacled Forest left off at 200 million years later. What might the next lifeforms and landscapes on earth be (assuming that the shows predictions about the first 200 years were correct) after the time when the show left off?--99.146.124.35 (talk) 14:58, 22 February 2009 (UTC) Perhaps 250 million 300 million 400 million years 1 billion years in the future?--99.146.124.35 (talk) 15:02, 22 February 2009 (UTC)[reply]

As to the first question, life would have to adapt to the higher percentage of UV radiation, so you'll see less life in the open and more animals digging themselves in, probably thicker skins and more eye protection. --Ayacop (talk) 16:03, 22 February 2009 (UTC)[reply]

Or maybe just darker skins. Lots of melanin would do the job, you don't necessarily need a thick skin. --Tango (talk) 16:25, 22 February 2009 (UTC)[reply]

Why does the iris have a colour?

I'm wondering why the human iris has a colour. Not so much from a physical standpoint (the wikipedia article on iris colour is quite thorough on that part) but rather from an evolutionary standpoint. Why isn't the iris just white like the sclera? What function do all those colours have? PvT (talk) 16:10, 21 February 2009 (UTC)[reply]

My guess is that it's the same reason as hair color: complex, somewhat linked to other pigments (e.g. skin) and ethnic history/climate, and especially subject to conjecture and junk science. --Scray (talk) 16:40, 21 February 2009 (UTC)[reply]
I realized that answer to part of your question is that the iris is part of the uvea, the pigmented layer between the sclera and the retina. This helps to understand that the iris is part of a pigmented layer of the eye, but precisely why the iris appears the way it does is more complex than that. --Scray (talk) 16:47, 21 February 2009 (UTC)[reply]
I believe the pigment is important for making the iris opaque, this means like only reaches the retina from the pupil resulting in better quality vision. Albinism#Symptoms and conditions associated with albinism might be useful (if you want to know why something is the way it is, it often helps to look at what happens when it isn't). --Tango (talk) 17:38, 21 February 2009 (UTC)[reply]
I understand that the iris is there to regulate the amount of incoming light. However why specifically does the iris have a colour as opposed to being white like the neighbouring tissue. I think Scray helped me out quite a bit. If the iris is an extension of the uvea then it makes sense that it would appear different from the sclera (it's a different type of tissue afteral). What puzzles me is that if the iris is only there to regulate the size of the pupil than why does it need any specific colour. To me it seems that to fulfill this job the iris doesn't need to have any specific colour, as long as it's not translucent. PvT (talk) 17:54, 21 February 2009 (UTC)[reply]
Precisely. It's also fascinating to note how variegated iris color is (just look at the image on that page, or take a close look at a loved one's). Is that beneficial or an accident of embryology? I have a feeling that this could lead into the field of psychology, etc - very complex and currently unanswerable in any concrete way. But certainly worth asking! --Scray (talk) 18:12, 21 February 2009 (UTC)[reply]
The trouble with using albinism as an illustration is that it's not isolated to the iris. The entire pigment layer is important to normal eye development, probably because light bouncing around inside the globe is as (or more) problematic as light passing through the iris. Without separating the effects of light transmission through the iris and light reflection within the eye, the benefits of iris pigmentation cannot be assessed at all. --Scray (talk) 17:49, 21 February 2009 (UTC)[reply]
I'd say that eye color, among many other features, is used to identify individuals. This is important to any social animal, as identifying different individuals is key to survival ("stay away from the one with the dark brown eyes, he's overly aggressive"). And, since humans use visual cues far more than smells or other senses, it would make sense that we would have more visual cues to distinguish one another than other animals that doesn't rely so heavily upon the visual. StuRat (talk) 19:14, 21 February 2009 (UTC)[reply]
Our article on Eye color, not yet linked in this discussion, has some really good discussion of the genetics and physiological aspects of eye color. --Jayron32.talk.contribs 19:37, 21 February 2009 (UTC)[reply]
Genes that influence eye colour are involved in other processes. Consider, for example, HERC2 and OCA2. They are also involved in regulating - among other things - skin colour. So its entirely possible that different eye colours are a selective side effect of pressures on the genes in other functional contexts. Sexual selection may also have driven variation. Note also the the variation in eye colour is largely due to physical, schemochromatic effects. So light bouncing around off structural, protein fibres and a few melanosomes gives eyes a blue colour. These fibres and melanosomes have other biological roles, so it could simply be that their colour is a subsequence of that. Rockpocket 00:59, 22 February 2009 (UTC)[reply]
Having the whites showing seems to be something humans have evolved specially so one can see where another person is looking. Most animals don't have any white showing so it probably is a bad idea normally though I'm not sure why. Presumably for the same reason as the skin is coloured - to protect from UV light. Dmcq (talk) 16:26, 27 February 2009 (UTC)[reply]

Dysfunction in the instinct of nature

So I assume that all species have instincts to protect and nurture their young etc. but was there ever any natural deviant in which any living being seemingly didn't work instinctually and did the opposite? Like instead of nurturing, hindering. If there was how could they naturally bear offspring in the first place if apparently their instincts malfunction? 94.196.9.90 (talk) 17:25, 21 February 2009 (UTC)[reply]

You mean, like parents that eat their young? Another article here. Like ROUS's, I don't believe they exist. ;-) --Scray (talk) 17:46, 21 February 2009 (UTC)[reply]
Every animal is capable of it in conditions of high stress and overcrowding, from what I've heard. Any relationship between this and Jerry Springer modern human society is purely coincidental. ;-) arimareiji (talk) 18:23, 21 February 2009 (UTC)[reply]
One could also claim that the instincts of self-preservation and species-preservation can come into conflict. After all, in times of scarcity, self-preservation may take over. After all, I can always have sex again. But if I don't live through this winter, maybe not... Just a WAG... --Jayron32.talk.contribs 19:32, 21 February 2009 (UTC)[reply]
The goal is always to have as many of your offspring as possible reproduce, but it doesn't matter which ones. If your current young aren't likely to survive that long whatever you do, caring for them is a waste of resources that would be better spent making sure you survive long enough to try again. I think that's the principle behind infanticide in animals. --Tango (talk) 20:53, 21 February 2009 (UTC)[reply]
I recall reading that a relatively large percentage of giant panda mothers either don't know how to care for their young or aren't interested in doing so. I'm not sure if this only applies to those born in captivity, but it's a challenge to increasing their numbers. StuRat (talk) 12:31, 22 February 2009 (UTC)[reply]
Are you asking about species that tend to eat their young, or specific animals that somehow don't have the nurturing instincts shared by the rest of their species? — DanielLC 17:22, 23 February 2009 (UTC)[reply]

Burns

Why is it exactly that the medical community (and, subsequently, the media) expresses burn injuries in terms of percentages? I have read the Wikipedia articles on Burn and on Total body surface area. But, they did not really address my question. When we read a newspaper report of a burn injury, it will invariably state something along the lines of "The victim suffered burns on 78% of his body" (or something like that). Why exactly is this important? Any insights? Thanks. (Joseph A. Spadaro (talk) 22:03, 21 February 2009 (UTC))[reply]

The medical community is probably, for the most part, more concerned with the degree of the burn. But a burn is essentially a wound, an opening in the skin through which infection can travel. So a larger percentage burn means a larger surface area that's open to infection, and a concomitantly greater burden on the immune system to defend. As the burn article notes, "Infection is a major complication of burns". The burn article, and the Servitt article linked from it, indicates that the burn affects the body's fluid regulation systems - bigger burn->greater disruption. 87.112.17.229 (talk) 22:57, 21 February 2009 (UTC)[reply]
The immune system can't the only one that's burdened either. I don't know specifics, but there must be other body systems that are hugely impacted by having to heal such a large amount of skin and other tissues. --Anonymous, edited 00:24 UTC, February 22, 2009.
The reason the percentage is reported is quite simple: prognosis. The larger the area, the less likely that the patient will survive. One formula, for example, where TBSA = total burn surface area is: percentage chance of survival = (100 - (age in years + TBSA)). So a 60 year old with a 30% TBSA has a 10% chance of survival, and a 20 year old with a 30% TBSA has a 50% chance of survival. A 25 years old with a 78% TBSA would by this calculation have a -3% chance of survival - that is, his death would be expected. Obviously these are estimates. To a certain extent, the prognosis will influence treatment by indicating who will be most likely to benefit from intensive specialized burn treatment at a dedicated burn center facility. - Nunh-huh 00:35, 22 February 2009 (UTC)[reply]
That formula says that a 60 year old person with no burns whatever has only a 40% chance of survival, so it seems pretty absurd. Survival for how long? 30 years? Who sits around making up such formulas? Are they based on empirical data or intuition? Edison (talk) 01:22, 22 February 2009 (UTC)[reply]
Obviously formulas for predicting the likelihood of surviving a burn are going to produce absurd results when applied to people without burns. The "survival" in question is survival of the acute burn episode - that is, recovery sufficient to eventually die of something else. Clearly the formulas are based on empiric data - no one would publish a formula based on intuition. For some people involved in using TBSA to predict survival, have a look at "Health Watch; Predicting Burn Survival". New York Times. New York Times. 10 February 1998. Retrieved 22 February 2009., or have a go with the editorial: Saffle, J.R. (5 Feb 1998). "Predicting Outcomes of Burns". New England Journal of Medicine vol 338. pp. 387–388.. Medline searching will also return various papers. - Nunh-huh 01:41, 22 February 2009 (UTC)[reply]
The formula is clearly a first-order approximation of the real formula. It's only going to be accurate near whatever point it was approximated about (which will have been a significant burn area and a non-elderly person - exactly where they approximated, I have no way to know). --Tango (talk) 02:17, 22 February 2009 (UTC)[reply]
And a 110 year old person without burns will have a -10% chance of survival. Sounds pretty depressing to me. --Taraborn (talk) 14:50, 22 February 2009 (UTC)[reply]

Hmmmmmmmmmmm. Maybe I mis-spoke when I worded my original question. I agree with all of the above. And it makes sense that the more burn area (percentage), the more danger to the patient/victim. But, isn't this all of interest and concern to the doctors and medical treatment staff only? Why is this of concern to journalists and the news media (and their audiences)? When I watch or read the news, I never hear anything like: "The victim required 28 stitches" ... or ... "The victim required 7 pints of blood" (etc.). Those details are important to the medical treatment staff, but not important to the television or newspaper audience. So, why is the burn percentage treated differently? Yes, it's important to the doctors and treatment staff to know the burn percentage. But, why is it so important to the general news media audience that it is nearly always reported? That was really what my original question was getting at. Thanks. (Joseph A. Spadaro (talk) 02:42, 22 February 2009 (UTC))[reply]

The number of stitches is of only passing interest medically. Same for units of blood transfused (sometimes, but not consistently of major concern). In contrast, burn area is always important when burn is the primary injury, it is mentioned every time clinicians present a burn patient to one another, and burn centers report it (along with survival rates) to the entities that fund them. So, it's almost always known to the person being interviewed by media. People being creatures of habit, they report it to the media, and the media pass it on. --Scray (talk) 03:40, 22 February 2009 (UTC)[reply]
And since the area burned affects the probability of survival, it is of interest to the news media's audience. Well, maybe not you, but to some of the audience. --Anonymous, 06:53 UTC, February 22, 2009.
Also, even if the audience isn't aware of the relationship between the burn area and survivability, it's still pretty informative: "78%? Geez, that's a lot, that's bad." -- Captain Disdain (talk) 08:07, 22 February 2009 (UTC)[reply]
What the media want is to concisely quote a single, supposedly empirical, value which conveys degree of injury. They really want to say someone was burned 30% to death, or the car crash reduced their life to 25%. Of course you can't really reduce complex medical conditions and their prognoses to a single statistic, which is why it's nearly meaningless; it's just like a report of a fire being "3 alarm" or a report of the "bodycount" of a military operation. And I don't agree that news outlets don't use the stitches count as another false statistic: [4], [5], [6]. or pints of blood transfused [7], [8], [9]. It's the same for the number of bones broken (or in how many places a given bone was broken) during an incident. 87.112.17.229 (talk) 11:44, 22 February 2009 (UTC)[reply]
I still say the formula is extremely silly and could not be based on data. A 50 year old person with no burns, or burns over .0000000000001% of the body has only a 50 % survival rate? Utter nonsense. As for the percentage of body burned, it is not a literal measurement. There are tables stating what percent the legs, arms, back face, or hands are, and it is just added up. There is no literal measurement of square centimeters burned, by careful measurement of irregular burned areas, versus total square centimeters. Edison (talk) 03:18, 23 February 2009 (UTC)[reply]
That formula is simplistic. This guideline is more helpful. Axl ¤ [Talk] 16:42, 23 February 2009 (UTC)[reply]
Here is the original study in the New England Journal. Alternatively, you could use this formula: logit = -7.37 + 0.05(age) - 0.15(year) + 0.11(% body-surface area) - 6.61 x 10-4(% body-surface area - mean % body-surface area)2 + 1.04 x 10-3(age - mean age)2 Axl ¤ [Talk] 16:45, 23 February 2009 (UTC)[reply]
As I explained above, the formula is quite clearly a first-order approximation (you can tell because it's all linear and the true formula obviously can't be since it is bounded). If you try and apply a first-order approximation significantly away from the point it was approximated about, you will get nonsense. --Tango (talk) 16:52, 23 February 2009 (UTC)[reply]

Thank you to all for the input and feedback. This discussion was very helpful. Much appreciated. Thanks. (Joseph A. Spadaro (talk) 05:48, 2 March 2009 (UTC))[reply]


February 22

Cardinal repeatedly flies into window

A male Cardinal is repeatedly flying into my windows, from dawn until evening. There is the fire escape and bamboo outside the windows for the cardinal to sit on. I assume it sees its reflection and thinks its another bird (competition) and tries to attack it. Apparently it's not hurting itself, since this has been going on for months.

I'm thinking of putting up window clings, such as the Halloween decoration ones that might be scary to the bird. But, it's not halloween, so Halloween decorations are probably not available now :( What type of window clings can I get this time of year (in the Maryland/DC/Virginia area) that would scare the bird? and what stores would sell them? Would a craft store (e.g. Michaels) have them? or would Home Depot have something to help solve my problem?

Or what other (inexpensive) suggestions might work to deter the bird from doing this? Any ideas would be appreciated. 75.196.30.110 (talk) 15:24, 22 February 2009 (UTC)[reply]

I have a window sticker in the shape of a raptor, which I got from a pet store (in the same department that sells wild bird feeders and food). It has successfully dissuaded similar bird attacks. 87.112.17.229 (talk) 15:42, 22 February 2009 (UTC)[reply]
I have to assume the bird suffered from brain damage the first time, which might explain why it never learns. :-) How about cat-shaped stickers ? Also, if you have lights on behind the window it will make the reflection less noticeable relative to the light. A more expensive option would be to install louvered window shutters. If you don't care about the bird you could also poison it. StuRat (talk) 16:13, 22 February 2009 (UTC)[reply]
It doesn't matter what you stick on the windows, as long as there is something there. In Switzerland I believe it is required by law to put such stickers on any glass larger than a certain size (they almost all use silhouettes of birds). --Tango (talk) 16:21, 22 February 2009 (UTC)[reply]

Gold

I have always ask this question ,,, why would any one buy gold ,,, what's the point ,,, its not usefull ,,, there's no god reason to buy this metal with that big amount of money . and the most confusing ,,, why countries take the gold as auniversal coin ,,,??? from my opinion gold worth nothing ... as amatter of fact even iron could be more usefull than gold.

That's not true - gold is used pretty extensively in electronics manufacturing - all of the connectors on circuit boards have a thin layer of gold plated on them. The tiny chip of silicon inside a the plastic or ceramic packaging of an integrated circuit is connected to the pins on the outside of the package with thin gold wires. Gold has practical uses. SteveBaker (talk) 19:15, 22 February 2009 (UTC)[reply]
See gold. You will find that there are applications for which gold is useful and for which creates value. If you are wondering why certain metals are precious, you might try reading precious metals. -- kainaw 18:52, 22 February 2009 (UTC)[reply]
Gold:
  1. Does not corrode. Ever.
  2. Is an excellent conductor of heat and electricity.
  3. Has an unusual and generally desirable appearence.
  4. Is relatively rare.
I'm sure there are more reasons, but these qualities alone make gold in high demand for electronics, medical applications, and jewelry. Thus, a high cost. -RunningOnBrains 18:53, 22 February 2009 (UTC)[reply]
Although what the above posters have said is true, Gold is still probably not worth $1000 an ounce. The desirable appearance that RunningoOnBrains mentioned is probably what emotionally influences people enough to want to buy it as a safe haven. I personally would buy honey or something else widely useful which has a very long expiry date and low (or preferably negative) depreciation. See also: diamond-water paradox. This is what the richest man in the world has to say about gold:

Buffett emphasized the non-productive aspect of gold in 1998 at Harvard: "It gets dug out of the ground in Africa, or someplace. Then we melt it down, dig another hole, bury it again and pay people to stand around guarding it. It has no utility. Anyone watching from Mars would be scratching their head."

--Mark PEA (talk) 20:53, 22 February 2009 (UTC)[reply]
To fully understand the value of gold, we should dig back in time to the era before stock markets and internet banking. Creating a universal currency boosted the trade and economy (it's easier to trade with gold, than barter two cows for five sheep, for example) It was rare enough for a small quantity of it to have a good value. If money would be made out of stones, everyone would gather tons of it, leading to an inflation where you needed a wagonload of it to buy a chicken. (With most of our money being created by banks and not having any physical form, this scenario could be not that far from being reached :P ) --131.188.3.20 (talk) 23:22, 22 February 2009 (UTC)[reply]

We just had this question a few days ago on the Miscellaneous reference desk. You will want to read the thread over there. --Anonymous, 00:21 UTC, February 23, 2009.

Actually, the non-utility of Gold is exactly what makes it a good medium for money. If gold had a use, then its utility value may someday exceed its monetary value, which would be bad indeed for the economy. Look at other materials once used for money: Salt was used by the Roman Empire as a means of exchange (hence, "salary"). However, salt is inherently useful in preserving and seasoning foods, and other applications. So if your money is salt, either a) you have to let food rot, because its too valuable to waste salt when you could be spending the salt to buy other things; OR the salt is needed to be used as a preservative, at which point it must be pretty worthless as money, since if it HAD monetary value, you wouldn't want to waste it. Good money has some common properties: 1) It is rare and hard to counterfeit 2) It is otherwise useless. You need both in order to have good money. Salt failed on both counts. Look at wampum as an example of of money. When made by hand, it had both properties, being hard to make, and it also had no use except as money. When the Europeans came along and figured out how to mass produce wampum in factories, it very quickly lost its rarety, and then just became worthless trinkets. Gold, silver, and copper in the ancient world were far to soft to be useful for what you would normally want metal to do, and they were all relatively rare compared to other metals. Thus, they made excellent metals to make money out of. --Jayron32.talk.contribs 03:28, 23 February 2009 (UTC)[reply]
See Gold standard. Cuddlyable3 (talk) 20:20, 23 February 2009 (UTC)[reply]

"Audible Drugs"

I just found this but I doubt that it is a proper name. What is it called? --211.243.241.178 (talk) 18:27, 22 February 2009 (UTC)[reply]

The article you linked clearly names them i-doser. Notice, that link is blue. That means that if you were to type it in the search box on the left and click Go, you would find an article on the topic. -- kainaw 18:48, 22 February 2009 (UTC)[reply]
Hmm, the redirection link was created quite recently (21 Feb 2009) and when I searched it didn't exist. Thanks anyway. --211.243.241.178 (talk) 19:08, 22 February 2009 (UTC)[reply]
And the article doesn't contain any information about them being like drugs, either. --98.217.14.211 (talk) 19:40, 22 February 2009 (UTC)[reply]
So... it means that some kids decided to take something that is well over 100 years old, give it a new name, and hype it up to be something it isn't. It isn't all that surprising, is it? -- kainaw 21:24, 22 February 2009 (UTC)[reply]
Without doubt, certain music and audio can create a psychological effect. But unless it creates a psychoactive effect with measurable neurological effects, I don't think it can be classified as "drug-like." Lots of behaviors can create feelings of euphoria or elevated perception without being "drug-like" ... From our article drug,
Evidently, until a reliable medical, legal, or pharmacological source makes a specific mention of audio-induced psychoactive response, these sorts of things will not be widely considered "drug-like." Nimur (talk) 15:37, 23 February 2009 (UTC)[reply]

Bubble´s prediction

What is the best way of predicting bubbles? Is any price increase x times above average inflation a sign of one?--Mr.K. (talk) 19:37, 22 February 2009 (UTC)[reply]

I'm pretty sure if there was some tried-and-true way of predicting bubbles, fewer people would be caught out when they burst. --98.217.14.211 (talk) 19:54, 22 February 2009 (UTC)[reply]
Do you have an infinite believe in human intelligence? Would people use a tried-and-true way of predicting bubbles or would they believe that they use an-even-better-tried-and-true way of predicting bubbles? Anyway, I want to know what are the signs of bubble forming, not necessarily how to predict when it will explode. --Mr.K. (talk) 20:00, 22 February 2009 (UTC)[reply]
If you could predict whether something was a bubble, you could make an awful lot of money. There was a very big and rapid one last summer. A certain Mr. Soros claimed oil was a bubble at $135 a barrel [10], it peaked at $147 per barrel (pb) and is currently at $39pb. I think the low since $147pb is about $30pb, so had Soros sold short 10,000 barrels at $135pb and bought back at $35pb, he would have made $100 * 10,000 = $10,000,000. I don't know whether he did, and if he did, what quantity (probably alot more than 10,000 as $10m profit is kinda small change to someone worth $9 billion). --Mark PEA (talk) 20:42, 22 February 2009 (UTC)[reply]
You can tell a bubble situation when the price is well above what it is worth, and it is purchased because of the belief that the price will rise. It is harder to notice then the pop will happen. But a good clue is when the item become unaffordable for those that have to buy it. Graeme Bartlett (talk) 20:50, 22 February 2009 (UTC)[reply]
Indeed, something that is supposed to serve a purpose being traded more by speculators than by consumers is generally a good sign of a bubble. --Tango (talk) 22:00, 22 February 2009 (UTC)[reply]
I disagree with your logic. Saying "a price is well above what it is worth" is impossible to know. No one can estimate the price consistently better than the market can. Otherwise they'd be filthy rich. So the best estimate most people (like the OP) can use for the price of a good is simply the market price. Thus the price is never more than what it is worth. Speculators aren't dumb. They buy when they think prices are going to rise. Often the speculators are right (or else they would have lost all their money long ago). Despite the portrayal of oil speculators recently by the media, speculators don't "artificially" increase the price. They increase the price based on uncertain factors which could potentially make the stock worth quite a bit more. When google offered their IPO, if I buy it because I think they'll do well their first year, that is speculation, but it also moves the price to where it SHOULD be, I'm not artificially increasing the price.
To answer the OP's question, you can't always tell by the price if something is a permanent change or temporary change no matter how much it changes. The best way to see a bubble coming is to look for factors that would shift a price from what it would be in a fair market. In a true free market there would not be bubbles. Most bubbles are caused by politians trying to manipulate the economy. Both President Bush and President Clinton made speeches, especially when addressing hispanic audiences, about trying to make it easier to get low income and minorities into affordable homes. By the Federal Reserve artificially adjusting the interest rates and passing laws to incentivize first time home buyers they caused the price to increase far above what market value would have otherwise been. Now that President Obama has passed the bailout which gives a $8,000 tax credit to any first time home buyers during calendar year 2009, again it is pushing the price far from the market value and as such is a very dangerous move.Anythingapplied (talk) 07:29, 23 February 2009 (UTC)[reply]
If you buy Google stock because you think Google will do well next year (fundamental analysis), that corrects the price. If you buy it because you think the stock price will go up, with no regard to the company, (technical analysis) then that "artificially" changes the price and results in bubbles. Saying "price greater than worth" doesn't make much sense without clarification. To calculate worth, in this context, you need to look at what the item is useful for and compare it to possible replacements. If genetically engineered blue rice is 1000 times the price of regular rice, despite it going just as well with a Chicken Korma, then that's a clear sign that there is a bubble on blue rice. People aren't buying it because they think it's worth that amount of money, they're buying it just so they can sell it later after the price has gone up. --Tango (talk) 12:12, 23 February 2009 (UTC)[reply]
Eye of newt, and toe of frog, wool of bat, and tongue of dog work for me. (Guaranteed to double, double your investment, without toil and trouble.) Bill "Lowest Commission" Shakespeare (talk) 08:13, 23 February 2009 (UTC)[reply]
I didn't know that there was actively traded market for those commodities... Dragons flight (talk) 15:43, 23 February 2009 (UTC)[reply]
The trouble is that it's a self-non-fulfilling prophesy. If you have an equation that predicts the moment when the bubble will burst, investors will buy stock knowing that the bubble is still growing and that they can safely sell on the trading day immediately before the equation says it'll burst. However, if the equation is sufficiently widely known that people would tell you about it on the Wikipedia reference desk then you can bet that all serious investors will know it. Hence they'll all sell their holdings on the day before the bubble bursts - the stock price will tank because every big investor dropped their holdings in the stock at once - and the bubble bursts a day before the equation says so (and if everyone knows that - then two days before - or a month before...) - and the equation is thereby proven useless. So not only is there no such equation - but there cannot ever BE such an equation so long as investors are aware of it and are able to take advantage of it. SteveBaker (talk) 23:29, 23 February 2009 (UTC)[reply]
OK, there is no way of predicting when it will burst. However, a partial question is how can we know the market is building a bubble? (if we can at all know that a bubble is growing).--Mr.K. (talk) 09:20, 24 February 2009 (UTC)[reply]
@Mr.K.: You can't know. You are effectively asking the question "What will the price of X be in the future?", but in an indirect fashion. If anyone can figure out that something is a bubble, but not know when it bursts, you can short sell it. However this assumes that when the bubble bursts, the price will be less than that you shorted it at. --82.21.28.65 (talk) 18:04, 24 February 2009 (UTC)[reply]
@Steve: That sounds like a variation on the unexpected hanging paradox. --82.21.28.65 (talk) 18:04, 24 February 2009 (UTC)[reply]
(Which is perhaps the only paradox that really seems paradoxical to me...the others seem entirely trivial to dispel. The only halfway reasonable way to explain it away is to assert that 'unexpectedness' is not a binary thing - there are degrees of unexpectedness. However, one would expect an equation to produce a definite answer. But you're right - and it could be that a 'when will the bubble burst' equation would be possible if it only predicted a statistical likelyhood of the bubble bursting at any given moment. As the predicted probability of the 'bust' happening increases (per the equation) - the risk-averse investors would pull out first with fewer and fewer of the high-risk-tolerant people staying around...that would allow the equation to predict the probability of a bust on any given day - despite all of the investors knowing about it.) SteveBaker (talk) 21:14, 24 February 2009 (UTC)[reply]

It's easier to spot a bubble from outside, rather than being inside the bubble. eg Most people working in McDonalds etc would be fairly aware that profitting by buying and selling commodities without adding any value to the commodity cannot continue forever. In other words look for the price of something increasing for no good reason beyond that explained by common or garden inflation. —Preceding unsigned comment added by 213.249.232.187 (talk) 15:51, 24 February 2009 (UTC)[reply]

electrical conductivity

I would like to measure the electrical conductivity of distilled water versus ionized water (for my sister's science fair). How would you do this with a voltmetre? I tried sticking a copper penny to one end of the voltmetre and a zinc nail to the other and putting this into the solution. Would this work? —Preceding unsigned comment added by 70.52.47.166 (talkcontribs) 15:25, 22 February 2009

No, that measures (approximately) the amount of voltage generated (you are creating a battery), not the amount of voltage blocked (the conductivity/resistance). to measure resistance, you need an external source of power and then measure how much of it is lost/blocked by the liquid you are testing. DMacks (talk) 20:36, 22 February 2009 (UTC)[reply]
You are measuring the voltage from a small cell. However since you want resistance, you need to use an Ohmmeter not a voltmeter. Your two electrodes should be the same as each other and not easily reacting. Two copper or two gold electrodes could be the way to go. The conductivity will depend on the geometry of electrodes and temperature, so record these. Graeme Bartlett (talk) 20:43, 22 February 2009 (UTC)[reply]
(ec) I'd say no without knowing what you mean by "ionized water" (Our article "Water ionizer" looks like voodoo baloney to me.). Electrical conductivity is the inverse of electrical resistance (ohms). With your dissimilar metals, you would be demonstrating something else, the galvanic cell. To measure conductivity, you could measure the resistance with the multimeter and divide that into 1, maybe using two long pieces of tubing, one for each water. If you really do have a voltmeter, you could measure the voltage across a resistor in series with a battery and the water, then use Ohm's Law. Two things: distilled water will insulate like glass, and salt water will have practically no resistance (extremely low conductivity and extremely high, respectively). Oh, one more thing: you can't get anything at all in the distilled water, not dust, not your finger, not a drop of tap water, not a probe that has any salt water on it. --Milkbreath (talk) 20:59, 22 February 2009 (UTC)[reply]
I suspect that the original questioner meant Deionized water, since ionized water doesn't make much sense. edit: Unless the purpose is debunking the aforementioned water ionizers. I'd expect the conductivity of distilled and deionized water to be very similar. --NorwegianBlue talk 22:02, 22 February 2009 (UTC)[reply]
Careful with copper or other metals that can do redox chemistry...you might wind up transferring the metal atoms from one electrode onto the other. In the process, you get ions of the electrode itself in solution, which alters the ionic content of the water. DMacks (talk) 22:52, 22 February 2009 (UTC)[reply]
There are special things called Conductivity cells. From memory, they comprise a cylindrical vessel that has 2 conducting metal rings on the inside surface a small distance apart (could be 1cm but don't quote me). It has 2 terminals on the outside to which you connect an ohmmeter. I think the cell is rigged to give a direct reading of conductivity in Siemens.

Thank you very much. BTW, by ionized water I just meant water with salt in it.


February 23

Red Hot Ice Cube

Here is an interesting found on the internet. Since there are lots of people smarter than me here, I'd like your thoughts. It is supposedly a video of an ice cube glowing red hot due to induction heating. For now I will withhold my thoughts, since I'm not particularly familiar with inductive heating. Thanks :) --Bennybp (talk) 03:56, 23 February 2009 (UTC)[reply]

Induction heating affects metal objects - not ice. I've seen demo's of inductive kitchen ranges where someone can put their hand on the range top with it turned on - and a pot held an inch or so above the hand would still heat up. So I'm not even slightly surprised that the ice doesn't melt. However, the red glow and the flames must be coming from something ELSE inside the inductive loop. SteveBaker (talk) 04:24, 23 February 2009 (UTC)[reply]
I'm surprised the ice didn't melt - if something in the system is getting hot enough to glow bright red and flame, there should be enough heat around to melt the ice. Ice on its own wouldn't melt, but that looks like ice in contact with (or, at least, very close to) metal - the metal would be heated by induction and it would then heat the ice. --Tango (talk) 13:24, 23 February 2009 (UTC)[reply]
Ice doesn't conduct heat very well - so the surface of the block is obviously going to melt - but s-l-o-w-l-y because it's a big block. This is why it's so important to defrost your thanksgiving turkey before you cook it - even though the oven is set at hundreds of degrees the ice inside won't melt quickly enough. In this case, little or none of the induction heaters' energy will be transferred into the ice because induction cookers rely on heating up the metal pot rather than heating the food directly as a microwave oven would. So the ice obviously does melt - but (evidently) slowly enough to not be noticeable over the few seconds it takes an induction heater to get red hot in the video clip. At any rate - production of light due to heat follows a fairly simple relationship between temperature and color. For the ice to be glowing red hot, it would have to be up at several hundred degrees - which would (of course) mean that it would have to be steam. Since it's clearly still ice, it is DEFINITELY not glowing. It's simply refracting the red light from whatever is glowing beneath and off to the sides. There is nothing weird or special going on here - it's just another typical bullshit-faked-science fraud that you see on YouTube all the time. Please - everyone - IGNORE "SCIENCE" VIDEOS ON YOU-TUBE - they are almost all stupid people out to trick you and spread disinformation. SteveBaker (talk) 13:56, 23 February 2009 (UTC)[reply]
Yes, of course the ice isn't glowing red hot, but if it is that close to something that is growing red hot I would expect it to melt faster than that. That kind of bright red hot corresponds to about 1500C, if memory serves - far hotter than an oven (unless you were right up close to the heating element). Is it possible the whole thing is faked and there is just a red LED under there? --Tango (talk) 15:58, 23 February 2009 (UTC)[reply]
I think Steve is being unfair. There are lots of good and very legit physics and chemistry demonstration videos on Youtube. There is junk too, so Steve is right about keeping your sceptic hat on. Besides content, things that tip me off to potentially suspicious "science" videos are handheld camera work (versus a steady tripod mounted camera), non-lab equipment (ex: using kitchen measuring cups instead of beakers), lack of expected safety equipment such as goggles, lab coats, gloves or plexiglas shields (whatever is relevant), "at home" filming versus in lab filming, youthful age of the commentator (not a very reliable predictor), and the maturity and "scientificness" of the dialogue. There is a very real chemistry demo superficially similar in appearance only to this which is the combustion of dry ice (carbon dioxide) by magnesium. You can see it here The ice though in this "ice induction video" evidenced by the liquid around it and the transparency is most likely H2O. If the ice itself is being heated to what the black body color suggests, then part of it should have flash boiled and there would be steam everywhere (not just a little). There may very well be some metal under the ice and the ice is put on top of it for aesthetic purposes. If that's the case it's a very poor design because it conveys the wrong idea.152.16.253.109 (talk) 22:46, 25 February 2009 (UTC)[reply]
Cheap trickery. Easy to do. I am most certain it is a metal object inside the ice, below the rim of the induction ring. Once the metal object heats up and starts to glow, the whole ice cube glows. It can take several seconds before the ice starts to melt fast enough not to be noticable, but it was on for mere seconds in this video anyway. My conclusion is it was simply modern day smoke and mirrors: camera angles and deliberate cuts.
Microwave ovens heat by induction, don't they? And they heat nonmetallic objects. Maybe the difference is a matter of frequency. There is a freuency allocation in the rf spectrum for microwave ovens.-—Preceding unsigned comment added by 98.21.104.187 (talkcontribs)
Yes, but the point is, an ice cube would not glow when heated. It would melt. Then boil. Then maybe, if the conditions were right, glow.-RunningOnBrains 14:41, 23 February 2009 (UTC)[reply]
No - microwave ovens use the dielectric effect to heat water - not induction. The dielectric effect doesn't work well on ice either - although it's highly effective on liquid water...which is why 'defrost' mode on your microwave turns the oven on and off repeatedly so it can heat the surface layer of water WITHOUT boiling it - then let that heat melt some of the ice into cold water - then it turns on again to heat that water and thereby slowly melt the ice. So no - that doesn't help the case for this crappy video (Also - where do the flames come from? Water doesn't/cannot-possibly burn!! There MUST be something else in there doing that). SteveBaker (talk) 14:55, 23 February 2009 (UTC)[reply]
I noticed the "burning" too, but thought it might just be a little bit of steam being colored by the red hot "ice." Of course, where does the steam come from? I agree that it is likely some metal in the ice. A one point, it seems like maybe the ice cracked somewhere, letting the steam out.
The demonstration might have been honest enough (showing how inductive heating can heat metal, even through ice) with some cool effect in the ice. But likely the person uploading the video misinterpreted. I agree about the science videos on YouTube - The only thing worse than the videos are the comments on science videos, but I digress --Bennybp (talk) 15:59, 23 February 2009 (UTC)[reply]
There are also videos on "Hot Ice", where a substance is added to hot water to make it freeze when poured. ~AH1(TCU) 00:12, 24 February 2009 (UTC)[reply]
"Freeze" or "solidify"? --Tango (talk) 14:59, 24 February 2009 (UTC)[reply]
I bet by hot ice you mean the supersaturated sodium acetate solution demo seen here. There are lots of great science videos on Youtube. There are more good ones than bad ones, and half the bad ones are just poorly or wrongly explained rather than flat out fabrications. You just have to watch out for the bad ones. I have to agree with you on the comments though. The ratio of stupid to useful/intellectual comments is too high for me to tolerate. 152.16.253.109 (talk) 23:02, 25 February 2009 (UTC)[reply]

http://www.huettinger.com/en/about-us/multimedia.html <--- this. 2secs on google

Although having a metal ball in the middle of the ice cube is more plausible, could the frequency of the AC current in the inducting coil not be set to the resonant frequency of a certain electron state in the H2O molecule, exciting them more than the electrons in the hydrogen bonds that hold the lattice together? Then these electrons drop down energy levels again, releasing photons, before the electrons in the H-bonds get excited enough to start the melting process.

Oil Platforms

The platforms are structures which including one or more systems. Oil platforms are used for processing the crude oil

I've put links in your statement to help your search. Do you have a question about your homework? Julia Rossi (talk) 10:59, 23 February 2009 (UTC)[reply]

--Scray (talk) 16:56, 21 February 2009 (UTC)[reply]


battery life in laptops

Hello Wikipedia.

I got told by a stranger in the library today that if my laptop was plugged in, it is better for the battery if it is taken out. Anyway, for those who know more about these things than i do, is this likely to be true? My battery is a recharable Li-Ion (for a samsung Q310) if it makes any difference... thanks81.140.37.58 (talk) 11:23, 23 February 2009 (UTC)[reply]

Essentially the answer is...no. Modern Lithium-ion battery batteries don't have to have the full-drain-recharge cycle that older batteries do, so you don't need to worry about your laptop battery being connected when you are using the mains-supply. Someone will explain the science behind why i'm sure. Maybe try Lithium-ion battery for starters. 194.221.133.226 (talk) 11:58, 23 February 2009 (UTC)[reply]

update: though a link in the article suggests that by removing the battery when using the power-supply it can help keep the battery cool which will reduce the degredation of the battery. 194.221.133.226 (talk) 12:02, 23 February 2009 (UTC)[reply]

My laptop manual similarly suggests that it is better not to leave it on continuous charge. I have ignored this advice, but I notice that the battery capacity has reduced by 70% over 18 months. Could I have extended the battery life by fitting it only when I needed to either use or recharge it? Dbfirs 13:18, 23 February 2009 (UTC)[reply]
If you have a MacBook, you actually have to leave the battery in or processing speed drops quite a bit. No idea why. arimareiji (talk) 13:51, 23 February 2009 (UTC)[reply]
(That's probably using a SpeedStep technology to slow the processor based on a reading of "low battery voltage" - but if the battery is removed, it is not clear why the sensor would be active, nor why the measured voltage would be isolated from the AC-DC power-supply's standard voltage. Complex systems fail in complex ways. Nimur (talk) 15:58, 23 February 2009 (UTC)[reply]

Edison,Tesla spat cost them a Nobel !!!

Isaac Asimov has calimed that bioth Edison and Tesla were supposed to share the Nobel in 1915 but niether was willing to shar the award with teh other hence it was awarded to Braggs -father and son duo who never came close to the two in terms of their acievement.Is this true?(Ramanathan)

Likely not true. The nobel is usually awarded for novel discoveries, and neither was much of a scientist. True, they both improved existing technologies, but that isn't what the Nobel is normally awarded for. They were successful entrepreneurs and "inventors," but scientists they were not. --Jayron32.talk.contribs 12:29, 23 February 2009 (UTC)[reply]
Actually, the physics prize is also awarded for technical work, and was in particular during the early years (e.g. Gustaf Dalén). And Tesla did in fact do important theoretical work. --Pykk (talk) 12:36, 23 February 2009 (UTC)[reply]
This is mentioned in Nobel Prize controversies. However, I spot several issues here. Number one is that the prize winners are simply never consulted about sharing an award, or indeed consulted at all, before they win the Nobel. The second issue is that sources seem to be contemporary rumors in the press. That's a bad source. The Nobel committee work is kept secret for 30 years (IIRC), so if Tesla and Edison were under real consideration, it'd be relatively easy to come up with real documentation of it. Also, it's worth noting that Tesla and Edison were big celebrities in the USA - whereas the Nobel committee in Sweden would be more aware of European innovators in the area, such as Mikhail Dolivo-Dobrovolsky and their own Jonas Wenström, both of whom independently developed three-phase AC systems within months of Tesla's work. Finally, Bragg was indeed deserving of the prize. --Pykk (talk) 12:34, 23 February 2009 (UTC)[reply]
Lest you underestimate the value of the Braggs' prize, note that it was for a technique that has been of immense value for physics, chemistry, and biology since then (the discovery of the structure of DNA hinged on it as well). Whether you think Tesla/Edison should have gotten a prize does not distract from the importance of the Braggs' work. --98.217.14.211 (talk) 13:52, 23 February 2009 (UTC)[reply]

The 1915 Nobel prize being offered to Tesla and Edison claim has been debunked by someone with access to all the Nobel Committee files. The talk page archives for the Nikola Tesla article say: Talk:Nikola Tesla/Archive 6#Nobel claims

"Another verifiable site for the initial incorrect report is the New York Times, Nov 6, 1915, p1: "Edison and Tesla to get Nobel Prizes" which based their statement on the Copenhagen correspondent of the Daily Telegraph. Also see Nov 7, 1915, p12, "Tesla's discovery Nobel prize winner". The article said "Nikola Tesla, who, with Thomas Edison is to share the Nobel Prize in physics, according to a dispatch from London, said last night he had not yet been officially notified of the honor. His only information on the matter was the dispatch in the New York Times." Tesla thought the honor was for the transmission of energy without wires. He said he thought Edison was worthy of a dozen Nobel Prizes. He had often expressed his friendship with and admiration for Edison, and gave no hint he would refuse the honor if Edison was also getting it. The reported antipathy toward Edison only showed up when he was elderly. Finally Dec 28, 1915, p83 the NY Times reported that the initial report was incorrect. It had also given an incorrect report for the person to receive the chemistry award, further disproving the claim that Tesal refused the prize because Edison was also getting one. Also disproof of any great antipathy between them causing them to refuse the 1915 Nobel Prize is that in 1916 Tesla accepted the Edison Medal'For meritorious achievement in his early original work in polyphase and high-frequency electrical currents.' This often repeated claim that Tesla refused a Nobel prize needs something like a rejection letter from the Nobel files, or a memoir by someone involved with the Nobel award, or papers from Tesla's files. In other words, something more than someone writing a book or creating a webpage and stating it is so without a good source. Edison 23:48, 17 July 2006 (UTC) There is not much room for controversy. Seifer in "Wizard" pp378-380 has obtained detailed info from the Nobel people. Tesla never received a Nobel nomination in 1915, so he did not refuse it. Neither did Edison. Out of 38 nominations for the prize in physics, Edison received one and Tesla zero. The winners were the Braggs, father and son. There was an erroneous press report as listed above. Tesla also received one bid out of 38 in 1937, again not enough for him to win the prize. No nomination, no controversy, other than perhaps that he might have deserved one.I am removing refrence to the 1915 Nobel Prize and Tesla.Edison 22:56, 22 July 2006 (UTC).

. Edison (talk) 20:31, 23 February 2009 (UTC)[reply]

Better luck next year, Edison DMacks (talk) 20:48, 23 February 2009 (UTC)[reply]
Yuh just haveta wait. And hope. Edison (talk)

To the original poster: I would be very surprised if Isaac Asimov made any such claim. Where did you see this? --Anonymous, 05:29 UTC, February 24, 2009.

Manual gear

I drive amanual gear car ,,, it's annoying how you should reach that balance

between the clutch and fuel at start of movment , specially at crowded streets,

i'am wondering , was it to difficult for them to make a medium gear which can

transfer the power from the engine to the gear to the tires smoothly at movement

starting without that precious driving actions ... its Twinty-one century ,,,  ???

Smooth transfer of power by manual gear-change improves with practice. Perhaps you would prefer a car with automatic gear change, where twenty-first century technology helps you to transfer power smoothly. I recall a continuous gear on some Daf vehicles in the mid-twentieth century which achieved the smooth transfer using a belt drive without separate gears, but I presume it had significant disadvantages. Does anyone know what happened to that technology? Dbfirs 13:14, 23 February 2009 (UTC)[reply]
That would be the Variomatic transmission, a type of Continuously variable transmission. DuncanHill (talk) 16:10, 23 February 2009 (UTC)[reply]
There is such a thing as a Semi-automatic transmission, where you control the gear changes as in a manual, but the clutch is handled for you - you just pull on a paddle on the steering wheel to change gear, usually. --Tango (talk) 13:19, 23 February 2009 (UTC)[reply]
The problem with what the OP is requesting is that a 'clutch-less' transmission relies on a kind of 'sloppy' coupling between engine and gears or between gears and drive-shaft. In an automatic (or semi-auto) gearbox, that's typically handled with a 'fluid clutch'. The problem with fluid clutches is that they are the precisely the reason you didn't want a fully automatic in the first place - they take a while to 'catch up' and that means that you have less acceleration and you waste fuel compared to a manual transmission being driven by a competent human.
Going to semi-automatic doesn't help. Essentially, a semi-automatic transmission is just a fully automatic transmission where you give hints to the on-board computer about when you'd like it to shift. My wife's MINI Cooper has that and if you try to over-rev the engine or stall out or lug the engine due to too few revs, the computer just quietly takes back control from the driver and sneaks back into fully-automatic until you get it right! Semi-automatic is the worst of both worlds in that a poor human driver can persuade the car to be in the wrong gear for much of the time - yet you still suffer all of the losses of a fluid clutch.
The idea that I love is currently (I believe) only available on the MINI Cooper - but for some entirely arbitrary and stupid legal reason - not in the USA. The UK version of the MINI turns off the engine when you come to a stop. When you start to move off the line, it uses the starter motor to get the car rolling again - and doesn't restart the engine unless you stomp on the gas - or until you are going faster than about 10mph (or if the battery voltage starts to get low). This means in annoyingly slow stop-start traffic, the car almost becomes an electric car - turning back into a gas-powered vehicle only once you need the speed and/or acceleration. That's such a simple trick - requiring almost zero additional parts in the car (it's mostly a software trick in the engine management computer) - I can't believe that all modern small cars don't have that.
Sadly - in other cases - the alternatives are to become better 'in tune' with your car so that you can get the clutch control just right - or to give up and drive an automatic.
SteveBaker (talk) 13:36, 23 February 2009 (UTC)[reply]
A semi-automatic does have some advantages over a fully-automatic - in particular, you can anticipate the need to change gear (just before you pull out to overtake, say). You don't get the fuel savings, though, you're right. --Tango (talk) 13:53, 23 February 2009 (UTC)[reply]
Yes - I agree. And in really top-notch cars the paddle-shifters can shift gears faster than you could in a stick-shift car, so there are some benefits to be had there in terms of not using engine power to uselessly spin the engine in the brief moment when you have the clutch pedal on the floor. I haven't driven semi-automatics much - but in playing around with my wife's new MINI over the weekend, I found that putting it in full automatic mode and just hitting the paddle shifter to drop a gear when you want to overtake or something was a pretty neat compromise. It has three modes - full automatic (with paddle shifter 'override'), semi-automatic-for-morons (where you shift with the paddles - but if you try to push the revs too high or too low it takes over and shifts for you) and semi-automatic-for-real-men (where you shift with the paddles and the car does what you tell it no matter what). But I'm a stick-shift enthusiast - my car is manual. SteveBaker (talk) 14:47, 23 February 2009 (UTC)[reply]
On my first driving lesson the instructor took me to an up-going hill. He made me hold the car stationary on the hill without using brakes, just sliding the clutch. At first I was clumsy, either over-revving the engine or stalling it. Finally I mastered the "feel" of the clutch and could even drift the car up or down at will. After that practice, smooth start-ups become instinctive and you have better control than any automatic transmission. But start a different car, such as a hired car, and it takes a little while to feel out the unfamiliar clutch.
Yes - you are quite likely to wear out the clutch while you're learning. If you can find a driving instructor with a stick-shift car who'll give you a couple of lessons - that's the best way to learn. I haven't seen stick-shift rental cars for a very long time - but if you can find one - that's better still. SteveBaker (talk) 22:36, 23 February 2009 (UTC)[reply]
The reason few cars have SteveBaker's electric start aid is that it would quickly wear out a conventional starter coupling gear and motor. The starting cycle is predetermined so I wonder how well it works in all conditions of hills and battery charge. These problems are worse for heavier cars and/or larger engines that need more torque to crank.Cuddlyable3 (talk) 19:06, 23 February 2009 (UTC)[reply]
It's not 'mine' - it's BMW's - and it's not hypothetical - it actually exists in real production cars. Of course the MINI is indeed one of the smaller/lighter cars available in the USA - but it's pretty normal by European standards. But bigger cars have bigger engines and therefore bigger startermotors and bigger gears - so there need not be a problem for larger vehicles. I suspect the reason most other cars don't have it is that without the MINI's Gasoline direct injection gizmo there is a risk of flooding the engine or something. The amount of gas you save on in-town driving by doing this is pretty spectacular - so even if a slightly meatier starter and coupling gear is required - it's still a pretty good deal. I presume the car can measure whether it's on a slope and whether the battery is getting low - and in those cases, it can simply start the engine as a number of cars that turn the engine off while idling do. I'm pretty sure the MINI knows when it's stationary on a hill because it knows to hold the foot-brake on for three seconds after you take your foot off of it so you can do easy hill starts without messing around with the hand-brake. Sensors for things like tilt and rotation are becoming very cheap these days - compared to tens of thousands of dollars for the car - a $5 sensor is nothing. The MINI already has yaw rate sensors and all manner of other stuff for figuring out what's going on. SteveBaker (talk) 22:36, 23 February 2009 (UTC)[reply]
I'm wondering why nearly every car in the USA has automatic gear, while in Europe most of the cars have manual gear. --131.188.3.20 (talk) 12:05, 24 February 2009 (UTC)[reply]
I don't know if (or why) that's true in Europe in general - but there is a REALLY good reason why it is so in the UK. In the UK, there are two different drivers licenses - one that permits you to drive any kind of car - and a lesser license that only allows you to drive automatics. In order to get the 'any-car' license, you have to pass your driver's test in a stick-shift car. Since hardly anyone wants to be limited as to what they could drive in the future - or to have to take a second drivers test later in life, nearly everyone learns to drive stick and passes their test in a stick-shift car. Almost the only people who have the automatic-only license are disabled people who for whatever reason are physically incapable of driving stick. I don't think I know anyone in the UK who has the automatic-only license. Since new drivers are generally a little nervous, the first car they buy tends to be a stick-shift car because that's what they are used to. This then becomes the habit of a lifetime. That in turn means that there is a much higher demand to manual transmission cars - there are more manual transmission cars on the used car market and so forth. Because car manufacturers have to go to some extra trouble to make cars with the steering wheel on the right hand side for the UK market - they make relatively few automatics - and the price of automatics is generally considerably higher than manuals. This feeds back still more because now you have a community of drivers who almost all are quite happy driving stick - and a car market where automatics are a lot more costly. Another factor is that gasoline prices in the UK have always been VASTLY higher than in (say) the USA - since automatics get worse MPG than a properly driven manual gearbox car, this is further pressure to drive stick. SteveBaker (talk) 13:14, 24 February 2009 (UTC)[reply]
Not in Europe obviously but NZ also has quite a lot of manuals. In NZ you can only drive an automatic if you pass the test under one while you have your probationary license. But once you have a full license you can drive either car no matter what you pass your full license test under. A bigger factor may be that NZ's car fleet is very old with an extensive used car market (many coming from Japan, Singapore and other countries with strict regulations and probably left hand drive where people don't keep their cars long). Manual cars tend to last longer and are cheap to maintain with fewer things that can go wrong. Of course we're ultimately also somewhat limited by what is available on the world market. Nil Einne (talk) 14:20, 24 February 2009 (UTC)[reply]

increase of relative mass

I have understood the time dilation and length contraction in special relativity. But i don't understand the mass increase.can someone explain why the mass of a body increases relatively when it is moving and why only by the formula in special theory of relativity? i am a ninth grade student. please explain accordingly.--harish (talk) 15:34, 23 February 2009 (UTC)[reply]

Short answer: When an object is moving it has kinetic energy, energy and mass are just different ways of looking at the same thing (E=mc2), so when you increase the energy you increase the mass. (It's worth noting, most of time when we talk about mass we mean "rest mass", which is the mass when the object isn't moving - that is constant. The concept of "relativistic mass" isn't actually very useful when doing calculations, etc., so it isn't used much.) --Tango (talk) 15:53, 23 February 2009 (UTC)[reply]
I have a feeling that you are referring to the common note that a person cannot reach the speed of light because as energy is added to increase speed, the mass increases. More energy is needed to make it go faster because there is more mass. If that is what you are referring to, the energy itself is adding to the mass based on e=mc2, which can be rewritten as m=e/c2. The mass increase for a little energy is very small, but when you are talking about the amount of energy required to get something up to the speed of light, the mass increase becomes huge. -- kainaw 15:57, 23 February 2009 (UTC)[reply]
There really isn't much special to say - if you have gotten your head around time and length changes due to fast motion with respect to some observer - then just apply the exact same principles to the object's mass...the equations are the same, the cause is the same, the sheer cosmic wierdness of it all is exactly the same. When an object moves rapidly past some observer, that observer judges the mass of the object to have increased. The closer the object gets to the speed of light, the more its' mass increases...just as time and length change when that happens. If you have your head around length and time changing - mass should be easy to imagine! SteveBaker (talk) 16:34, 23 February 2009 (UTC)[reply]
As Tango said, physicists do not like the concept of a varying mass. That is because it does not add anything to our understanding. Length and time changing is enough to explain everything, including the speed limit of the speed of light. You do not actually need a varying mass to explain that! Essentially, "relativistic mass" is just short-hand for E/c2. The really important concept of mass is rest mass, which is E/c2 for a particle that is at rest. Still, for a lay person it is apparently easier to accept a varying mass, and it's probably easier to "understand" the speed limit as being due to increasing inertia than to the structure of Minkowski space-time, and therefore it is probably okay to use the concept in popular presentations. --Wrongfilter (talk) 17:55, 23 February 2009 (UTC)[reply]

wood - properties

is wood an isotropic solid. please qualify your answer.

Sathyanarayanan.d (talk) 15:50, 23 February 2009 (UTC)[reply]

"please qualify your answer" sounds like something from a homework question, we won't do your homework for you. Do you have a textbook that you can look up what "isotropic" means? I think once you understand the definition the answer should be fairly obvious (if you've ever done any woodwork, at least!) --Tango (talk) 15:53, 23 February 2009 (UTC)[reply]
Maybe lumber and wood grain will give you some insights. Isotropy means that the properties, like strength, hardness, and even color are the same in all directions. Nimur (talk) 16:02, 23 February 2009 (UTC)[reply]
And the opposite of 'isotropy' is 'anisotropy' - which (oddly) has it's own article. SteveBaker (talk) 16:27, 23 February 2009 (UTC)[reply]

Please do your own homework. Welcome to the Wikipedia Help desk. Your question appears to be a homework question. I apologize if this is a misevaluation, but it is our policy here to not do people's homework for them, but to merely aid them in doing it themselves. Letting someone else do your homework does not help you learn how to solve such problems.
Please attempt to solve the problem yourself first. You can search Wikipedia or search the Web.
If you need help with a specific part of your homework, the Reference desk can help you grasp the concept. Do not ask knowledge questions here, just those about using Wikipedia. —Scheinwerfermann T·C16:24, 23 February 2009 (UTC)[reply]

{{dyoh}} might be better for the refdesk. Zain Ebrahim (talk) 18:39, 23 February 2009 (UTC)[reply]

Spread of Disease from America to the Old World

Which Pre-Columbian diseases spread from America to the Old World, from the 15th Century and on? I know Syphilis is assumed to have spread from America to Europe ([11]), but other subspecies of Treponema pallidum already existed in Asia and Africa. Which pathogens were endemic to America before they spread to the rest of the world? Thanks, ליאור (talk) 17:39, 23 February 2009 (UTC)[reply]

The Spanish Flu is supposed to have originated in America.--217.84.62.226 (talk) 18:57, 23 February 2009 (UTC)PS But alas its a Past Columbian one.[reply]
Potato blight.Cuddlyable3 (talk) 19:29, 23 February 2009 (UTC)[reply]
Well, the H1N1 strain causing the Spanish Flu is just a subtype of a globally common virus, that happenned to mutate in Kansas. And potatoes were cultivated in the Andes, so one clearly needs potatoes to have potato blight. Is there a pathogen that was once only found in America and is now also found in Europe\Africa\Asia\Australia, affecting local hosts? ליאור (talk) 20:11, 23 February 2009 (UTC)[reply]
Tobacco smoking originated in the Americas and has probably killed more Europeans than all the other health risks originating there. Edison (talk) 20:15, 23 February 2009 (UTC)[reply]
Cocaine addiction? 76.97.245.5 (talk) 00:01, 24 February 2009 (UTC)[reply]
Nicotine and Cocaine are lethally addictive indeed, but these are alkaloids, not pathogens. Any more ideas? ליאור (talk) 05:59, 24 February 2009 (UTC)[reply]
There is also always the option of Argentine hemorrhagic fever or one of its relatives, though again I dont really know how old that one is. Coccidioidomycosis is also endemic in America.--91.6.42.166 (talk) 08:40, 24 February 2009 (UTC)[reply]
Chagas disease is also endemic to America, but neither disease spread to the Old World. Which ones did spread? ליאור (talk) 08:53, 24 February 2009 (UTC)[reply]
It seems that the Great French Wine Blight matches my request. Will it be safe to say that Borrelia burgdorferi, Trichomonas vaginalis, Francisella tularensis and Bartonella bacilliformis all originated from America? Was Xylella fastidiosa detected outside of America as yet? Thanks, ליאור (talk) 10:13, 24 February 2009 (UTC)[reply]

"... Irradiated the world's supply of steel"? Clarify?

I was reading the article on the Scuttling of the German fleet in Scapa Flow, and came across this line: "Minor salvage is still carried out to recover small pieces of steel that can be used in radiation sensitive devices, such as Geiger counters, as the ships sank before nuclear weapons and tests irradiated the world's supply of steel." Now, I can understand that atmospheric tests would leave residual radiation, even very small amounts, but wouldn't steel that has yet to be mined be similarly clean? Or is it that the steel in the ground has been irradiated, but the steel under Scapa Flow is somehow radiation free? Can someone elaborate on the meaning of the above quote? Thanks —Preceding unsigned comment added by 38.118.48.5 (talkcontribs)

That seems odd to me. Being irradiated shouldn't make the steel radioactive. It might have radioactive substances on the surface that need to be cleaned off, but that isn't too difficult. Even if it has been, all steel is going to be slightly radioactive since it contains carbon, some of which will be carbon-14, which is radioactive (only slightly, but still). That means a Geiger counter is always going to need to be calibrated to take into account the radiation of the counter itself, so you just have to calibrated it differently if it is made with irradiated steel. And even that seems pointless since background radiation from everything from uranium in the bedrock to people's smoke detectors probably results in a significantly higher order of error. --Tango (talk) 19:37, 23 February 2009 (UTC)[reply]

This question was raised and discussed last month on this reference desk: See World steel supply irradiated ? Abecedare (talk) 19:47, 23 February 2009 (UTC)[reply]

Iron ore mining does not take place in a "clean room," so any surface or subsurface dust naturally gets mixed in and is tin the resulting steel. Back in the late 1950's and early 1960's there was atmospheric nuke weapon testing by the various nuclear powers, and children were warned not to eat snow because of the Strontium 90. There was also concern about radioactivity in milk. There was no atomic bomb detonation before 1945, so the older steel has less radiation in it than post 1945 steel. When scientists want to use steel to shield nuclear testing instruments, there is apparently a meaningful difference in the radiation levels of the two vintages of steel. In i=other words, the modern level may be low so far as health risks are concerned, but they want the lowest possible level. Edison (talk) 20:13, 23 February 2009 (UTC)[reply]
Couldn't they just clean pre-1945 above water steel? The radioactivity will only be on the surface. --Tango (talk) 21:14, 23 February 2009 (UTC)[reply]
In the production of steel, oxygen produced from air is bubbled through molten iron. That removes impurities by oxidizing them. The oxides appear as slag on the surface of the molten metal, or are passed off as a gas. It is possible that strontium 90 left in the oxygen, and other nuclear-explosion products from the air, remain in the molten metal. One of the chief materials removed from the iron ore is carbon. But perhaps some of the carbon 14 from the air also remains in the molten metal. The OP will get a good homework grade for this. – GlowWorm.
Disagree - first the OP will have to separate out the slag, as it were. arimareiji (talk) 22:30, 23 February 2009 (UTC)[reply]
There's no need to do expensive diving to get old steel. Structural steel (I beans, channel iron) from old buildings and bridges will do as well. – GlowWorm.
One would think if that were the case then nobody would bother with the diving. Either the divers and their customers are idiots or it's harder than RefDeskers think. --98.217.14.211 (talk) 00:28, 24 February 2009 (UTC)[reply]
If anyone finds out, let me know (I'm personally curious). T'only ref I saw supporting this contention was an offline one. arimareiji (talk)
Huh. Found it through Google books, and it's legit - but heaven only knows how well the author sourced it. arimareiji (talk) 00:38, 24 February 2009 (UTC)[reply]

Is it not possible that steel present on the surface of the earth (structural) collects enough residual radiation over time to make it less useful, and that old steel wouldn't have been subject to such levels of residual radiation at the time, and is thus "cleaner"? Diving for steel is certainly more cost effective than mining, so there must be something less desirable about modern scrap (used) steel. Perhaps the problem with , say steel columns and rebar from 1944 skyscrapers its that the steel cannot be machined down to make the required shielding members for nuclear instruments, while 16 inch thich armor can be used as raw materials and cut to shape and used more economically. Still, one would think that pre 1945 steel columns and beams from demolished buildings could be ground free of surface contamination, then melted and used to cast the required shielding members. Edison (talk) 02:38, 24 February 2009 (UTC)[reply]

Perhaps induced radioactivity is involved. This might be produced in the iron in the steel, or in some other element in the steel. (There are various kinds of steel, produced by adding other elements to it.) The induced radioactivity may be produced by nuclear-explosion products in the air, and the radioactivity may not be able to penetrate depths of the ocean. (How deep is Scapa Flow? The German ships there are popular diving sites, which would indicate the depth.) But if the induced radioactivity is only at the surface of old structural steel, it could be ground off, sandblasted off, or milled off. – GlowWorm. —Preceding unsigned comment added by 98.21.104.187 (talk) 06:00, 24 February 2009 (UTC)[reply]

How can I depigment hair extensions??

I am looking for a way to depigment my hair extensions without harmful bleaching. Is there some agent I could soak the hair in for a period of time (even days) to slowly remove pigment from the hair extensions?? —Preceding unsigned comment added by 76.110.2.85 (talkcontribs) 19:34, 23 February 2009

I have no clue how to answer this - but I bet whoever does will need to know what the hair extensions are made of. SteveBaker (talk) 22:17, 23 February 2009 (UTC)[reply]
Not only that. See Artificial hair integrations. We'd also need to know how the extensions were attached. Your hairdresser is probably the better source for information. They should have the manufacturer's directions for your product. Your goal of lightening your extensions while protecting them may not be realistic though. AFAIK most/all decolorizing agents are oxidative. That damages hair by definition. The best you could probably hope for is to find a product with a conditioner that will coat some of the damage. Google coughed up this patent [12]. 76.97.245.5 (talk) 22:55, 23 February 2009 (UTC)[reply]


February 24

Measuring digits

Is there an easy way to measure the length of ones digits accurately? My left thumb is noticably (to me, not so much to other people) stumpier than my right, and I think it's because of the length of my first and second digit, as opposed to the position/length of the nail (which is also short, I believe as a result of the short digits), but I can't figure out a way to measure the lengths accurately to within 1mm or so, because I don't have any expensive measuring equipment. Any ideas? Thanks! 219.102.220.90 (talk) 00:40, 24 February 2009 (UTC)[reply]

A clear plastic tube, such as a coin tube, should do the trick. Insert your finger in as far as possible and mark the depth. Repeat and compare. -- Tcncv (talk) 01:21, 24 February 2009 (UTC)[reply]
The other option is tracing round your hands onto a piece of paper, then you can measure with a ruler. You would need to trace very carefully, though - keep the pencil vertical the whole time. --Tango (talk) 15:06, 24 February 2009 (UTC)[reply]
Extend your thumb, first and second finger at right angles to one another. Then use a ruler to measure each in turn as the side of a triangle. Cuddlyable3 (talk) 19:17, 24 February 2009 (UTC)[reply]
One biometric security device for confirming identity uses the lengths of the fingers. It was pretty good at distinguishing the hand of the person being checked from other random hands, and far faster than fingerprint or retinal scan. I believe some early ones used pegs which fit between the fingers and position sensors for the length, but now they seem to use electrical conduction. See [13]. A flatplate scanner can also be used [14]. Edison (talk) 00:02, 25 February 2009 (UTC)[reply]
I got a plastic vernier caliper at a dollar store (for one dollar) a few years ago. That is probably the simplest way to measure accurately. You can get fairly nice digital calipers from places like Harbor Freight for under 10 bucks, but be careful ordering from them, they will spam your mailbox with catalogs afterwards. I'm guessing you want accurate measurements because of this story.207.241.239.70 (talk) 06:18, 27 February 2009 (UTC)[reply]

The atmosphere on comet Lulin

This article on the comet states: Lulin has a greenish cast because sunlight illuminates two gases — cyanogen and diatomic carbon — in its Jupiter-sized atmosphere.

Am I supposed to believe that the comet has a big enough gravity field to hold gas at a range equivalent to the radius of Jupiter?? I can't find a source that states the estimated size of the comet, but I find it hard to believe it has enough mass to support such a size. 219.102.220.90 (talk) 01:39, 24 February 2009 (UTC)[reply]

Gravity of Comet Lulin may not be holding that gas in place, instead it is probably flowing outward from the comet, and dispersing in interplanetary space. What is there is travelling in the same orbit as the comet, with some being blown out into the cometary tail. Graeme Bartlett (talk) 03:47, 24 February 2009 (UTC)[reply]
Doesn't "atmosphere" imply that it is being held within the gravity field? 210.254.117.186 (talk) 13:50, 24 February 2009 (UTC)[reply]
Yes, they're using the wrong (or, at least, imprecise) word. It's called a coma. --Tango (talk) 15:03, 24 February 2009 (UTC)[reply]
Unfortunately, that news article is correctly quoting a NASA press release. In other NASA press releases, it is called a coma. In the one being quoted, it is called an atmosphere. -- kainaw 15:08, 24 February 2009 (UTC)[reply]
I guess we have to go with "imprecise" rather than "wrong", then. --Tango (talk) 19:14, 24 February 2009 (UTC)[reply]
Comets' comae can easily be larger than Jupiter, even larger than the Sun. In fact, Comet McNaught's coma was larger than the Sun, and Comet Holmes' coma was much, much larger than that. ~AH1(TCU) 22:10, 24 February 2009 (UTC)[reply]
Well I guess they can be arbitrarily large/long can't they, assuming gas is being pumped out constantly. Either way that settles it, thanks everyone! 219.102.220.90 (talk) 00:15, 25 February 2009 (UTC)[reply]
Yes, it really depends on how you define the edge. There is no clear end to the coma, it just gets thinner and thinner. Of course, similarly, there is no clear end to Jupiter or the Sun, so as long as you use the same definition you can get meaningful comparisons. --Tango (talk) 14:23, 25 February 2009 (UTC)[reply]

Requirements for being published in Science Mag: Policy Forum

What are the requirements for having a piece published in the "Policy Forum" section of Science (journal)? The WP article says that research articles are peer reviewed, but what about the Policy Forum section? What is the scope of this section? Thanks, NJGW (talk) 02:06, 24 February 2009 (UTC)[reply]

Have you tried contacting them directly? Their website is linked from our article, and if you click the "feedback" link on their homepage, they have a means to contact the editors and ask a question directly of them. --Jayron32.talk.contribs 02:10, 24 February 2009 (UTC)[reply]
The "Informational URL" listed in our article points to the "Information for Authors" webpage, from which you can find information about the submission process, selection criteria, etc. DMacks (talk) 02:13, 24 February 2009 (UTC)[reply]
Editorial articles (signed or unsigned) for science journals are generally not peer-reviewed. (They are, as the name suggests, provided at the pleasure of the editor.) --98.217.14.211 (talk) 02:14, 24 February 2009 (UTC) nevermind, the Policy Forum articles don't seem to be editorials. I suspect they are peer-reviewed in one way or another, based on the way they are presented (identical to the research articles). --98.217.14.211 (talk) 02:17, 24 February 2009 (UTC)[reply]
Thanks for the tips. All that the info for authors states is that the "broadly accessible commentary... presents issues related to the intersections between science and society that have policy implications." NJGW (talk) 02:46, 24 February 2009 (UTC)[reply]
The authors'-info page has a link to "Contributors FAQ", on which "What kinds of manuscripts does Science consider?" which notes: "A variety of material under the general heading of Commentary -- including Editorials, Book Reviews, Education Forums, Policy Forums, Perspectives, and Reviews -- is usually solicited by Science editors, although on occasion the editors will consider unsolicited manuscripts of these types." So it's not clear there whether it's peer-reviewed, but it's definitely not open-submission. DMacks (talk) 02:59, 24 February 2009 (UTC)[reply]

Amoebas and White Blood Cells

We all know that amoebas and white blood cells send out pseudopodia to engulf their prey or pathogens (respectively). Here's a picture of a mouse macrophage sending out pseudopodia to engulf two bacteria.

My question is: without a nervous system, eyes, or any kind of control system, how do amoebas and white blood cells have the coordination to know how to engulf something, much less reach out long "arms" like in that picture? To my understanding, cells are basically just little bubbles floating around, and it boggles my mind to think they can do such coordinated tasks. Inasilentway (talk) 02:43, 24 February 2009 (UTC)[reply]

Well, they're not such coordinated tasks; they are by and large simple trophisms. Specific stimuli for phagocytosis in the immune system are discussed here. - Nunh-huh 03:29, 24 February 2009 (UTC)[reply]
Did you mean tropisms? AndrewWTaylor (talk) 09:43, 24 February 2009 (UTC)[reply]
Oops, indeed I did. - Nunh-huh 04:04, 25 February 2009 (UTC)[reply]
Phagocytosis in three easy steps

(e/c) I hope you don't mind, but I reduced the size of your picture to assist those people viewing this page with restricted bandwidth. A basic answer to your question can be found at Pseudopod. Unicellular locomotion is a fascinating subject, and involves a co-ordinated process of protein interactions at the lamellipodium. It includes microfilamental structural elements and lots of other proteins (such as the Rho family of small Ras-related GTPases) that regulate the actin polymerization/depolymerization. This process essentially creates a cytoskeletal "treadmill" on which the cell can extend processes or move itself.

They engulf something on detecting it by a surface receptor. This creates in intracellular signaling cascade that results in phagocytosis. Our phagocyte article is also very informative about this process. Rockpocket 03:33, 24 February 2009 (UTC)[reply]

Cells are far more than "little bubbles floating around". And yes, they are unfathomably complex. An entire field of biology is devoted to understanding cell biology, including (but not limited to) cell motility, chemotaxis, intracellular communication, intracellular signaling, molecular control of gene expression, protein synthesis, endocytosis and exocytosis, cell division, programmed cell death, and many other topics. Cell biology is fundamental to the understanding of development in multicellular organisms, as well as much of what we know about disease. I highly recommend this series of online seminars, from the American Society for Cell Biology. --- Medical geneticist (talk) 15:09, 24 February 2009 (UTC)[reply]
As mentioned, macrophage has receptors that bind to bacterial surface. Bacterial surface has unique signatures, or common molecular patterns, that the macrophage are able to recognize. Also, when there's an infection, we produce antibodies (proteins that bind to the invaders). The antibody-bacteria complexes are like extra glue, making it easier for macrophages to attach and eat up the bacteria. The receptor make-up of the macrophage is very complex as to be able to recognize invaders. Check out http://www.ncbi.nlm.nih.gov/books/bv.fcgi?highlight=macrophage&rid=imm.figgrp.157 199.76.164.202 (talk) 03:39, 25 February 2009 (UTC)[reply]
If you think white blood cells or amobea are "simple" bubbles just floating around, have a read of Chromosome and DNA ;) Vespine (talk) 05:59, 26 February 2009 (UTC) 05:56, 26 February 2009 (UTC)[reply]

Photosynthesis

if u were to feed a plant sugar through the roots, would the plant need leavs(or photosynthesis)?

This, erm, documentary explains how the human equivalent to the above question works. --Jayron32.talk.contribs 06:20, 24 February 2009 (UTC)[reply]
Only if you would consider fungi to be plants and their mycelium to be roots. They are generally considered to be different. For "ordinary plants there's a problem because the sugar is transported in the Phloem from the leaves to the roots, the xylem transports salts and water from the roots to the leaves. AFAIK it doesn't work in reverse.--76.97.245.5 (talk) 08:01, 24 February 2009 (UTC)[reply]
You should look at the substances used to prolong the life of cut flowers, these are usually just vitamin B. Graeme Bartlett (talk) 22:37, 24 February 2009 (UTC)[reply]
This is exactly what parasitic plants such as striga do but they still need leaves etc so that they can set seed.Smartse (talk) 20:16, 28 February 2009 (UTC)[reply]

low radiation

would low levels of radiaton from a red laser pointer help of harm light loving plants?

Your Question is quite difficult to understand, but as far as i know, a laser Pointer does not produce what is commonly called radiation (its actually ionizing radiation). Also the wavelength of red light (around 700 nm) is not absorbed by either of the two chlorophylls so it probably wouldn't even give the plant energy... But what do you mean with "help of harm?"--91.6.42.166 (talk) 08:51, 24 February 2009 (UTC)[reply]
Probably meant "help or harm". Clarityfiend (talk) 08:53, 24 February 2009 (UTC)[reply]

91.6.42.166 is wrong about red light not being useful to plants energetically. The exact opposite is true. Photosystem I and Photosystem II (our plant articles are so deficient...), the big two light absorbing components of the electron transport chain necessary for photosynthesis, optimally operate in the red end of the spectrum at 700 and 680 nanometers respectively. In fact photosystems I and II are often called P700 and P680. There is a famous experiment by German plant physiologist T. W. Engelmann in 1881 which used filamentous algae and mobile aerobic bacteria. Engelmann used a prism to split white light up into the colors of the spectrum and laid some algae across the length of it. The oxygen-needing bacteria would congregate around where the algae was maximally performing photosythesis (and making the most oxygen) The results he found were that the bacteria congregated most strongly around the red end of the spectrum, and slightly less so around the blue. Of all the individual wavelengths, red light produced the most photosynthesis. See a pictoral representation of the results here.

As for the laser pointer question the answer is no. Most red light lasers operate in the 650 to 680 nm range rather than 700nm. I did some rough order of magnitude calculations and it appears that leaving a plant with 20 or so quarter sized leaves even in partial sun will deliver much more usable energy to the plant than with a handheld laser pointer, especially taking into account that the photosystems saturate above a certain photon density. An intense enough laser pointer could cause photobleaching and thermal damage to the photosynthetic machinery, but full sunlight can do this too. Plants have evolved robust ways of dealing with this sort of damage through repair mechanisms and by minimizing light exposure by turning the leaves parallel to the sun's rays, relocating chloroplasts so they are stacked on top of one another, and decreasing the number of photosystems and photosynthetic compounds. Plant physiology is pretty interesting. 152.16.253.109 (talk) 21:26, 25 February 2009 (UTC)[reply]

Latent index and manifest index (in psychology)

I would be very grateful if somebody could explain the terms latent index and manifest index for me in the following sentence that I found in an article:
"Factors represented by circles (e.g., early childhood adversity) constitute latent indexes of primary theoretical constructs (i.e, they are respectively composed of correlated but conceptually distinct elements). Genetic factors constitute a manifest index and are therefore represented by a box (see Bollen, 1989, for conventions relating to path model representation)."
By the way, Bollen 1989 is a book that I haven't read - and do not wish to read :) Lova Falk (talk) 09:06, 24 February 2009 (UTC)[reply]

Ripen my mango

Does anyone know how to ripen a mango? Or do I just have to leave it and wait? My inclination would be to put it on a sunny windowsill, but Moseley is cold and grey at this time of year.--Shantavira|feed me 09:50, 24 February 2009 (UTC)[reply]

Ethylene might work. They build countermeasures into fruit containers and warehouses so the fruit will stay fresh longer. Some fruit emit it naturally, so all you have to do is toss one of those into a plastic bag with your mango. 76.97.245.5 (talk) 11:08, 24 February 2009 (UTC)[reply]
If it's an ethylene-ripened fruit, then putting it near an old apple or banana, both of which produce ethylene as they become over-ripened, should work. --Jayron32.talk.contribs 12:12, 24 February 2009 (UTC)[reply]
According to [15] and [http://www3.interscience.wiley.com/journal/119468364/abstract mangos are indeed ethylene sensitive Nil Einne (talk) 12:26, 24 February 2009 (UTC)[reply]
In which case, putting it next to an old banana is your best option. --Tango (talk) 14:46, 24 February 2009 (UTC)[reply]
Thanks. Yes, I had forgotten the old banana-in-the-plastic-bag trick.--Shantavira|feed me 17:56, 24 February 2009 (UTC)[reply]
Make it a paper bag instead. Plastic bags keep moisture trapped against the fruit, encouraging rot. Matt Deres (talk) 21:44, 24 February 2009 (UTC)[reply]
When the mango is ripe, it should be deep yellow (no green and not too brown), soft, and sweet rather than sour. ~AH1(TCU) 22:06, 24 February 2009 (UTC)[reply]
You don't really need a bag at all, unless you are in a hurry. Just being in the same fruit bowl as a bunch of ripe bananas will do the job. --Tango (talk) 00:18, 25 February 2009 (UTC)[reply]

adrenal and gonadal androgen

Do I understand correctly from the article on androgens that there exist adrenal androgens, produced in the adrenal gland, and gonadal androgens, produced in the testes?
And, next question, do females produce just as much adrenal androgens as males? Lova Falk (talk) 10:06, 24 February 2009 (UTC)[reply]

You understand correctly. The adrenal glands produce three main types of hormones: glucocorticoids that have a role in the stress response and glucose homeostasis, mineralocorticoids that help regulate the salt/water balance, and androgens that are involved in the process of adrenarche (i.e. pre-puberty where pubic hair and body odor begin to develop). I don't know if there are(changing my wording... thanks, 159.14.241.253) specifically what the quantified differences in adrenal androgen output are between females and males, but during puberty the effects are dominated by gonadal production of testosterone in males (testes)and estrogen in females (ovaries). Abnormal adrenal steroid production is a key pathophysiologic mechanism in congenital adrenal hyperplasia. --- Medical geneticist (talk) 15:25, 24 February 2009 (UTC)[reply]
Of course there are age and sex differences in adrenal androgen production. All questions about normal human steroid hormone levels are only answerable for specified age and sex. Here, browse away to your heart's content. http://www.esoterix.com/files/expected_values.pdf —Preceding unsigned comment added by 159.14.241.253 (talk) 15:44, 24 February 2009 (UTC)[reply]

Scientist head of states!

Apart from Chaim Weizman and A.P.J.Abdul Kalam ,are there any instances of scientists heading a state?(Ramanathan) —Preceding unsigned comment added by 212.247.70.129 (talk) 12:25, 24 February 2009 (UTC)[reply]

An interesting non-example is that Einstein was asked to be Israel's second president after Weizman died, but he declined. --Sean 12:57, 24 February 2009 (UTC)[reply]
Weizman and Kalam are not scientists, but engineers. Very good engineers, no doubt, but not scientists. I found one, maybe, head-of-state scientist in Wikipedia, Stanislau Shushkevich, "first leader and head of state of independent Belarus after the dissolution of the Soviet Union". What the article means by "scientist" is impossible to tell, since it gives no examples of any discoveries of his, and the references are in Russian. --Milkbreath (talk) 13:14, 24 February 2009 (UTC)[reply]
Weizmann was a chemist, just FYI. He studied in chemistry, he did work in chemistry. Yes, there were industrial applications but that doesn't make him any less a chemist than Fritz Haber. From what I can tell, Shushkevich was a professor of nuclear physics (ядерной физики). --140.247.243.27 (talk) 21:09, 25 February 2009 (UTC)[reply]
Angela Merkel has a physics PhD, which is a lot closer to be a scientist than most, though she hasn't been an active researcher for decades. Dragons flight (talk) 13:16, 24 February 2009 (UTC)[reply]
Margaret Thatcher has a chemistry degree and worked as a research chemist before entering politics. Gandalf61 (talk) 14:43, 24 February 2009 (UTC)[reply]
Thomas Jefferson lacked any formal degree in sciences, but he was an avid researcher, and his work in fields such as botany and zoology and archeology could easily qualify him as a scientist. --Jayron32.talk.contribs 16:45, 24 February 2009 (UTC)[reply]
Adding to the list of female chemists, on a more unpleasant note, Elena Ceauşescu was a polymer chemist. She was only deputy Prime Minister, but was often rumored to have been the brains behind her husband. --Pykk (talk) 18:49, 24 February 2009 (UTC)[reply]
The article suggests she may not really have been a chemist and instead relied on other people to do the work for her Nil Einne (talk) 03:37, 25 February 2009 (UTC)[reply]
Ephraim Katzir, president of Israel, was a scientist. Also, Chaim Weizmann was actually a chemist / biochemist rather than an engineer, AFAIK; our article, too, says "chemist" rather than "engineer". --Dr Dima (talk) 00:25, 25 February 2009 (UTC)[reply]
Right, but as the "father of industrial fermentation", I figured Weizmann for a chemical engineer. "Chemist" doesn't necessarily mean "scientist", but the line is blurry, so he can come in for my money. --Milkbreath (talk) 00:34, 25 February 2009 (UTC)[reply]
If you're the father of figuring out how to turn some bench science into engineering science, it doesn't make you an engineer and not a scientist. It makes you a pretty impressive scientist! --140.247.243.27 (talk) 21:09, 25 February 2009 (UTC)[reply]

Woodrow Wilson was a political scientist. -- Mwalcoff (talk) 04:20, 26 February 2009 (UTC)[reply]

Abel Pacheco was a psychiatrist who did some scientific research (which may or may not count). Rockpocket 06:55, 26 February 2009 (UTC)[reply]

a pill i found

a friend of mine dropped a round white tablet in my car and now i'm curious what it is. it has a few numbers on it on two rows. 44 and 438 on the second row. —Preceding unsigned comment added by 98.210.135.91 (talk) 13:19, 24 February 2009 (UTC)[reply]

A Google search for "44 438 pill" finds several sources agreeing that it's 200mg of Ibuprofen, like this pill. jeffjon (talk) 13:52, 24 February 2009 (UTC)[reply]
Ibuprofen! Duh, what a disappointment. Richard Avery (talk) 14:42, 24 February 2009 (UTC)[reply]

Singapore history

Hi all, I am currently doing a history assignment. I am supposed to gather accounts of people living from the previous generations to give an account of Singapore during their time and suggest the history od modern singapore. Please give me your account of Singapore and how it was like. Thank you very much.(you could help me by helping me find accounts of other people). —Preceding unsigned comment added by Invisiblebug590 (talkcontribs) 13:34, 24 February 2009 (UTC)[reply]

That's not really the kind of references we can help with. I can point you to History of Singapore if you want the facts, but if you want personal accounts you will have to find them yourself, I'm afraid. --Tango (talk) 14:44, 24 February 2009 (UTC)[reply]
This newsreel gives an account of pre-WW2 colonial Singapore. Cuddlyable3 (talk) 19:02, 24 February 2009 (UTC)[reply]

Wave equation

I'm having trouble understanding waves. I find wave equation explainations insufficient. I can see how rates of speed in circular motion can translate to expressed different kinds/shapes of waves and appreciate the descriptive graph/animations on the wikipedia pages for waves and trigonometry. But I don't believe it sufficiently explains why concepts of trigonometry like sin,cos which a beginner like myself associates with triangles are brought into describing waves, there is a connection gap. Can anyone elaborate on how sin, sohcahtoa, e.g. opposites divided by hypotenuse (one never sees this process taking place and which parts they correspond to in the waves) relate/interact to aspects of the waves. Thanks in advance for any insight here. —Preceding unsigned comment added by Dbjohn (talkcontribs) 15:15, 24 February 2009 (UTC)[reply]

You associate trigonometry with geometric shapes like triangles, but that mathematics can be applied to a very large set of geometric and other conceptual problems. In many ways, you can think of a wave equation as "some parameter" (such as pressure), traversing around unit-circle. This means that it is repetitive, cyclic, and can be described as a "phase" (angle) parameter and a strength (magnitude) parameter. The conceptual leap from physical space to phase space is a big one - when "pressure" goes through an angle from 0 to 360°, the pressure isn't "moving" anywhere (although the wave may actually propagate as a result). The wave equation, for whichever physical situation you are considering, will describe the relationship between the phase of the parameter and its spatial coordinate (position) as a function of time. Let me know if you need some more clarification; this is conceptually a big leap, but after you can bridge the ideas, things will seem much simpler. Nimur (talk) 17:59, 24 February 2009 (UTC)[reply]
I suggest you look at these two figures[16][17]. In the second figure only the lengths marked sin and cos are of interest. There you have your right triangle where AO is the hypotenuse. Think of the hypotenuse continually rotating anticlockwise so angle theta keeps increasing. The sin and cos values vary over time (angle) as shown in the first figure. In practical terms, either could be the voltage wave produced by a rotating generator. Cuddlyable3 (talk) 18:39, 24 February 2009 (UTC)[reply]

Band-stop filters

I am considering using a digital filter to remove electrical background noise from facial surface EMG data. I have seen a number of different band-stop ranges in the literature to remove 60 Hz (I am in the USA), but there is no particular theoretical justification I can find regarding why one would use a particular range (for example, 59-61 Hz as opposed to 58-62 Hz). Is this an arbitrary decision? Does anyone here know what the standard deviation for 60 Hz mains hum is? Thanks, PotionsMasterSnape (talk) 15:37, 24 February 2009 (UTC)[reply]

Utility_frequency#Long-term_stability_and_clock_synchronization ~<0.1% deviation per day. —Preceding unsigned comment added by 213.249.232.187 (talk) 16:00, 24 February 2009 (UTC)[reply]
Two things come to mind: There might be times when you'll be getting power from a generator that won't have such a tight frequency spec. I looked online, and the specs I saw just said "60 Hz". The lack of an expressed tolerance bodes ill for their accuracy, I think. The other thing might be only in my fevered brain, but I imagine it's possible that line hum could feed some low-Q stray resonance in a circuit close to 60 Hz. --Milkbreath (talk) 16:13, 24 February 2009 (UTC)[reply]

Our article on Utility frequency notes that utilities try to keep quite close to the nominal frequency so that synchronous clocks only deviate a few seconds from actual time. That article says that utilityes may start automatic load shedding when the frequecy drops as much as, say .5 Hz from a nominal 60 Hz. to prevent a widespread blackout due to inadequate generation online to serve the connected load. So +/- .5 Hz should cover extreme frequency deviations, and a somewhat tighter band (+/- .1 Hz?) should cover normal day to day deviations, outside of system instability or overloads. If the power is provided by stand-alone generators, then the frequency deviation could be much wider, perhaps several Hz.Outlet power or even the power on the utility feeders also contains harmonic multiples of the mains frequency, the amount varying greatly depending on transformer balancing and the loads customers have connected. Edison (talk) 17:32, 24 February 2009 (UTC)[reply]

I've spent a lot of time filtering out power-line harmonics (and going to great lengths to avoid power-lines in the first place). Your particular measurements will have to dictate your removal method for power-line harmonics (have you looked at an FFT of your recording? You can probably see the 60Hz hum spike). In many cases, a narrow notch-filter works well; sometimes, I've used noise-fillers or statistical estimators to interpolate in the frequency or time domains to smooth out the final signal. The power-line hum can vary by a large amount from place to place; its mean frequency might not be exactly on 60 Hz; its amplitude fluctuates by extreme amounts (I have seen 100 dB range) based on the actual amount of current in the lines and how well your receiver couples in to it at any given instant). I have seen bandwidths of more than 20Hz on pathological datasets. It will largely be arbitrary; you can make an intelligent nonlinear filter or estimator to automate the estimation of the filter parameters, depending on your needs. (E.g. measure the original signal's frequency spectrum; estimate the f3db width of the spike closest to 60 Hz, and make a new notch-filter based on those). Nimur (talk) 18:06, 24 February 2009 (UTC)[reply]
You may also want to remove harmonics of the 60Hz such as 120 or 180Hz which are also likely to be present, especially if you have mains power supplies nearby. And if you want your system to work in other countries you may want to stop out 50Hz instead. If you make a very narrow bandstop filter it will have a high Q, and a long delay, so as to be more a problem for real time results. You should also look at the side effect on your signals, try to minimize the phase shift near the stop point. This can happen with the Gaussian filter but there will be a trade off with the shape of the stop band. Graeme Bartlett (talk) 22:30, 24 February 2009 (UTC)[reply]

Conservation of mass

The law of conservation of mass states that the sum total of mass in a system remains constant no matter what, right? Visualize the following situation: A couple (male and female) of animals (cows, dogs, goats, or even humans) is kept in a closed chamber for a year. The chamber is stocked with enough food to last them for quite a while. After a year, the food reserve is finished, and the female has borne a child. Does the law of conservation of mass apply here?? The food is finished, and there is addition of a third organism. How can the sum total of all the masses still be the same?? Thanks in advance. 117.194.230.71 (talk) 15:58, 24 February 2009 (UTC)[reply]

Enough food, water, and air to last a year would be a much larger amount of mass than most people would imagine. It would be many, many times the size of the third organism - the vast majority would be actually converted to waste solid, liquid, and gas. arimareiji (talk) 16:05, 24 February 2009 (UTC)[reply]
And don't forget heat, which hasn't really been discussed in the original scenario. Remember that conservation of mass applies to closed systems, meaning no heat in or out of the chamber, which will introduce all sorts of issues. ArakunemTalk 16:11, 24 February 2009 (UTC)[reply]
(EC) Yes. Everything would tally up, assuming you have a way of totalling ALL of the energy in this closed system (i.e. the masses of the people/goats/pink mongooses and all the food, plus the thermal energy "waste", because E=mc²), and I'm assuming you mean properly "closed", as in, say, a hermetically sealed chamber.--Rixxin (talk) 16:09, 24 February 2009 (UTC)[reply]
The heat released from the system is NOT converted to or from mass. It comes from chemical potential energy stored between atoms in chemical bonds and is NOT from mass-to-energy conversion which requires a very specific sort of thing, i.e. nuclear fission. If you have a truly closed system, the mass will not change at all, and there is no need to invoke E=mc2 to make it happen. The existing atoms in the closed system don't disappear, and no new atoms are created out of whole cloth. Since a closed system will have, atom-for-atom, the same exact atoms in it over any length of time, regardless of how those atoms are physically rearranged over time, the entire system will retain a constant mass. QED. --Jayron32.talk.contribs 16:36, 24 February 2009 (UTC)[reply]
No, he's right. E=mc2 applies to all forms of energy, including chemical bond energies. It's just that the amount of energy change associated with breaking chemical bonds is so tiny that the mass change is nearly always below the limit of detectability. If you burn and dissipate the heat from 600,000 gallons of gasoline the resulting CO2 + H2O would be about 1 gram lighter than the sum of the initial gasoline + oxygen. Dragons flight (talk) 16:52, 24 February 2009 (UTC)[reply]
I think the first two sentences of this section of the E=mc² article might shed some light on, if not settle, the matter.--Rixxin (talk) 17:12, 24 February 2009 (UTC)[reply]
I think the question is whether the light will settle into mass, not matter. --140.247.243.27 (talk) 21:14, 25 February 2009 (UTC)[reply]
Mass-energy equivalence is tossed around a lot these days. In this case, it is correct that energy-changes could result in minute mass changes, but those are truly minuscule and absolutely negligible for the purposes of this scenario. The original poster is probably just underestimating how much food it really takes - maybe some statistics on agriculture will help. A cow can consume 300 pounds of water in a single day. In a year, this will equate to tens of thousands of pounds of water. Does this help put into perspective? Nimur (talk) 18:15, 24 February 2009 (UTC)[reply]
Or even just consider the air: According to Oxygen: "An adult human in rest inhales 1.8 to 2.4 grams of oxygen per minute." We'll call that 2 grams. That's 2g*60*24=2880g per day. That works out to just over 1 tonne of oxygen in a year (which will have been converted into CO2). The newborn baby might have a mass of about 5kg, it's neglibible. --Tango (talk) 18:50, 24 February 2009 (UTC)[reply]
Well conservation of mass mostly applies to chemistry now (where E=mc2 can be safely neglected, as mentioned). And if you view this from a chemical viewpoint, it's not very strange. Humans catalyze the reaction: Food + O2 ---> Excrement + CO2 + Babies. --Pykk (talk) 19:00, 24 February 2009 (UTC)[reply]
Are there any other catalysts (other than other lifeforms) that catalyse a reaction that creates the catalyst? --Tango (talk) 19:07, 24 February 2009 (UTC)[reply]
Yep. See autocatalysis. There are a number of chemical reactions where one of the products is a catalyst of the reaction; these reactions will start slowly (if at all), and accelerate (as more product/catalyst is generated) until the supply of raw materials is depleted. TenOfAllTrades(talk) 20:03, 24 February 2009 (UTC)[reply]
There's a lot of math here to answer a simple question. Isn't the more obvious answer that the original questioner forgot about poop? APL (talk) 20:10, 24 February 2009 (UTC)[reply]
Yep - this stuff about E=mc2 is really a red herring. That's so negligable - we can ignore it for all practical purposes. So long as the container is absolutely air-tight (gas tight), water-tight and no solids, liquids or gasses come into or out of the container for the whole duration of the experiment - then the contents of the container will weigh exactly the same at the beginning as at the end. SteveBaker (talk) 20:54, 24 February 2009 (UTC)[reply]
By my extremely rough calculations, the energy losses would correspond to about 10kg over a year. That's pretty small compared to the tonnes of food, air and water than would be required, but it is probably measurable if you tried hard enough. However, if energy can be lost from the container, it may be possible for it to be gained as well (sunlight coming in the window, for example), so it's rather more difficult to work out what would happen to the mass. --Tango (talk) 21:10, 24 February 2009 (UTC)[reply]
Of course, I only realise I divided by c and not c^2 immeadiately after clicking "save"... my rough calculations actually come out at about 30μg. Ok, that's negligible! --Tango (talk) 21:13, 24 February 2009 (UTC)[reply]
Antoine Lavoisier was a pivotal figure in the discovery of the law of conservation of mass. Unfortunately, he was caught up in the french revolution and lost his head. Vespine (talk) 04:25, 26 February 2009 (UTC)[reply]

Ketonuria caused by a high protein diet.

Hi I read the article on ketonuria which states that the condition can be caused by a high protein low carbohydrate diet. My question is why? Say we had a non-diabetic individual who ate nothing but high protein foods such as meat and fish. Would not their bodies be converting the amino acids produced from the digestion of their food to glycogen by the process of gluconeogenesis? Why would their body then need to metabolise fats (it is my understanding that the catabolism of fatty acids produces ketones) for energy if glycogen can be produced from the protein they are eating? Would I be right in thinking that the person's body would not be able to get all the energy it would need from glycogen produced by the gluconeogenesis of amino acids and so fatty acids would need to be broken down to provide additional energy. Hope this makes sense and any suggestions would be great. —Preceding unsigned comment added by 139.222.240.66 (talk) 16:04, 24 February 2009 (UTC)[reply]

  • It's simplest to answer your second question first, actually.
  1. The body is very efficient at breaking down glucose into smaller particles and energy.
  2. The body is very inefficient at driving the same cycle in reverse to turn small particles into glucose. (gluconeogenesis)
  3. Functionally, the brain will not operate on any fuel other than glucose. If your brain stops, you die.
  4. Gluconeogenesis is a last-ditch emergency effort by the body to keep from dying.
  5. Anything becomes fair game as a matter of survival - fat and muscle included.
  • Back to the first question:
  1. Remember point #2 above - it's somewhat akin to trying to drive a water wheel backwards.
  2. You get a lot of the components for gluconeogenesis building up / pooling.
  3. Those components are high-energy, i.e. very reactive chemically.
  4. They're almost as happy to start engaging in side-reactions with themselves (condensation) as they are to be driven through gluconeogensis.
  5. The products of those side-reactions are ketones.
  • As you can imagine, this type of diet is very effective - but it's also a very bad idea if you prefer long-term survival. It plays hell with your liver and glucose regulation, among several other niceties. arimareiji (talk) 17:14, 24 February 2009 (UTC)[reply]

The article on ketogenic diet and particularly the mechanism of action section may be helpful. If your body is in starvation mode (or thinks that it is in starvation mode) then any glucose made via gluconeogenesis will go straight to the blood stream and not be converted back into glycogen, which is the storage form. The hormones that control this process (insulin and glucagon) have opposing roles and once you're in a catabolic state the breakdown of fatty acids will be an integral part of the process. By the way, the above statement that "the brain will not operate on any fuel other than glucose" is incorrect. The brain CAN and DOES utilize energy sources other than glucose, namely the ketone bodies that are produced during starvation. --- Medical geneticist (talk) 17:52, 24 February 2009 (UTC)[reply]

Please note that I already qualified the statement with "functionally" for the exact reason that you specified. arimareiji (talk) 18:32, 24 February 2009 (UTC)[reply]

Thank you very much for your help to both of you. —Preceding unsigned comment added by 139.222.241.116 (talk) 09:56, 25 February 2009 (UTC)[reply]

The answer can be made a lot simpler. Insulin levels are primarily determined by carbohydrate intake (carbohydrates stimulate insulin secretion). The processes of ketogenesis are amplified whenever insulin levels fall below a threshold level, and suppressed when insulin levels rise. Although a few other factors besides carb intake affect insulin secretion, and a few other factors besides insulin levels affect ketogenesis, the basic answer to the question is this simple. 159.14.241.230 (talk) 19:52, 25 February 2009 (UTC)[reply]

You could make it simpler yet and say "Because it just does." arimareiji (talk) 23:14, 25 February 2009 (UTC)[reply]

Here's an afterthought - does anyone know if the concentration of ketones in the urine would significantly differ between a non-diabetic person on a high protein diet and person who has undiagnosed type 1 diabetes? I expect the diabetic would have a greater concentration of ketones in the urine. Could a person on a high protein diet give themselves ketoacidosis by eating protein rich foods exclusively? I believe a people persue a high protein diets to try and put on muscle mass, but surely then, persuing a high protein diet to such extremes that only protein rich foods were eaten would be counter-productive to trying to put on muscle mass as it would just result in ketogenesis which depletes fat reserves and causes muscle wasting. Once again thanks to all that have helped. Any thoughts? —Preceding unsigned comment added by 139.222.241.116 (talk) 11:13, 26 February 2009 (UTC)[reply]

Blood and urine ketone levels can reach much higher levels (like >10x) in diabetic ketoacidosis compared to ordinary low-carb ketosis, because even the low levels of insulin in starving healthy people are enough to prevent the extreme degree of ketosis that leads to DKA. Ketogenic diets are used in the treatment of several conditions, especially difficult-to-control epilepsy. A high fat diet is more effective than a high protein diet for maintaining ketosis. PS, I just looked at the ketogenic diet article-- it is superb: kudoes to user:colin and others. 159.14.241.230 (talk) 13:52, 26 February 2009 (UTC)[reply]

Tiny Springs

Hello. I think I may wish to acquire tiny springs - 1mm diameter, length >3mm.

Does anyone know where I could get such things (UK), the springs preferably should be slightly compressable, and on the weaker side of spring constant (for their size).

Alternatively (my main question) - how could I turn tiny coils of wire into tiny springs (in real life practice) - what substance should I used, and what are the ways of making wire springy (copper wire as bought makes a coil - but isn't springy)

Thanks. FengRail (talk) 21:45, 24 February 2009 (UTC)[reply]

Google, in their "sponsored links" section, has several spring manufacturers and sellers, as seen here: in this google search. If you refine your search, you may find one to meet your need. --Jayron32.talk.contribs 16:16, 24 February 2009 (UTC)[reply]
Could you cut a bit of a spring from a "clicky" biro?--82.70.123.254 (talk) 16:25, 24 February 2009 (UTC)[reply]
Remember, 1 mm isn't much thinner than your fingernails in cross-section. arimareiji (talk) 16:29, 24 February 2009 (UTC)[reply]
Most devices that small utilise tiny pads of foam rubber instead.--Shantavira|feed me 18:01, 24 February 2009 (UTC)[reply]
McMaster-Carr, my preferred mechanical stock supplier, has springs down to 0.12 inch (3mm) outer diameter. Nimur (talk) 18:18, 24 February 2009 (UTC)[reply]
You'll find a spring of about the size you are looking for inside a leather watch strap pin. You know, that metal tube with a sprung tip at each end that fastens the strap to the watch and to the buckle. --Heron (talk) 19:09, 24 February 2009 (UTC)[reply]
Yes that's the sort of thing - thank you. I might need a few but I guess that if I buy 100 they won't cost much.
  • By the way I'm still interested in making my own...(I think the watch spring pins might be just over 1mm, .... mmmh - are ladies watch pins smaller?) If anyone can give material hints....FengRail (talk) 21:45, 24 February 2009 (UTC)[reply]
Hop over to your local dollar store and grab a selection of their $ 1 each watches. Some, but not all have a spring like what you are looking for under the battery. If you are lucky you might even find one model that has springs at the "push buttons". You could also contact your local watch repair shop and ask them if they have any. Unless you can talk them out of a couple of irreparable broken movements, the brand name stuff is likely going to cost you an arm and a leg. I tried to find a source online, but there are only a handful of manufacturers that actually make watch movements. So the springs aren't a mainstream item. Your local goldsmith will be able to point you towards a source for fine wire to make your own springs.76.97.245.5 (talk) 01:00, 25 February 2009 (UTC)[reply]
The springs under the pushbuttons will be leaf springs. I think FengRail wants a coil spring. The spring pins that hold the watchband on as Heron suggested are the most obvious source. If you want to wind your own from spring steel I expect you'd have to heat treat them. 207.241.239.70 (talk) 06:40, 27 February 2009 (UTC)[reply]

Botany of antractica

Are there any plants naturally occuring in Antactica? Simply south is this a buffet? 18:23, 24 February 2009 (UTC)[reply]

Yes - Flora of Antarctica should give you more detail. DuncanHill (talk) 18:27, 24 February 2009 (UTC)[reply]
Thanks. Simply south is this a buffet? 00:59, 27 February 2009 (UTC)[reply]

Sexuality being decided?

I just had an argument with a friend today who said she believed that our sexuality (ie: gay/straight/transexual/whatever) was decided for us, chemically, before we are born. I argued that there was no evidence for this and that it was much more likely that one of the trillion influences on our development have an effect at some point which eventually determin how we feel about ourselves sexually. I wondered if there is any scientific evidence either way? Is there really (as she claims) a bunch of chromosomes that are instrumental in deciding our sexual fate before we are born? Has there ever been any social studies to explore the possibillity that a certain type of upbringing is more likely to produce a homosexual? I put this on the science desk to limit the level of 'opinion' that a question like this might generate, so please try and keep answers as factual as possible, thanks.91.111.86.8 (talk) 20:04, 24 February 2009 (UTC)[reply]

Here's a relevant article: Biology_and_sexual_orientation. Friday (talk) 20:09, 24 February 2009 (UTC)[reply]
Another suitable article (well, it is more of a stub) is Xq28 and the references. --Cookatoo.ergo.ZooM (talk) 20:12, 24 February 2009 (UTC)[reply]
In the purest sense, a gene for outright homosexuality would be selected out of the gene pool in a single generation. So we know it can't be as simple as that. Such complex behaviors are highly likely to be a combination of nature and nurture - genes and upbringing. Bisexuality could perhaps be a gene because it wouldn't out and out prevent itself from being passed on to the next generation (although you'd still imagine it would be strongly selected against over a number of generations unless it conferred some other benefits that ultimately (and paradoxically) aided reproduction). SteveBaker (talk) 20:47, 24 February 2009 (UTC)[reply]
A dominant gene for homosexuality would be selected out in a single generation (assuming culture allowed it - plenty of homosexuals have married and had children since that was what was expected of them by society), a recessive gene could last longer, particularly if being a carrier held some advantage (like with sickle cell anaemia). One (very small, perhaps) advantage of being a carrier of the homosexual gene would be that you are likely to have homosexual siblings that wouldn't have children of their own so could help you support yours. (That's more of a mitigation of the disadvantage than an outright advantage, I suppose.) However, I agree, there is clearly something more going on than a single gene. --Tango (talk) 20:58, 24 February 2009 (UTC)[reply]
(See also, kin selection. Note though that it is certainly not the case that 100% of all homosexuals do not reproduce genetically. In fact one can probably make the argument that the history of homosexual oppression in the last few hundred years would have led to more homosexuals reproducing than the current more liberal sentiment, as I suspect those in the closet have kids more often than those out of it do.) --98.217.14.211 (talk) 05:21, 25 February 2009 (UTC)[reply]
It's possible that homosexual individuals who do not have children provide some benefit to their own close family members. If so, then while not themselves reproducing, those with a homosexuality trait may indirectly encourage the propagation of their own genes (via their siblings). I seem to recall reading about such a hypothesis once in a sociobiology class I took many, many moons ago. I'll see if I can find a source when I get home. – ClockworkSoul 22:27, 24 February 2009 (UTC)[reply]
The problem with the question in many contexts is that, if one were to say "This is 100% choice" then it many people take this to mean "this is a valid reason to discriminate against someone who is homosexual", and yet we have laws in most western nations that disallow discrimination based BOTH on genetic-based traits (such as gender or skin tone), choice-based traits (such as religion), and environmentally-determined traits (such as historical national origin). The reality is likely that, like every single behavioral trait we experience, there are some combinations of genetics, environment, and choice involved in it, and every individual has a unique melange of these factors determining their own unique personalaties. To isolate one factor of a personality and say "This bit here, this bit must be all genetic" or "This bit here, this bit must be all choice" is wrong for several reasons. First, its never all genetic or all choice or all environment, and secondly, you cannot isolate a single aspect of someone's life in that way and treat its cause in isolation from the rest of their personality. Its just not that simple. --Jayron32.talk.contribs 04:59, 25 February 2009 (UTC)[reply]
There is no doubt that there is much to be said in terms of political correctness for there being a genetic component - but science is what science is - and sometimes the truth just isn't politically correct. It's perfectly possible for there to be no genetic component whatever - although it's extremely hard to imagine that a purely genetic reason is a possibility. I agree that the most likely explanation is a mixture of genetics and upbringing - but I don't think we know that yet. SteveBaker (talk) 16:55, 25 February 2009 (UTC)[reply]
Clockwork, there may be some benefit to the family of a homosexual but notice that this sort of explanation isn't used for anything else. You don't speak of the (any type of trait that makes him highly unlikely to reproduce, like being infertile or disgustingly unattractive, etc) Uncle being selected for by natural selection because he provides benefits for his kin.--droptone (talk) 13:11, 25 February 2009 (UTC)[reply]
The simplest answer to that is kin selection. Most of what I would say is included or linked in that article. If you treat humans as social creatures that evolved in tribes, rather than individually, a lot of things make more sense; competing tribes try to out-survive each other. 79.66.56.21 (talk) 16:18, 25 February 2009 (UTC)[reply]
Kin selection is a rather weak form of selection. It can explain things like that we tend to live past child-rearing age...but for something as specifically significant to reproduction as homosexuality - I think that's a long stretch! Even tribes need people to breed - and specifically in a small tribe where there is likely to be much inbreeding - a latent gene such as this would have to be would doom the tribe pretty solidly. If there is a genetic component to this - it really can't be the whole answer. SteveBaker (talk) 16:55, 25 February 2009 (UTC)[reply]
I would doubt it is the whole picture (seriously doubt it), and there are many proposed ways in which selection may work on genes giving an increased tendency towards homosexuality (aiding with rearing nephews and nieces, increased fertility in women who carry the genes that tend to be associated with homosexuality in men, etc). Tribes do indeed need people to breed, but they don't necessarily need all members to breed. Particularly not all men. But anyway, that is beside the point; I was replying to the claim that this reasoning is only ever applied to homosexuality. Kin selection is a wider thing applied in more areas than merely sexual orientation, and that is what my reply was supposed to indicate. 79.66.56.21 (talk) 22:05, 25 February 2009 (UTC)[reply]
E.O. Wilson is the one who suggested homosexuality and kin selection are likely related; if I recall, he points out (in On Human Nature) that tribes that had, say, 1/10th of the men who were on whole more dedicated to the support of others in the tribe (and not breeding), and are willing to help with "women's work" would be quite an advantage over time. But it's a 1970s sort of argument on many fronts. Part of the difficult here is that we are used to, in the west, defining homosexuality/heterosexuality as some sort of simple binary. While I don't think sexuality is as evenly distributed on the bell curve as, say, Kinsey argued, the idea that there should be some sort of behavioral spectrum seems fairly likely (given that almost all other behaviors exist on such a spectrum). What we call "homosexuality" could really just be the extreme case of a more subtle effect, one that really only manifests when you start talking about really large sample sizes. Obviously cultural/environmental values must play into it in some way as well, of course. --140.247.11.54 (talk) 01:59, 26 February 2009 (UTC)[reply]
"Specifically significant to reproduction as homosexuality" - pardon, but kin selection allows selecting for infertile-by-birth members; homosexuality - which, if viewed entirely from a breeding standpoint offers similar advantages without the lock-in cost - in extremis, a tribe with one homosexual and one alternate sex can procreate, a tribe with one infertile-by-birth eusocietic member and one alternate sex is doomed. 98.169.163.20 (talk) 07:23, 27 February 2009 (UTC)[reply]
It's worth remembering that 'determined before birth' does not mean 'determined entirely genetically'. There are many traits where conditions in the womb and other things have some effect but which are determined largely or completely before birth Nil Einne (talk) 02:43, 26 February 2009 (UTC)[reply]
Exactly. I'd say the actual decision comes from the womb, hence the reason why there's about a 50% concordance rate among identical twins, but only a 25% concordance rate among fraternal twins. They're exposed to the same womb, but because their brains are developing at significantly different rates (as opposed to only slightly different rates), the effect of a hormone surge at one period is less likely to affect both of them at the same time. Thus, the "gay gene" is, I would argue, a "womb instability gene," and not causal to being gay or not-- it would merely influence how unstable the womb is. Combined with womb memory research finding that the more kids a female has, the greater the likelihood that subsequent children will turn out gay, I'd say there's ample evidence to suggest the womb is at fault-- not simple genetics.
It makes sense, too, as a population control mechanism, and it's presumably why homosexuality is rampant in higher order animals (including aves and mammalia). The easier it is for them to survive in their environment, the more likely it is for them to overpopulate, exhaust their resources, and die off. In this instance, survival of the fittest creates a situation where the fittest would die and therefore not be the fittest. Thus the "womb instability gene" probably came along as a way to naturally mediate overpopulation without the danger of creating underpopulation, thereby continuing the trend of "fittest" without worry of the fittest multiplying like rabbits with no fox to eat them.
Thus, the "gay gene" (or really, "womb instability gene") is simply another evolutionary advancement: it prevents the species from sexually selecting for totally gay/asexual in times of plenty, thus dying out; it allows for a gay member to reproduce during a population crisis without guaranteeing homosexuality to its offspring (thus preventing dying out); and, finally, at the same it prevents the species from staying totally heterosexual, which, when it comes to high-order animals like humans, would also result in the species dying out. As a spillover benefit, gays still retain mothering and fathering instincts and are able to care for abandoned children or children whose parents have died, thereby also increasing the chances of successful continuance of the species when, under certain circumstances, the abandoned child's lineage would die out when the child did.
Taken in this light, the persecution of gays is actually what's more logically unnatural—not gay people themselves—while the uninhibited population growth by forcing gays into lives of procreation actually worsens the problem and makes it more likely that humans will die off due to overpopulation.
--slakrtalk / 05:43, 26 February 2009 (UTC)[reply]

DC power supplies in parallel/series

I am building a guitar pedal at the moment and am having trouble with the power supply. I originally designed the pedal to run on 9V batteries (two of), but as I use a powered pedal board, it seemed a better idea to open it and replace the batteries with DC sockets(two of). I have the correct sockets, and have wired them up in sort of "Y" configuration - where the two tips of the Y connnect to a 9V DC socket (either +ve or -ve), and the bottom tip of the Y is the +ve or -ve supply respectively. Upon use, I have found that it does not work as it used to. It was a distortion pedal, based on the fact that op-amps saturate after a certain level of input - so I think the problem lies in the configuration of the DC supply. Right now, they are connected in some kind of pseudo-parallel configuration, and I am wondering if I connect them in series, will it solve my problem. My thinking is that when batteries are connected in parallel (+ve to +ve and -ve to -ve) the overall voltage is the same, but the current is doubled, whereas when they are connected in series (+ve of one to -ve of the other) the voltage doubles, but the current stays the same. In the battery, however, the +ve and -ve poles are connected via the chemicals in the battery - but with my DC socket, the -ve and +ve poles are isolated from each other, and to connect them (in a series fashion) would ground the power, and thus not be able to power any of my pedals. Is there any way around this?

Summary: How do I connect two DC sockets together (bear in mind, the +ve and -ve terminals are isolated from each other) so that the voltage across the two of them is equal to the sum of the two input DC voltages?

Any help is appreciated. Thanks! —Preceding unsigned comment added by 81.158.231.154 (talk) 20:29, 24 February 2009 (UTC)[reply]

You need to know whether the pedal has the two 9 volt batteries driving different parts of the circuit at 9v each - or whether they are in series (to generate an 18 volt supply) or in parallel (to last twice as long or to produce twice as much current as a single battery). I strongly suspect the latter because there aren't many (if any) simple audio circuits that need 18 volts! However, if the two batteries WERE in series - then you just shorted out your power supply because the pedal connected the + of one plug to the - of the other...but they go back to the same power supply via the 'Y' lead! This might have damaged the power supply...or maybe blew a fuze inside - or maybe just caused a thermal trip to trip out...depending on the kind of power supply. If the batteries were in parallel - then there was no point in wiring up two separate jack plugs because they are both going to the same place. In that case the problem might be that your power supply can't produce as much current as two 9v batteries.
At any rate - you cannot double the voltage of a power supply by putting in a 'Y' lead and then connecting the + of one plug to the - of the other. That not only doesn't work - but is highly likely to do damage to the power supply. If you truly need 18 volts (which seems VERY unlikely) then you either have to go back to using 2 batteries - or you need a proper 18 volt power supply.
SteveBaker (talk) 20:41, 24 February 2009 (UTC)[reply]
Many op amps require dual (split) supplies; +V and -V, and I would bet that your two 9V batteries were connected in series so one provides the positive and one the negative voltage, with the remaining two battery terminals tied together and connected to the circuit ground. Yeah, powering it with a single supply in a Y won't fly. To power the pedal you will need two 9V DC supplies, such as a pair of "wall wart" transformers. You can put sockets on your pedal that accept the wall warts' plugs. Just be sure they connect to your circuit with the same polarity as your batteries. As an alternative, at thrift shops you can often find wall warts that provide split supplies. Look for one with terminals marked, +9V and -9V, and GND or COM, then connect the COM line to the two (opposite polarity) battery terminals which were tied to the circuit ground. You can tell which ones these are because they are connected together, or you can use an ohmmeter. Then connect the +9V and -9V lines to the remaining + and - battery connectors. If you can't find ±9V transformers, your op amps will probably work ok with any voltage between ±7V and ±15V, as long as both positive and negative voltages are the same. --ChetvornoTALK 22:40, 24 February 2009 (UTC)[reply]

Yes, I am powering an op-amp, and yes, I am using 18V. The op-amp I am using (NE55532) can take a 3V-25V power supply, and so I decided 18V was the most I could feasibly use. Thanks for the tip that I won't be able to generate as much current with two power supplies as with two batteries. I'm just going to revert back to batteries for now. If you can find any kind of information on turning two 9V DC supplies into one 18V DC supply then let me know. Thanks! —Preceding unsigned comment added by 81.158.231.154 (talk) 10:41, 25 February 2009 (UTC)[reply]

But why do you need 18v just to generate a few volts of audio signal? You're putting the signal into an amplifier at the end - and it CERTAINLY doesn't want 18 volts of audio! SteveBaker (talk) 16:43, 25 February 2009 (UTC)[reply]
The reason for the somewhat crazy voltage requirements are that many op amps (and certainly the NE5532) cannot swing their output close to the supply rails. In the worst case they can only get within 3 V, so a power supply of ±9 V only gives an output dynamic range of ±6 V. There are other reasons to use large supplies for an op amp (eg. better slew rate, better bandwidth) even if the output dynamic range doesn't need to be large. To the OP, if you have a 555 timer and some largish capacitors, you can make a "voltage inverter" which can obtain about -7 or -8 V from a +9V supply (google "voltage inverter" for some circuits). However, these types of circuits are typically charge pump circuits which may have unacceptable ripple voltage. Another way to obtain a dual supply when you only have a single supply is to get a 24 V supply and split it with a 7812, then call the regulated +12 V "ground", the old ground "-12 V" and the old +24 V "+12 V". TI also makes supply splitters (TLE2425 and TLE2426) which are designed for this purpose, so you can try using those if you can find them. 130.194.72.111 (talk) 00:21, 27 February 2009 (UTC)[reply]

hydrodynamic and aerodynamic

On the front of large ocean going vessels (and probably other kinds of ships too) there is a bulb shaped thingy (can't remember the name) that aids in the hydrodynamics of the ship. Thus, with this bulb thingy, the ship moves through the water easier (due to a decrease in water resistance). Does this same principle apply to cars, planes and other vehicles as they pass through the air? —Preceding unsigned comment added by 69.77.185.91 (talk) 21:07, 24 February 2009 (UTC)[reply]

Oh. I found the name: bulbous bow! D-uh. —Preceding unsigned comment added by 69.77.185.91 (talk) 21:10, 24 February 2009 (UTC)[reply]

A bulbous bow works on ships because they're travelling on the interface between air and water. Nothing else travels on such an interface, so you won't see bulbous bows on submarines or airplanes. --Carnildo (talk) 22:58, 24 February 2009 (UTC)[reply]
We really need an article about that. Clarityfiend (talk) 23:52, 24 February 2009 (UTC)[reply]

Water Hammer Arrestor

When I bought my house, the washing machine was causing bad water hammer, so I installed a water hammer arrestor which solved the problem. Years later, the problem has returned. I think I remember reading something at the time that said that eventually the arrestor fills up with water (or something like that) and something must be done to replenish it.

My question is: What do I need to do to 'replenish' it? Just remove it and shake out the water? Or do I actually need to replace it with a new one?

BTW: We need an article on water hammer arrestors

ike9898 (talk) 22:13, 24 February 2009 (UTC)[reply]

Drain out the water. An arrester works by creating an air-filled gap in the water line. Since air is compressible while water isn't, it absorbs the shock rather than transmitting it. --Carnildo (talk) 23:00, 24 February 2009 (UTC)[reply]
We've got this article Hydropneumatic device. DuncanHill (talk) 23:01, 24 February 2009 (UTC)[reply]
"Drain out the water" is the correct answer. However, it doesn't explain how to do that. What I did was turn off the water at the main line - so no water would flow into the house. Then, I opened all the taps. This pretty much drained all the water out of the lines. Then, I closed all the taps, turned the water back on, and the arrestor started working again. Obviously, this is limited to an arrestor that is higher than the lowest tap - since you can't drain anything below the lowest tap. -- kainaw 05:04, 25 February 2009 (UTC)[reply]
I have linked the Hydropneumatic device from water hammer arrestor. Graeme Bartlett (talk) 05:18, 25 February 2009 (UTC)[reply]

February 25

Pipe noise

If I turn on the hot water in my shower with limited flow (not a trickle, but not a gush), within a minute the flow slows to a trickle and it starts making a rapidly-repeating banging noise that doesn't stop until it's turned off or the flow is increased. Cavitation, water hammer, or something else? arimareiji (talk) 00:13, 25 February 2009 (UTC)[reply]

Yes, mine too! A whistle, indeed. What is it? I guess that that particular speed of the flow corresponds to a proper frequency of the tap and make it resonate. --pma (talk) 00:32, 25 February 2009 (UTC)[reply]
Usually, but not always, it's just something stuck in your aerator (faucet) or showerhead. Unscrew that and descale it with some vinegar. See if the noise persists. 76.97.245.5 (talk) 01:03, 25 February 2009 (UTC)[reply]
That jogged my memory. What you describe happens when the screw holding the washer works loose. I've actually seen that, the prohibition against original research be damned. --Milkbreath (talk) 05:09, 25 February 2009 (UTC)[reply]
This little phenomenon occurs to me as well. The banging noise is called Water hammer, and if it starts, you should stop it quickly by turning it off/increasing the flow, whatnot. The signature bang characteristic of water hammer is caused by rapidly increasing pressure in your pipes. If it continues, your pipes could burst from the pressure, so I would remedy this with whatever remedies happen to appear here. Until It Sleeps 05:24, 25 February 2009 (UTC)[reply]
More OR, recently the plumber changed the washer and ended 18 months of water hammer. Ya-aayy! Julia Rossi (talk) 12:53, 26 February 2009 (UTC)[reply]

why is AgNO3 kept in coloured bottles??? —Preceding unsigned comment added by 123.50.168.74 (talk) 13:15, 25 February 2009 (UTC)[reply]

Although it is, as our article says, "relatively stable to light," keeping light out of the containers it's stored in minimizes the breakdown of the compound. Deor (talk) 14:12, 25 February 2009 (UTC)[reply]
Because it is sensitive to light. If it is exposed to too much light it turns into silver (I think). As our article says, it isn't as sensitive as silver halide, but it is still sensitive. --Tango (talk) 14:15, 25 February 2009 (UTC)[reply]
You're right about elemental silver being a decomposition byproduct. I think the photodecomposition reaction is the same as the thermal decomposition reaction which is 2AgNO3 ---> 2Ag + O2 + 2NO2 152.16.253.109 (talk) 18:49, 25 February 2009 (UTC)[reply]

Silver nitrate used to be used for photographic work - so it is indeed sensitive to light. But the main reason it was replaced is because it's not sensitive enough for dim light or short exposures. So it's RELATIVELY insensitive (compared to other photosensitive substances) - but a dark glass jar is good enough. SteveBaker (talk) 16:39, 25 February 2009 (UTC)[reply]

Actually, it's because it happily forms silver oxide if the jar isn't completely airtight and humidity gets inside, or if it gets opened a lot. If you work in a chemistry lab with a solution of it and it touches your bare skin, you'll soon know because it turns an ugly brown/black as soon as it's exposed to sunlight. Fortunately, it wears out of your skin in a week or so.
Which is a lot better than what happens to people who try to medicate themselves with silver ions - they can turn permanently blue (argyria). No, I'm not kidding. arimareiji (talk) 22:50, 25 February 2009 (UTC)[reply]
Clear glass would stop air or moisture getting in. Coloured class is used because of light. --Tango (talk) 00:01, 26 February 2009 (UTC)[reply]

high electric tension

Hello I heard that when there is very high electric tension (eg 1000000 Walt), there is no longer a risk to humans. 1 Is it true? 2 What is the explanation for this phenomenon?

In the article Electric shock it is written that the danger is primarily in the tensions between 30 and 250 volt, it may be related to the subject.

Thank you very much —Preceding unsigned comment added by 82.81.19.138 (talk) 13:26, 25 February 2009 (UTC)[reply]

I think the reason high tension cables aren't dangerous is because they are kept up high out of the way so you can't form a complete circuit to ground. High voltage means lower current, which might make a difference, but as electric shock from a high tension cable will still kill you. --Tango (talk) 14:19, 25 February 2009 (UTC)[reply]
There may be some confusion, as electric tension is a measure of voltage, not wattage (electric power). A high-tension power line will be VERY dangerous - "lower current" does not mean "safe levels". A Van de Graaff generator, on the other hand, is high voltage and very low current, and is generally not harmful (but this is not a guarantee - you can still get a painful shock, depending on what you do with it!) Finally, high frequency electricity will be subject to skin effect and will probably not penetrate deeply into the human, making it "safer" than lower frequency AC current. Again, none of these are guarantees of safety - there are situations where skin-effect is not sufficient to protect you. In the case of very high power (watts), there are other safety issues, including thermal heating, which may be equally dangerous as electrocution. Nimur (talk) 16:06, 25 February 2009 (UTC)[reply]

I think there is a fundamental confusion here. The risk due to touching - or even getting within a few feet of a high tension power cable is most definitely very high indeed. What I think the OP is referring to is that it is (perhaps) no longer considered dangerous to live, work or generally be close to high tension power lines. There has been much controversy over this in the past where people who live or work close to these gigantic pylons have complained of a variety of long-term medical conditions that have been loosely attributed to some aspect of those power lines. There has been more talk of this in the media quite recently because the 'Obama stimulus package' includes a boatload of cash for creating a much improved electrical grid throughout the USA in order to allow far distant wind and solar power plants to distribute their power all across the country - even though they are (by necessity) situated in fairly remote an/or inhospitable places. This huge build-out of new high tension power lines will doubtless upset a great many people - and the debate as to whether they are safe or harmful over the long term has resurfaced. However, there is absolutely no doubt that if you get within some short distance of the cable itself, you could come to a great deal of harm! SteveBaker (talk) 16:36, 25 February 2009 (UTC)[reply]

The prime element in the lethality of electricity is the internal resistance of the voltage source. If internal resistance is high, only a small amount of current can flow in an external circuit. It is current that harms a person. A high voltage can impose a shock, but will not cause harm if the internal resistance of the voltage source is high. There are other factors involved, such as the amount of sweat on the skin, place of contact, etc. But internal resistance is the prime element. The internal resistance of a voltage source is often overlooked. For instance, the Wikipedia articles "Cattle prod", "Electric shock", "Electroshock weapon", and "Taser" make no mention of it. Also, electronic discussions in general often ignore the internal resistance of a voltage source when it should be mentioned. – GlowWorm. —Preceding unsigned comment added by 98.16.66.64 (talk) 17:25, 25 February 2009 (UTC)[reply]

Internal resistance is very rarely high enough to avoid electrocution! Dbfirs 17:40, 25 February 2009 (UTC)[reply]
In reply to the OP:
1. No, it is not true. There are a few special types of very high-voltage sources that are not always lethal to humans (like this), but that in no way justifies the general claim that 'there is no longer a risk'.
2. See 1.
Electric shock: the statement in that article is unhelpful, but it probably means that more people get electrocuted by 110 to 250 volts because that is the voltage range that is supplied to practically every building in the developed world. That's why these voltages cause more deaths than any others.
--Heron (talk) 18:41, 25 February 2009 (UTC)[reply]

Could someone with CIPA still die from a chest stab?

(First off, why isn't there a Medical section of the RD?)

If someone with CIPA (the inability to feel pain) got stabbed in the heart, would they still live since there's no pain to kill them? --129.130.239.68 (talk) 15:50, 25 February 2009 (UTC)[reply]

Pain isn't lethal, but bleeding (from being stabbed in the heart, say) can be. I'm dubious that someone who can spell "anhidrosis" doesn't know that. --Sean 15:55, 25 February 2009 (UTC)[reply]
It was pasted, not spelled, Sean. --129.130.239.68 (talk) 16:10, 25 February 2009 (UTC)[reply]
There isn't a medical section of the RD because we aren't allowed to answer medical questions. General questions about biology (which you might reasonably describe this one as) belong here - with the other science questions. We generally resist the temptation to add new RD sections because we only have a fairly fixed number of respondants and increasing the number of desks doesn't really change that...if anything, it makes matters worse because someone who is not generally interested in answering 'medical' questions wouldn't see your question because they wouldn't patrol that desk. Putting the question here in science guarantees the most 'eyes' on the problem. SteveBaker (talk) 16:27, 25 February 2009 (UTC)[reply]
People or animals under general anesthesia, or unconscious due to lack of oxygen, also do not feel pain, but can die from blood loss associated with stabbing or cutting, shooting, decapitation, or other Wounds, as the original questioner might realize on a bit of reflection.The article Death may be helpful in understanding how Life processes in animals can be interrupted even without the experience of Pain. Edison (talk) 17:44, 25 February 2009 (UTC)[reply]
Pain is not what kills you when you are stabbed in the heart, generally. Blood loss, probably blood pressure dropping, things like that—that's what kill you. Whether you feel it or not hardly matters except in cases when you even have time to go into shock—I suspect getting your heart stopped is not one of those cases. --140.247.243.27 (talk) 20:59, 25 February 2009 (UTC)[reply]
Technically, it's actually because:
  1. Your heart is enclosed in a tough protective sac called the pericardium.
  2. If a wall of your heart is punctured, every time it tries to pump blood through your body it squirts some into the pericardium.
  3. It doesn't take long for the pressure from the blood outside the heart, in the pericardium, to make it impossible for the heart to pump blood.
And as the worthies before me have noted, this is completely unaffected by the ability (or lack of ability) to feel pain. arimareiji (talk) 23:08, 25 February 2009 (UTC)[reply]
How can you stab a hole in the heart without also putting at least one hole in the pericardium ? So, then, why would pressure build up there, as opposed to leaking out of the pericardium into the abdominal cavity or completely out of the body ? StuRat (talk) 19:19, 26 February 2009 (UTC)[reply]
Spot on, Stu. Arimareiji was trying to describe cardiac tamponade, which most commonly occurs because of pericarditis with a fluid effusion, and from causes of cardiac bleeding, such as rupture of a coronary artery, such as might occur as a result of atherosclerosis, or from rupture of a ventricle wall, such as might occur following a myocardial infarction or as a consequence of a ventricular aneurysm. The atria of the heart don't tend to rupture because they are low-pressure chambers. Tamponade may also arise from other conditions leading to fibrosis of the pericardium. If you've been stabbed in the chest, then the things that will cause you to die are 1)blood loss due to perforation of the heart, great vessels (aorta, pulmonary trunk etc) and/or lungs; 2)Hypoxia, as a result of either 2a)transection of the trachea, or either bronchus in the mediastinum; 2b) interruption of blood supply to the lungs. Furthermore, any mediastinal or lung injury will cause either a pneumomediastinum, haemopneumomediastinum, pneumothorax, or haemopneumothorax; any of these can go on to become the "tension" variety, which causes a mass effect in the chest, which exacerbates either the tamponade and/or respiratory compromise, hastening death. No-one ever died from pain alone! Mattopaedia (talk) 07:17, 27 February 2009 (UTC)[reply]
It does seem like pain could cause death, via the following mechanisms:
1) Acute pain and the associated stress could trigger a heart attack, stroke, or other fatal events.
2) Chronic pain and the associated stress could trigger or worsen an existing medical condition, leading it to be fatal where it might have otherwise been survivable.
3) Chronic pain could cause the individual to choose suicide. StuRat (talk) 15:46, 27 February 2009 (UTC)[reply]
...Time to reset the indent...

Well, no. You see, all of those are indirect causes of death associated with pain:

  1. Pain is the spinocerebral response accompanying nociceptive stimuli. In other words, pain is the aversive signal you get when unconscious signals tell your brain some part of you is being damaged. It is not damaging in itself.
  2. Pain is subjective. All people percieve pain differently for a given stimulus. Our article on pain even suggests there are heritable factors related to this.
  3. We all respond differently to pain.
  4. Our responses, which are largely conscious decisions and therefore largely modifiable, have a significant bearing on the consequences of the pain:
    1. If one chooses to panic, then one will experience emotional stress. If it happens one also has ischaemic heart disease, a cardiomyopathy, or a vasospastic disorder like Prinzmetal's angina, the result of the physiological effects of the stress (the adrenergic effects of the fight or flight response) could induce enough transient ischaemia in the myocardium to result in myocardial infarction, or could elevate blood pressure enough to rupture a weak cerebral artery and cause a haemorrhagic stroke.
    2. If one chooses not to seek appropriate assistance to learn to develop effective coping strategies for their chronic pain, then one may develop a reactive or situational depression, which, if untreated, can lead to attempting suicide.

In each example you cite it's not the pain that's primarily responsible for death. It's associated, sure, but ultimately it's our response that determines the consequences. Hence my earlier assertion: no one ever dies from pain alone. Mattopaedia Have a yarn 02:43, 28 February 2009 (UTC)[reply]

You seem to be assuming that pain management strategies which are 100% effective are available to all. I doubt that they are. Thus, some who seek help will still suffer from the consequences of pain, possibly including death. I also forgot to mention a fourth way that pain can kill:
4) Pain can make exercise difficult, and thus can lower the level of fitness of an individual, leading to health problems like obesity, which may eventually cause death. StuRat (talk) 14:31, 28 February 2009 (UTC)[reply]

Over at the colors of noise article, there are several images of power spectra of the various "colors". While I am not an expert in these I do know the difference between white and pink noise. When I looked at their respective spectra images however, what was called white noise was flat on a logarithmic scale, yet that is the definition of pink noise. I am afraid that several other images may be wrong as well. Could someone with some expertise here check out those images and figure out what needs to be fixed? Thanks, Baccyak4H (Yak!) 17:50, 25 February 2009 (UTC)[reply]

I know very little about noise, but there does seem to be a contradiction between the text and the graphs in that article. I think the horizontal scale on the graphs is just labelled incorrectly. --Tango (talk) 18:02, 25 February 2009 (UTC)[reply]
It is not just the axis labels; the grid is logarithmic as well. For the "white" noise image, I suppose it could be used to illustrate pink, but again they all look like they were made together, so I worry about all of their accuracy.
I may try and figure out which image is which by reading the descriptions in the text, and shuffling around as needed. It is humorous that the illustration for one "color" might use a file with a different color in its name. Anyway, more eyes, and hands, welcome there. Baccyak4H (Yak!) 18:19, 25 February 2009 (UTC)[reply]
I've attempted to sort this out. The correct white noise image was hidden away in the Deleted files archive, so I restored it, and I created a new pink noise image by relabelling the previously misnamed one. The pink noise and white noise articles should now show the correct images.
I think I can see where the confusion arose. The present white noise plot is on a linear frequency scale and the present pink noise plot is on a log frequency scale. It is perfectly possible to show white noise on a log frequency scale and pink noise on a linear frequency scale, if that floats your boat, so there are two ways to show each plot. Someone had got log and linear frequency scales mixed up. --Heron (talk) 19:23, 25 February 2009 (UTC)[reply]
Good job. I now reverted colors of noise with the correct images. After checking on the file:image pages of the white and pink images, those are all the pages that use these. Also, as far as I can tell all the rest of the images appear to reflect what their descriptions say. Baccyak4H (Yak!) 19:39, 25 February 2009 (UTC)[reply]
These kinds of discussion really belong on the talk page of the article itself. If everyone who had an issue with a science article brought it up here - there would be several thousand new posts each day! SteveBaker (talk) 21:08, 25 February 2009 (UTC)[reply]
Are you suggesting we have something better to do with our time? ;) --Tango (talk) 23:50, 25 February 2009 (UTC)[reply]
Oh - no! Certainly not. What could be more important than explaining to people that gritting your teeth and ignoring the pain won't save your life in the event of a stabbing-in-the-heart incident? SteveBaker (talk) 01:18, 26 February 2009 (UTC)[reply]
(This remark is definitely talk-pagey and shouldn't be here, but I'm posting it anyway.) I see these questions as useful and productive. These questions (and answers, and fixes) show very directly how the rest of Wikipedia benefits from having the Reference Desk. While the first stop for article-related problems should be the article talk page, I see no harm in trying Ref Desk if the talk page approach fails. (I'm prepared to change that stance if we are inundated, but so far the load is manageable.) In this particular case, Talk:Colors of noise has sparse comments about incorrect figures going back to at October 2007, but no one got around to really correcting the problem.
We regularly answer questions of the form, "I read foo on this website/in this paper/on the news, but it seems to contradict bar/itself. What's going on?" If we're willing to do that for every other site on the web, we might as well do it for Wikipedia. At least with Wikipedia, we can go back and directly fix the misinformation afterward. TenOfAllTrades(talk) 15:16, 26 February 2009 (UTC)[reply]

Is (or was) Antarctica really a single continent?

Hi. Now, I realise that there really aren't really any strict definitions for a "continent", but let's look at it from a scientific prespective. I know that areas with a single continental shelf is often defined as a continent (but Europe, Asia, and Africa are considered individual continents, and the Americas are split into North and South America), and areas with separate tectonic plates are often considered one continent (as is the case with Asia, India, and Indonesia). However, what I really mean is, do we really know if Antarctica consists of a single tectonic plate, or if there were rifts resulting from a collision of landmasses that stayed together ever since? Old rifts and plate edges could easily have degraded over geologic time, but areas like the New Madrid Fault and the Appalachian Mountains in North America still cause earthquakes today, even though they are both over 100 million years old. One of my hypotheses is that there are far more rifts, faults, and plate boundaries than we know of, even if they don't reach from the surface to the mantle. So, this would make even tectonic plates not have a strict definition. However, could some of these pseudo-boundaries be responsible for some subglacial and other Antarctic features, such as the Transantarctic mountains, Ellsworth Mountains, the mountains east (clockwise) of the Ross Ice Shelf, some Antarctic Volcanoes (I know some are probably caused by hot spots, but hot spots require a weak spot in the crust above the mantle, doesn't they? Also, no tectonic boundaries are known to extend as far south as the continent itself), the Bentley Subglacial Trench, the Gamburtsev Mountain Range? I know that ice can be partly responsible for many great features (such as the Great Lakes), but the ice should act to smooth out the contrasts in elevation, not enhance them. The Gamburtsev Mountains especially seem similar to the Alps in topography, but also potentially in how they formed. Why is this such a mystery? Wasn't that part of East Antarctica supposed to have collided with Australia around the time those mountains were formed, and if not, then could additional plate boundaries be responsible? Could the stress of ice create some new fractures, and if the ice is lifted, could others potentially form (this would imply that global warming could cause earthquakes, because if it can quickly relieve stresses in the crust, why couldn't it cause earthquakes as well)? Do we just assume Antarctica is a single plate with no rift zones or other faults simply because the entire region is covered by a single ice cap that extends to the bottom of the ocean, and because we simply don't have enough information? Thanks. ~AH1(TCU) 23:06, 25 February 2009 (UTC)[reply]

I can't address all of your question but take a look at orogeny which refers to mountains formed by the actions of plate tectonics. There is a section for Antartica. Mountain#geology talks about some of the ways mountains can be formed. This website seems to discuss some of the relevant Antarctic mountain building events. I don't speak geology-ese though so someone else will have to explain the terminology.
I flipped through List of tectonic plates and minor plates which border the Antarctic plate include the Juan Fernandez Plate, the Shetland Plate, and the South Sandwich Plate. The Bellingshausen Plate and Charcot Plate are ancient plates which fused onto the Antarctic plate. The ancient Phoenix Plate subducted under the Antarctic plate and fractured. It seems that the current Antartic plate is indeed a composite of plates. Gondwana seems particularly relevant to Antarctic plate events. 152.16.253.109 (talk) 23:52, 25 February 2009 (UTC)[reply]
Also see West Antarctic Rift, which explains those active volcanoes in West Antarctica. If you feel masochistic take a look at The Antarctic Rift: Plume vs. Plate Dynamics. You are right that studying Antarctic geology is difficult due to the covering of ice, the difficulty of travel within the continent, and generally remote location.
With regards to ice, the weight of the ice is significant enough according to a picture caption in Antarctica that removing the ice would cause the landmass to rise by several hundred meters over a few tens of thousands of years. I'm no geologist but that sounds like a fairly rapid rise which would probably cause local faults and earthquakes. I'm not so sure about ice only smoothing out the contrasts in elevation instead of enhancing them. From the article glacier, glaciers deepen mountain valleys they flow through. A lot of glacier related features are fairly steep like fjords, Arêtes, and Glacial horns. It is true that glaciers can't make mountains taller - tectonic and volcanic activity is responsible for that. 152.16.253.109 (talk) 00:24, 26 February 2009 (UTC)[reply]

February 26

Mousetraps

Supposing that I was a mouse, what would be the best way to go about disabling/disarming a spring-loaded mousetrap? --84.68.65.232 (talk) 00:46, 26 February 2009 (UTC)[reply]

Tell another mouse about it first. "Say, there's some nice cheese over there, why don't you go take a look at it..." Then after he/she bites the dust, you get the cheese they missed. (Sound fanciful? I've seen mice do this, stepping over dead, trapped mice to get some peanut butter.) --98.217.14.211 (talk) 01:00, 26 February 2009 (UTC)[reply]
If your name was Jerry, then your best bet would be to trick the cat (Tom) into setting it off (probably with his tail). If you are not Jerry but still anthropomorphised, then lob something heavy onto the plate to set it off. Alternatively, you could wedge the spring down with some handy implement, but this is fairly risky. If you are an ordinary mouse, then find your supper elsewhere: those things can be lethal and you wouldn't want to risk setting it off by mucking around too close. Gwinva (talk) 01:04, 26 February 2009 (UTC)[reply]
It certainly seems that the risk of trying to wedge the thing open would be more than you'd want to take - so we should probably concentrate on setting it off. That means dropping something onto it - probably from a safe distance. If there is a shelf or some furniture conveniently above the trap - then dropping a small pebble ought to be enough to set it off...failing that - I think you'd need something long enough to provide leverage from a safe distance - yet light enough for the mouse to carry. Mice can make nests and carry their young around - so they are certainly mechanically capable of doing that. SteveBaker (talk) 01:12, 26 February 2009 (UTC)[reply]
I agree that setting it off from a distance is the way to go. You're pretty small and frail, you're fast but not that fast, and that long tail of yours is a liability if you're near the trap. These traps look crude, but they're nothing to mess around with. Take a look at our "Mousetrap" article, if you have the stomach for it—it's bloody horrifying. Anyway, if Mr. Human has set the trap right, it's against the wall with the bail out. You have to disturb the bait pedal enough to trigger the trap, but there are two problems. One is that the bail is going to come around with surprising force, tending to impart violent motion to whatever you're using on the pedal. The other is that it's going to make a loud snap, loud enough to alert your enemy and bring him around. If he does come around, you'll lose the bait, and why fool with the trap at all if you're not going to get a treat for your trouble? My advice is to see if you can find a tennis ball or something similar on the floor, and roll that at the trap. With luck, it will interfere with the trap's mechanism enough to mute the snap somewhat. But back away fast once you've rolled the ball, because the trap could tiddly-wink it back at you. Bon appetit! --Milkbreath (talk) 01:32, 26 February 2009 (UTC)[reply]
OR: I once lived in an apartment building where the local mice disarmed spring traps by turning them upside down. Don't know the way they did it because no one ever caught one at it. They weren't smart enough to evade box traps the exterminator put out. 76.97.245.5 (talk) 17:58, 26 February 2009 (UTC)[reply]
The real trick is to just avoid the mousetrap. Spend the extra time to find the food that isn't in a spring-death-machine. The effort and risk required to set it off safely is far greater than the value of the food in a single trap. --140.247.11.54 (talk) 01:42, 26 February 2009 (UTC)[reply]
A smart mouse would take a twig (or anything else long and thin) in its mouth, press down on the bait with the other end of it, and then enjoy the food. To learn how to catch rats - the big cousins of mice - read Full Revelations of a Professional Rat-Catcher. It's avaiable on Gutenberg.org. You'll have some good laughs. – GlowWorm. —Preceding unsigned comment added by 98.16.66.64 (talk) 02:59, 26 February 2009 (UTC)[reply]

Constituents of InkJet Printer Ink

What is the chemical dye/pigment that makes yellow inkjet-printer ink yellow? (And I guess that hearing about Cyan and Magenta would be interesting too). I'm thinking of the regular thermal inkjet Epson/Canon/HP types - not the fancy commercial grade gizmos. Our article Inkjet printer says it's a Volatile organic compound - but doesn't say what precisely. Ink cartridge and Inkjet refill kit were no help. I followed all of the links to various pigments in Yellow and none of them admit to being used in inkjets. ColorWen and Kolorjet Chemicals appear to be the manufacturers of the stuff - but their websites aren't much help ([18] and [19] SteveBaker (talk) 01:02, 26 February 2009 (UTC)[reply]

Many dye compounds are proprietary and kept secret by the companies that produce them. I had a friend that worked for DuPont in their titanium oxide/white pigment division, and he had to sign a non-disclosure agreement and a non-competition agreement where, if he left the company or was terminated, he could not work for a competitor in the same field for 10 years. There are probably some general ideas of things that make yellow colors, but each company probably either buys its dye from a manufacturer without knowing what it is chemically, or it makes its own ink, and keeps the formulation a secret. --Jayron32.talk.contribs 02:08, 26 February 2009 (UTC)[reply]
Googling for Template:Websearch got me lots of patents, like this one, which say things like "the coloring material comprises at least one selected from the group consisting of C.I. Direct Yellow 86, C.I. Acid Yellow 23, C.I. Direct Yellow 173, and C.I. Direct Yellow 59", which refer to dyes standardized in the Colour Index International, some of which are listed at list of dyes. --Sean 13:50, 26 February 2009 (UTC)[reply]

Particles or waves in radiation

In describing radiation one can start with a nuclear particle escaping the sun's boundary some of which find their way to Earth via the solar stream.Gauss saw these as static particles or unbound electrons when he devised his law of statics. The law of statics when broadened to include radiators while maintaining equilibrium and then applying a time vary field becomes mathematically the same as Maxwell's laws. Thus it can be stated that a efficient radiator can be any shape, size, or elevation as long as it is in a state of equilibrium. When a radiator surface is covered with static particles and a time varying current applied it is the "weak force" or Foucalt current that applies levitation with spin to the particle such that the charge accelerates in a straight line projection to a receiving resonant diamagnetic radiator to complete the communication sequence. The ejection of the particle follows the same sequence that metal salvage yards apply to sort various types of material from each other via levitation from a conveyor line. Thus the question come about with respect to radiation and light being purported as a subject of "waves", when the above points to it being a subject of "particles" ( Neutrinos?) being accelerated by the "weak force" per the standard model?Slimylimey (talk) 01:40, 26 February 2009 (UTC)[reply]

In your specific example, the solar wind is not considered radiation. However, there's wave–particle duality. Clarityfiend (talk) 02:01, 26 February 2009 (UTC)[reply]

The solar stream is the vehicle for particles from the sun to arrive on Earth (from Gauss). Radiation is created by the acceleration of a charge (from Maxwell ala time varying field ) which in this case is the unbound particle.There is nothing in the above that shows wave - particle duality behavior Slimylimey (talk) 02:57, 26 February 2009 (UTC)[reply]

I guess I wasn't clear enough. Radiation can behave like waves in many situations and particles in others (e.g. the photoelectric effect). However, I didn't mean to say that there was much duality in the solar wind. Clarityfiend (talk) 04:45, 26 February 2009 (UTC)[reply]
It appears that spammers are in action. There is no duallity present in my statement. I never said that static particles were charged. Einstein predicted that the weak force was involved in radiation as well as being part of another std model force, so who proved he was in error? How was the weak force identified. Please describe. Photo electric effect is conformation that particles and not waves are the source. I thought this was moderated to keep out the spammers! Slimylimey (talk) 14:06, 26 February 2009 (UTC)[reply]
Foucault currents have little to do with the Weak force. And neutrinos aren't charged particles, so they can't be accelerated by a electrical or magnetic field. And your question is remarkably simpler to similar crackpot ramblings that appeared last year on Usenet's rec.radio.amateur.antenna, btw. --Pykk (talk) 07:48, 26 February 2009 (UTC)[reply]

water

Is it true that Water will keep you full for longer and stave off hunger pains? —Preceding unsigned comment added by Historyfan101 (talkcontribs) 03:30, 26 February 2009 (UTC)[reply]

Longer when compared to what? Dismas|(talk) 04:21, 26 February 2009 (UTC)[reply]
Longer than without it, presumably. I don't know but you don't want to drink too much; see water intoxication.--Shantavira|feed me 09:56, 26 February 2009 (UTC)[reply]
According to this article it is better to eat water rich food to stave off hunger pangs rather than simply drink water.--Lenticel (talk) 13:24, 26 February 2009 (UTC)[reply]
Personally, I prefer eating food to cure hunger... --Tango (talk) 16:14, 26 February 2009 (UTC)[reply]
Yes, buffalo is much more satisfying than water. --Scray (talk) 17:44, 28 February 2009 (UTC) [reply]

Largest open-shelf library

What is the largest open-shelf library?--Mr.K. (talk) 12:26, 26 February 2009 (UTC)[reply]

Sorry to see this question languishing here. Perhaps it is not optimally drawing on the knowledge base of the Science reference desk (I certainly don't have an answer). I assume you searched using Google or similar engine, which does not give an obvious answer. Humanities might be a better place to ask, though I am not sure. --Scray (talk) 11:18, 27 February 2009 (UTC)[reply]

Free university

Are there any university out there that is free in both meanings of the word? Free to join like the Open University and fee-free like some European state universities?--Mr.K. (talk) 12:32, 26 February 2009 (UTC)[reply]

Does Wikiversity count?--Lenticel (talk) 13:28, 26 February 2009 (UTC)[reply]
If it is an accredited institution, yes. Is it one?--Mr.K. (talk) 13:44, 26 February 2009 (UTC)[reply]
No. APL (talk) 13:55, 26 February 2009 (UTC)[reply]
You may be under a misapprehension about 'free' universities. European state universities are rarely 'zero cost'. What happens though is that some countries fully fund university education; the University charges tuition fees, but the state picks up the tab. For example this was true in the UK twenty years ago, when I was a student. But there were conditions; you have to pass the entrance requirements, you have to keep up a minimum standard, and you have to be a citizen of the UK. If you were a foreigner who went to the same university I did you certainly had to pay fees.
On the other hand you may be interested in this: [20] train to be a medical doctor, entirely without tuition fees in Cuba. DJ Clayworth (talk) 21:39, 27 February 2009 (UTC)[reply]
Britain isn't Europe though! German universities certainly used to be free in the second meaning. If you meet the entrance criteria and get a place you can study there without fees. Most (all?) German universities now charge an "administration fee", but at ~EUR300/semester this is still well short of what you'd call a fee in the Anglo-Saxon world. Getting a place might be straightfoward as well, at least 15 years ago for unfancied subjects (such as mathematics) you could just turn up on matriculation day.195.128.250.9 (talk) 23:41, 27 February 2009 (UTC)[reply]

derivation of lorentz transformation

in the following wikibook about special relativity, [21] i am able to understand upto equation 5. But after that, einstein has wrote that Now t can be eliminated from equations (1) and (2) and combined with and (4) to give in the case where x = 1 and t' = 0:

   (6)  .

But i don't understand how t can be eliminated. Even if i substitute t with , it cancels out and it ends at . Please explain the steps very elaborately.--harish (talk) 12:42, 26 February 2009 (UTC)[reply]

Maybe you just misread what they are suggesting. First, you use equations (1) and (2) to eliminate t (if you aren't sure of how to do this read simultaneous equation). Then you get an equation in terms of , and . Now we set and use to put the answer in the form they gave. Vespertine1215 (talk) 17:28, 26 February 2009 (UTC)[reply]

Milky way part two

I finally had the chance to study the Milky Way near Sirius and Orion (new moon + not too cloudy sky). Anyways I want to verify if it was what I saw. Anyways I detected a faint band of light near Orion. Using a toy binocular, I found that it was made up of small stars that aren't visible to the naked eye. Orion also have several of these minute stars inside him.--Lenticel (talk) 14:20, 26 February 2009 (UTC)[reply]

A simple constellation map is the easiest way to verify this. The band of the Milky Way should pass above Betelgeuse, Orion's left shoulder. Note that you're going to see lots of faint stars in every direction, though, when you use binoculars. — Lomn 16:13, 26 February 2009 (UTC)[reply]
Orion is pretty big, could you be more precise? Orion (constellation) has a map showing where different things are in the constellation, you may be able to identify what you were looking at from that. --Tango (talk) 16:12, 26 February 2009 (UTC)[reply]
The most significant object in Orion is without a doubt Messier 42 The Orion Nebula, it is the single brightest diffuse nebula in the sky, visible even to the naked eye under dark skies. That might be what you saw. The Orion nebula is at the middle star of Orion's "sword". As for looking and identifying stuff in the sky, I always recommend this http://www.skymaps.com/downloads.html and this http://www.stellarium.org/ enjoy. Vespine (talk) 22:09, 26 February 2009 (UTC)[reply]
The Orion nebula wouldn't look like it was made up of stars, though, would it? I believe it contains an open star cluster, but from my recollection of when I observed it through a small telescope (I never quite got the focus right, admittedly), it was more of a diffuse cloud than a collection of stars. --Tango (talk) 22:55, 26 February 2009 (UTC)[reply]
Well, the constellation of Orion and nearby sky is a region pretty rich in stars, assuming relatively poor conditions of observation, a "faint band of light" could be the Hyades, the nearest open star cluster from the solar system, roughly at the location of Aldebaran (reddish star opposite Sirius from Orion's belt) in the nearby constellation of Taurus. A little farther in that direction, you might have spotted the most striking open star cluster in the sky: the Pleiades, also in Taurus. Orion's head above and Orion's sword just below Orion's belt are other possibilities. Equendil Talk 00:58, 27 February 2009 (UTC)[reply]
Okay I was able to confirm that what I saw was indeed the milky way (although much fainter than in the pictures) using the maps that you provided. I thought Orion's nebula was a star. No wonder it look fuzzy when viewed more closely. Thanks guys. Now I'll be hunting meteor showers and comets!--Lenticel (talk) 15:43, 27 February 2009 (UTC)[reply]
Lentice, what you have seen might be part of the Milky-Way, but if you have really never seen it, do yourself the favour and try to get out on a moonless, clear night to somewhere well away from any light source. Not sure where in the world you live, but it must be possible with a bit of effort. And leave the binoculars at home. You can't miss it if the conditions are right. 195.128.250.9 (talk) 23:48, 27 February 2009 (UTC)[reply]

opened bottled water shelf life

I tried to search to determine if this question had been asked before, but the search bar turned up quite a few threads and my time is limited. How long is a bottle of water still drinkable after it has been opened and left sitting out a room temperature? I often take one to bed and leave half a bottle on the night stand. I always throw it away the next day and grab a fresh one. I am now wondering if the previously opened bottle would still be good. thanks and cheers, 10draftsdeep (talk) 14:38, 26 February 2009 (UTC)[reply]

I don't think there is a precise answer. If you got some bacteria in it from your mouth the day before, then it may start to grow in the water. However, it may not find much food there. If you were eating while drinking the water, though, you may have gotten food particles in the water which the bacteria can snack on. This might make it bad the next day. If, on the other hand, you opened it and closed it right up without any bacteria or food getting in, it may be good for a very long time (until the chemicals leaching from the plastic ruin it).
The obvious solution is to dump the water from the bottle and refill with tap water, since that's likely what came in the bottle anyway. The only reason not to do this is if your tap water is really nasty. A bit of lemon helps the taste, in any case. StuRat (talk) 16:27, 26 February 2009 (UTC)[reply]
In the UK at least, mineral water often has a higher bacterial load than tap water in the first place. DuncanHill (talk) 16:55, 26 February 2009 (UTC)[reply]
If you left it for a couple weeks it would probably evapourate, which would make drinking difficult. Until then, it should be fine. WilyD 17:04, 26 February 2009 (UTC)[reply]
Use a glass. Put the top back on tightly after you poured your water. If you want to be extra cautious you can use a thermos-bag and toss in some Ice packs or use a bucket with some ice cubes. If you finish it the next day it should be fine without, though. (Apart from the nasties already in the water that Duncan mentioned.[22]) If you don't live in a hotel or have tap water issues like lead or silt a filter on your tap might be better. Those can have issues, too, though. 76.97.245.5 (talk) 17:21, 26 February 2009 (UTC)[reply]
Buy one of these. Fill it from your kitchen sink. Drink from it as needed. It will be identical to the bottled water you buy in the store. Remember, bottled water is made from your municipal water suply, most likely from a local bottling plant. If you are buying bottled water in a local supermarket, the people that put the water in that bottle got the water from the same place your kitchen sink does. In fact, the water from your kitchen sink is probably better than the bottled water, because compounds in the plastic readily leach into the water, especially as the water usually spends some time in hot trucks or on shelves in warehouses. Of course, if you obtain water from a well, as opposed to a municipal supply, you may be using water softeners that make your tap water unpalatable. In that case, bottled water may be a better option. But for most people, its just a scam. --Jayron32.talk.contribs 03:06, 27 February 2009 (UTC)[reply]
We have Pur water filters on our kitchen taps - they make getting rid of stuff from tap water a breeze. But bottled water is a scam - and a disaster for the environment (plastic bottles don't recycle well - shipping water in trucks is a lot worse than along those handy pipes you have running to your house)...there is no excuse for drinking the stuff. SteveBaker (talk) 03:28, 27 February 2009 (UTC)[reply]

Thanks for all the great advice. I once owned a Brita filter pitcher and have been seriously considering a new one. I guess the answer (going back to my question)is that the water is not going to spoil horribly just because it sits unrefrigerated for awhile once opened. thanks, cheers and happy weekend to all.10draftsdeep (talk) 13:40, 27 February 2009 (UTC)[reply]

television

i am struggling to get this answer . Who really invented television? —Preceding unsigned comment added by 59.92.247.138 (talk) 14:39, 26 February 2009 (UTC)[reply]

History of television seems to have a lot of information. Algebraist 14:54, 26 February 2009 (UTC)[reply]
You need to specify whether you mean the device or the application as a broadcast medium. Television is a very broad concept and thousands of scientists and inventors contributed to its current incarnations. Pick a suitable definition for your purposes, and you will be able to narrow down which accomplishment "counts" as the invention. Nimur (talk) 16:24, 26 February 2009 (UTC)[reply]
The modern television as we know it can probably be best assigned to the work of Philo Farnsworth. He certainly built heavily on the work of others, but the modern CRT-based television is pretty much identical to one he invented (well, 20th-century TV....) Interestingly, he was about 14 years old when he worked out the basics of it. --Jayron32.talk.contribs 19:43, 26 February 2009 (UTC)[reply]
Sure...but that's the cathode ray tube... Whenever I watch television, (which is rare these days) I do not use a CRT. Between the LCD display and DLP, I'm using 100% CRT-free televisions. In fact, most of the time, for my purposes, the entire broadcasting process does not use a conventional television camera (now replaced by semiconductor-based digital video cameras), conventional RF television modulation (now replaced by digital encoding), or audio processing; this is especially true when I watch "internet television" encoded digitally and broadcast over a packet-switched network. So, no part of my experience relies explicitly on the "television" as it was known fifty years ago. This brings me to my original point - "television" is more subtle to define than just a single device or invention. Nimur (talk) 19:50, 26 February 2009 (UTC)[reply]
I still have 5 CRT TVs and 4 CRT PC monitors in use in my house. Why ? Because I refuse to throw out anything that still works. Some of these are decades old. I have needed some amusing fixes for some, though, like a strategically placed magnet to prevent color distortion in one, adjusting the vertical height to avoid blurry corners in another, and cutting the speaker wires in one and attaching external speakers. Another just needs a good kick whenever it goes down to a single bright line on the screen. And, yes, if you can't tell, I'm partially Scottish (I like to say I have about a fifth of Scotch in me at any given time). StuRat (talk) 04:16, 27 February 2009 (UTC)[reply]
Phooey on LCD displays and CRTs. Just build an electronic scan converter and use a Nipkow disc which Paul Nipkow invented in 1883 to display the image. Edison (talk) 02:01, 27 February 2009 (UTC)[reply]
Reply to Nimur; the deal is, the changed since Farnsworth's original ideas have (color TV, High Def technologies (plasma/LCD), digital broadcasts, etc. etc.) represent rather incrimental changes to the basic system. Even modern, satellite television in 1080i Hig Def on a 52 inch plasma TV run on the same basic system that Farnsworth put together. It wasn't the CRT, it was the camera-broadcast-display conceptual system that he turned into a practical reality which resulted in Television. Yes, these parts had been in place in bits and pieces already, and yes, as Edison notes, earlier television systems had been tried, but it was Farnsworth's system that stuck, and became the modern Televsion system. --Jayron32.talk.contribs 03:00, 27 February 2009 (UTC)[reply]

Einsteins prediction regarding the "weak force"

(Moved from talk page. – 74  16:25, 26 February 2009 (UTC))[reply]

Einstein predicted that the solution to identifying the "weak force" lies in the subject of radiation, where it is part of or connected to another of the "standard forces" This points to the Foucault current which is part of the standard force of electromagnetic force Thus this very specific questions Who proved Einstein was in error, and what was eventually identified as the "weak force" ? Please, no spamming guessing or personal conjectorSlimylimey (talk) 16:13, 26 February 2009 (UTC)[reply]

The weak nuclear force is a fundamental force, it hasn't been identified as being something else. You may be interested in reading electroweak force, which talks about how the weak force and electromagnetism become the same thing at high enough energy levels. --Tango (talk) 17:20, 26 February 2009 (UTC)[reply]

Thanks for responding but I wanted a specific answer. Gauss established the presence of particles (static) By applying a time varying field to the statics equation so that it equals Maxwell's laws ala radiation Therefore the Foucault current should be identified as the weak force or the magnetic field generated by it. It is now a tangiable force as Einstein predicted. Why does such a question cause none specific answers? If electro magnetic can be defined as one of the forces why cannot the reaction to same be identified as the weak force without any accompanying mumbo jumbo per prior unproven theories of Feynman? To sum up, particles exist at rest on all diamagnetic materials including radiators. An applied time varying field generates a swirling electrical field (eddy field)that dislodges the particles (accelerates) by applying an accelerating force with spin for straight line projection that exceeds gravity, another standard model force.Slimylimey (talk) 18:51, 26 February 2009 (UTC)[reply]

What you do mean by "particles"? You seem to be using the word to mean something specific, rather than just any particle - are you taking about electrons, possible? Also, be careful saying "radiation" without clarification - I think you mean electromagnetic radiation, but ionising radiation is also relevant to this discussion (the weak force is very significant in radioactivity). --Tango (talk) 19:46, 26 February 2009 (UTC)[reply]

Bad medical terms give me high blood pressure...

Can anyone explain the origin of the terms hypotension and hypertension ? I get the "hypo" and "hyper" parts (aside from the obvious stupidity in using nearly identical-sounding terms with opposite meanings in a life-critical situation). But, how exactly does "tension" mean "blood pressure" ? (OK, that was the Q, now for a little rant: The terms "low blood pressure" and "high blood pressure" appear to be far superior, both in clarity to all and the lack of potentially fatal medical mistakes. They are also the same 4 syllables as the "proper" medical terms.) StuRat (talk) 19:26, 26 February 2009 (UTC)[reply]

Well, the pressure of blood puts the blood vessels under tension, so I guess it makes a certain amount of sense. I agree that hypo- and hyper- are terrible prefixes. I think doctors speak Greek just to sound clever half the time - they're not the only discipline guilty of that, though! --Tango (talk) 19:41, 26 February 2009 (UTC)[reply]
I believe it's because 'tension' means strain, and both low and high blood pressures cause a strain on the body. I agree, however, that blood pressure is a much better term, and it's used almost solely for patients. However, in an educational context, the official term of hypo/hypertension is used much more often. —Cyclonenim (talk · contribs · email) 20:11, 26 February 2009 (UTC)[reply]
Just for interest's sake, the OED counts one of the meanings of 'tension' as precisely 'pressure', especially in biological or medical contexts. So, it doesn't really need to be interpreted literally in this context. Vespertine1215 (talk) 22:18, 26 February 2009 (UTC)[reply]
OK, this explains why hypertension might mean "high pressure", but there's still no reference to blood. So, how, other than memorizing that this is what hypertension refers to, would one know that ? It could just as well mean pressure in the bowels, for example. How'd we end up with such an imprecise terminology for these conditions ? StuRat (talk) 23:43, 26 February 2009 (UTC)[reply]
Also, medical terminology is purposely outdated. That keeps it from changing. A hundred years from now, hypertension should still be called hypertension. It wouldn't be good if your diagnoses was misunderstood in ten years because terminology drifted with whatever the latest cool rap song that recently came out and the new doctor had no clue what "blood pressure" was because he only learned about "blasizzle" in school. -- kainaw 23:21, 26 February 2009 (UTC)[reply]
It's also nice to have internally-consistent terminology. 'Hyper' always means 'high' or 'too much': hyperhidrosis - too much sweating; hypervolemia - too much fluid in the circulatory system; hyperthermia - high body temperature; hypertension - high blood pressure; hyperkalemia - elevated potassium. 'Hypo' always means 'too little' or 'low': hypohidrosis - to little sweating; hypovolemia - not enough blood volume; hypothermia - low body temperature; hypotension - low blood pressure; hypokalemia - low potassium levels. Everyone has a nice, consistent, easy-to-remember set of terms, and everyone uses the same ones. There are a number of other standard prefixes and suffixes, which allow for specifically and precisely describing medical conditions in a reasonably concise way that – and this is key – all medical practitioners will interpret in the same way. TenOfAllTrades(talk) 23:39, 26 February 2009 (UTC)[reply]
"High" and "low" also have precisely consistent meanings and yet don't sound alike. They are also easier to abbreviate, as in HDL and LDL. StuRat (talk) 23:48, 26 February 2009 (UTC)[reply]
"High" and "low" are neither Latin nor Greek. They aren't even French. --Carnildo (talk) 00:51, 27 February 2009 (UTC)[reply]
You could go for "Hi-" and "Lo-", then at least it isn't English! --Tango (talk) 01:06, 27 February 2009 (UTC)[reply]
Well, I think you are being prejudiced. We should use 上 and 下. They haven't changed for few thousand years. (In case you don't have Chinese characters installed, they are the Chinese characters for up and down used to point out the reason that a dead language was chosen). -- kainaw 01:49, 27 February 2009 (UTC)[reply]
I'd be happy to use any language that doesn't have a tendency to use nearly identical-sounding words for opposites. StuRat (talk) 03:25, 27 February 2009 (UTC)[reply]

Well, then you'd have stop using English (at least), right? There are (obviously) dozens (hundreds, perhaps?) of words that while not precisely opposites sound alike but mean different things or are spelled alike but mean something different. At any rate, it would seem to be moot, because in ordinary conversation, you probably already say "high blood pressure" or "high blood sugar". Healthcare professionals are most likely use the "jargon" and they can tell the difference pretty easily. —Preceding unsigned comment added by Brewfangrb (talkcontribs) 08:52, 27 February 2009 (UTC)[reply]

We don't have to use English, or any one language, we should use whatever language is best in each case. As for it not being a problem for medical professionals, you're probably right in that it's only a problem for their dead patients and malpractice insurance providers. There are thousands of deaths each year due to medical mistakes, and I'd bet a substantial portion are due to mishearing or misreading medical terms. Can anyone find any statistics on this ? This is similar to the recent heparin overdose problem, where different dosages were stored in similarly labeled containers. And yes, "medical professionals should be able to tell the difference", but it's also inevitable that easily confused labels or terms will occasionally be confused, even by those who should know better. StuRat (talk) 15:34, 27 February 2009 (UTC)[reply]
It would be interesting to post this at the languages RD for response by someone who knew something about linguistics (Which I've done, BTW) . Seems to me that, spoken correctly, without panic, hyper- and hypo- sound rather different, since completely different mouth movements are required to make the vowel sounds. To my mind, stuff-ups occur from stuff like this because inexperienced people get flustered easily, talk too quickly & imprecisely, and don't check what they're doing (or about to do). One of the reasons doctors spend interminable years in training is to learn to deal calmly with situations most people freak out over. All that running around and yelling "STAT" a lot like they do on ER is not really what it's like. Mattopaedia Have a yarn 02:57, 28 February 2009 (UTC)[reply]
You've reposted this Q on the Language Desk ? Can you provide a link ? If we could guarantee that every medical professional always said and wrote everything clearly, you might have a point. However, doctors still can't write a prescription that patients can read, and less experienced medical professionals, like interns and nursing aides, are also involved. So, in the real world, we need words that are difficult to confuse. StuRat (talk) 14:22, 28 February 2009 (UTC)[reply]
I found it here: Wikipedia:Reference_desk/Language#Similar_sounding_terms.3F. They seemed to agree with me that the words look similar and sound similar in many accents. StuRat (talk) 15:47, 28 February 2009 (UTC)[reply]

"Treatment" for Aspergers Syndrome?

Ok, no medical advice sought, none given. Peachy, but are there treatments recognized for the above? Sensitivity training aside, I'd look it up myself IF i had time (STEVE BAKER HELP ME HERE>>>PLEASE!!!). I know that there is a syndrome for everything nowadays but it appears to simple little me to be the same as cynicism with a healthy dose of paranoia and Compulsive behaviour thrown in? Please excuse my ignorance if it is otherwise. Furthermore, what if it's just a case of trying to care and then getting worn down? What if i just tried too hard to care too much and couldn't carry it off as I age? Messiah complex? I tried to save the world, the world wasn't interested! Furthermore AGAIN, what if I can express emotions, especially those that I don't feel (empathy especially) Practice,practice practice! But I did want to be the First CAnadian Pope when i was young BUT i settled for being an environmentalist ( both equally futile ambitions BTW). I care, for the whole planet my fellow man and the cosmos as we know it. Ok 1 last quick Q? Would this then, not make me prone to addictions to escape what i see as my dark side or is there a genetic connection there TOO (I'm Metis, i inherited the worst of both worlds and the good of ????).

Sorry but many of the above q's are rhetorical, but the opinion of others is always welcome, if not appreciated. Again sorry if I come across as arrogant. I'm not, IMHO. —Preceding unsigned comment added by 67.193.179.241 (talk) 22:25, 26 February 2009 (UTC)[reply]

I'm sorry, I don't really know what you're asking. That was a rather incoherent stream of consciousness... As for your main question, have you read Asperger's syndrome#Management? I don't think there is any treatment beyond managing the condition and learning how to cope with it. I'm sure Steve can give you a more authoritative answer once he gets here. --Tango (talk) 22:52, 26 February 2009 (UTC)[reply]


Regarding treatment, "Some researchers and people with AS have advocated a shift in attitudes toward the view that AS is a difference, rather than a disability that must be treated or cured." as our article says. So, why have a label, then. One reason is to be able to identify individuals with needs that are not met with the training and facilities offered by the standard systems. Programs facilitating the "management" Tango mentioned might not be offered if there wasn't a label. Another is for job security. A boss could send someone to a stuffed business meeting presenting group results. Ordinarily if s.o. would say, "I can't deal with crowds." said boss would have the right to say "Get over it, or leave." An Aspi can't get over it. If the boss would knowingly fire someone for being put in a situation they couldn't handle the employee could take measures like complaining to a level higher up or HR or sue if they were fired. As Steve pointed out in an earlier post (see the archives) the unique workings of a mind that fits under the "Asperger" umbrella can also offer a lot of benefits. Being left-handed used to be considered an aberration that you could "treat" people for. Thank goodness the people who consider homosexuals as just "choosing" to want to be different (and/or needing to be treated) are getting fewer and fewer. There are people who find it hard to cope with the fact that we humans come in many varieties and those who don't fit the mold don't need to be beaten into shape. 76.97.245.5 (talk) 00:30, 27 February 2009 (UTC)[reply]

Sorry but doesn't that ALLOW us, as ASPIES , the right if not the priveledge of not "trying" to change? Sorry again, but labels seldom fit in the space allotted to humankind? Again my apoligies if i seem condescending, I AM trying to understand!!!! I'm honestly trying just to understand/comprehend my place in this dark old world. Because i may be eloquise or verbiose (or too drunk to type apparently!). Please don't take take me too seriously, I don't. I'm not looking to be ENABLED, I'm just looking to "be myself". And no, thanks for your concern, BUT I AM NOT CONSIDERING SUICIDE!!!! Cheers For NOW, , must sleep. SHiftwork sucks. as does 12 hr shifts!! See you all in the dreamworld but Tango, we'll see you sooner ( in my nightmares...)! Sorry, my Thanks for your expediate replies, i KNOW you are only trying to help. Myself? I try not to hurt the other animals, the earth, people/plants or fungi But I find myself "not up to the task" repeatedly. I can only hope that i haven't offended anyone's "common sensibilities" with my diatribe(sp?) and drunken drivel AND in the END I've done more good than harm. Cheers Vin. Save page before i dribble on. —Preceding unsigned comment added by 67.193.179.241 (talk) 01:48, 27 February 2009 (UTC)[reply]

First and foremost - this is a 'spectrum' disorder. At one end normality - mild geekishness - then "Aspergers" - then more severe stuff through to profound, terrifying Autism. There are no definite limits...no solid definitions...so the labels don't help much. There is a point on that spectrum where the problem is extreme and any benefits are irrelevent. Having a cure available for those people would be a wonderful thing. Equally there is another point where a cure is not only unnecessary - but actually agressively NOT wanted. That's because it's not a one-sided condition. There are benefits. And I'm also left-handed...which is a similar deal.
So where is that line? It's different for everyone. I for one don't want to be "cured" - since I've learned where my limitations are - and that I can do a reasonable (but imperfect) job of working around them - I'll take the benefits anytime. But I can't (and won't) speak for everyone - because we're all at different points on the line.
One of the defining features of Aspies is that we can't automatically figure out what other people are thinking...that's one of the things that makes big meetings so uncomfortable. Strangers are also a problem because you don't (yet) understand them - and they don't understand you. But we aspies are good with 'mechanical' things - things with rules - things that behave predictably. The trick is to understand the rules of the interpersonal-relationship game. So if you have to go to meetings - give yourself things to do - watch the other participants' body language - respond to it. Find the people you are agreeing with - now play this silly game where you fold your arms when they fold their arms...lean back when they lean back. Wait a few seconds after they change posture - then mirror it...look at the other people and their poses - did the guy just change his mind and switch 'body-language' teams? It seems stupid - but this is what normal people do - they don't KNOW they're doing it - but you can learn to fake it. You just can't do it unconsciously and automatically. A meeting is a process - treat it like that. You have tasks - taking notes - formulating replies - handling body language. This is a mechanical process like programming a computer or and you can do it.
Your place (well, MY place - at least) is to do what you can to fit in - because that's what makes life smoother. Perhaps you shouldn't have to - but there is little point in trying to fight the system. So fake the 'fitting in' thing - then use your abilities to get what you want.
If you are on the other side of the line - where your downsides are too severe to allow you to function in society - my heart goes out to you. All I can say is: Get Help. There are people out there who can coach you in the interpersonal skills stuff that you lack. Unless you are falling severely into the Autism end of things, you CAN make your life work out better by using your ability to get obsessional and use that laser-like focus to get interested in figuring out what makes humans tick. The right trainer can tell it to you like it's a mechanical system...how often to make eye contact - how to mirror other people's body posture when you agree with them - and don't mirror when you don't - how to recognise the facial expressions and body postures that tell you that you're talking for too long on a subject that nobody else cares about.
It's just like you learned the latin names of ALL the dinosaurs when you were a kid...it's a pile of arbitrary stuff that you can get into. All of these things can be turned into formulaic rules - follow the rules and it's like magic. Sure you're only PRETENDING to do what the 'normals' do magically and without thinking...but that doesn't stop it from working. So revel in what you can do - use those special talents. Stop feeling sorry for yourself. Sure you'll screw up - lots of times. But you're lucky - you know! I didn't find out until I was already in my mid 40's for chrissakes! I look back on some of the TERRIBLE faux-pas I made in my teens, 20's and 30's...it makes me shudder! But now I know and understand the problem - I can use my skills to overcome the worst of the difficulties.
If our OP wants to email me personally (you'll find a way to email me on my Talk: page) I'll be happy to chat in a calmer, less public environment. SteveBaker (talk) 03:18, 27 February 2009 (UTC)[reply]

Validity of Wikipedia

So for my Inquiry class I am required to give an oral presentation on a topic of my choice (as long as it's science related). Pretty much everyone in my class bashes Wikipedia, saying how it's so inaccurate, blah blah blah, so I'm thinking about giving a presentation of the validity of Wiki (not as a substitute for scientific journals, obviously, just as an equivalent source to other encyclopedias). However, there's an obvious problem; most of the evidence for this (at least the stuff I can find) is on Wiki itself, which is not considered a valid source (there's been a lot of talk about paradoxes on the ref desk lately and clearly this is one of them, but I cant remember what kind). Is there substantial evidence supporting Wiki not on Wiki? There's also the problem of my prof and my peers tearing my argument to shreds immediately following the presentation, and that seems even more difficult to avoid. Any advice? -Pete5x5 (talk) 23:52, 26 February 2009 (UTC)[reply]

Wikipedia has an article on everything, including Reliability of Wikipedia. Particularly pay attention to the reference section - (theoretically present in every article) - in communities which do not respect Wikipedia outright, reliable third-party references will still carry weight in the discussion. Be sure to emphasize that Wikipedia is an encyclopedia and is not a general replacement for research journals, newspapers, and other types of references. The most general complaint I hear levied against Wikipedia is that the anonymous contributions result in non-trustworthy, non-expert opinions. Consider refuting that argument from a sociological standpoint. Compare various definitions of anonymity, and the construction of trusted networks (i.e. a "peer review" process is typically performed by people you do not know - why do you trust them?) This actually has quite a bit of subtlety, if followed to its logical end. ("Reputable journals are trustworthy because they are reviewed by reputable people..."). Such reasoning, I believe, inevitably devolves into an admission that anonymous sources must be trusted, to some extent; and that the majority of the burden lies with the consumer of information to really evaluate quality and credibility. As for the expertise of the anonymous sources... isn't it self-evident? Nimur (talk) 00:39, 27 February 2009 (UTC)[reply]
Take a look at ANY of these articles, and challenge your audience to find the flaws. They are scrupulously referenced (look for footnoted references in Britannica sometimes, eh?) and well written. Ultimately, what makes Wikipedia work as a reliable source is the huge emphasis we place on verifiability espcially with specifically footnoted statements. Now, take the same audience, and ask them how they would grade a scholarly paper which was referenced to Encyclopedia Brittanica and World Book Encyclopedia. They would laugh the student who wrote that out of their classes. And Brittanica And World Book don't tell you their original sources, so you can find the good stuff yourself. Wikipedia does. So which is a better research tool? Britannica or Wikipedia? See where I am going with this. People often cite some shitty unreferenced article about a soap opera character or a garage band or a train station and say "See, this is why Wikipedia sucks". But show them the text of a Featured Article, and don't tell them it's from Wikipedia, and they will likely say its great writing and research. --Jayron32.talk.contribs 02:52, 27 February 2009 (UTC)[reply]
If you can get access to Nature online, this article] is spectacular. Wikipedia compared favorably with brittanica online in 50 randomly selected articles (er....I think that was the gist of it. I actually don't have access to the article myself, and it's been sometime since I read the original). --Shaggorama (talk) 04:20, 27 February 2009 (UTC)[reply]
This has details about the Nature findings. Clarityfiend (talk) 04:40, 27 February 2009 (UTC)[reply]
It's not that spectacular... there have been many concerns with whether it was a meaningful study. See Wikipedia:External_peer_review#Nature_.28December_2005.29. IMO it doesn't say much about how great Wikipedia is, it just points out that EB and other encyclopedias aren't exactly perfect either. --98.217.14.211 (talk) 14:31, 27 February 2009 (UTC)[reply]
Wikipedia isn't a source reference but if you really really want to reference it as a source you should probably specify a time stamped version of an article rather than the latest version. It may have a mistake that's been corrected later but at least you'd be specifying what you saw. Dmcq (talk) 16:09, 27 February 2009 (UTC)[reply]
As I have experienced, Wikipedia is excellent for science, but bad at history and politics with all those ninja-editors "let's alter statistics in every article to make our country/religion/ideology look more favorable, and do it in a group so we can accuse our reverters for 3rr breaking. If someone argues against us, we label them as nazis, racists, etc.."
Besides this, however, Wikipedia is very good and trustworthy in science-related articles, or, every type of article that is not heavily based on political interests. --131.188.3.21 (talk) 16:24, 27 February 2009 (UTC)[reply]
The trouble is that while FA's are excellent, there are only 2,400 of them...out of 2,800,000 articles - that's less than 0.1%. For the rest, readers need to exercise some care. Sure, there are statistical studies that have been done that show that we are at least as accurate (on a random sampling) as the other encyclopedias out there. But when you are standing in a large, university library looking for a specific scientific fact - you DON'T start off by reading Encyclopedia Britannica. You go to the stacks - look in the correct Dewey-decimal section and start browsing books. Encyclopedias are for not-critical stuff...and for that, Wikipedia is head and shoulders above the rest - for breadth of coverage - for documentation of sources - and for traceability of authors.
Take some examples:
  • If you are watching a movie on TV and you wonder who some actor is - Wikipedia is a spectacularly good resource because even if there is an occasional error in who played who in what movie, it's hardly a critical thing.
  • If you are a Doctor looking up treatments for obscure diseases - then you'd better not use what you read here as the basis of a life-or-death decision. In that case you should use Wikipedia ONLY as a means to find links to 'reliable sources' that you can check.
It's still incredibly useful...but not as a primary source. Somewhere between those extremes is the place where things get tricky. Suppose you are a fact-checker for a newspaper - you might be happy to use Wikipedia to confirm things that you already pretty much believe to be true...and if you're really not certain - you can check the references. In the end, you simply have to use your best judgement. You can see with most articles whether they appear solidly written or not - you can check the sources - you can look back at the edit history and see how long the particular fact you are interested in has gone unchallenged - whether it's been the subject of repeated revisions or revert wars. Heck, you can even find out who precisely wrote every single word in every single article and you can go to that person's talk page and ask them how they know that...and you have a better than even chance of getting a good reply too! The breadth of coverage is now so great that there are frequently multiple articles written by different people with overlapping coverage - you can follow links and confirm that all of the versions of the same information agree. There is no other resource on the planet that gives you those options. So Wikipedia is DIFFERENT. You have to learn how to use it well...and (most importantly) when NOT to use it!
SteveBaker (talk) 16:56, 27 February 2009 (UTC)[reply]
As for profs and peers tearing your arguments apart, it is best to be prepared. While coming up with your talk, at the same time come up with rebuttals for all your main points by playing the devil's advocate. Then come up with good counter arguments to those. By doing this you will know the strong and weak points of your argument beforehand and will be able to defend your viewpoint without having to come up with arguments on the spot. When it comes time to talk, if you can't keep it all in your head bring a list of quotes with the sources from references. It may be troublesome if your peers use Anecdotal evidence as counterarguments because it is easy to cherry-pick examples where something has gone badly wrong (i.e. stumbling onto the Evolution page after someone replaced it with the first chapter of Genesis). I'm not sure what you can do about this other then point out that arguing from one or two examples isn't a solid argument.
One of what I think is Wikipedia's strongest points is its networking: the unparalleled-by-other-information-sources connection of related or pertinent topics to one another through Wikilinks, see also sections, categories, and lists. It makes searching for related topics much easier. Like Steve Baker said above, wikipedia is transparent. You can see who worked on what and all past versions of the article. It is also in real time, many Wikipedia articles can and do keep up to date with news and findings. Sifaka talk 03:10, 28 February 2009 (UTC)[reply]

Those are all very good points. I hope I actually can do my presentation on this (is a study of evidence scientific enough? We'll find out!) because I'm convinced that I can sway a lot of people's opinions. I may put a presentation together anyways, just to shut people up. They'll tell me that Wikipedia's BS and I'll be like "please watch my 4 minute powerpoint and you may just change your mind.." -Pete5x5 (talk) 07:40, 28 February 2009 (UTC)[reply]

Tut tut, WP:NPOV :) Dmcq (talk) 18:39, 28 February 2009 (UTC)[reply]

February 27

can hydrostatic testing damage household pipes?

I understand that hydrostatic testing is a method for testing pipes for leaks. There are a number of plumbing companies that offer diagnostic services where they run water into the plumbing system via the sewer line and then monitor the pressure to see whether any leaks could exist underneath the house. My question is whether introducing pressure to a system of pipes (particularly those of an older, 1940's era house) could actually induce a rupture that would then be interpreted as an existing leak -- thus costing money to "repair" the defect. Does anyone have references on such an issue? —Preceding unsigned comment added by 64.91.177.4 (talk) 00:25, 27 February 2009 (UTC)[reply]

If there is already a weak spot in the pipes, then hydrostatic testing can convert the weak spot into a leak. However, if the pipes are already that weak, then ordinary events (fluctuations in supply pressure, or water hammer from closing a faucet too quickly) can also cause the weak spot to leak. --Carnildo (talk) 00:59, 27 February 2009 (UTC)[reply]
Applying higher than normal presssure to any system of pipes could certainly be the cause of them breaking. A stress test can cause the tested system to break. If if has a leak, the test might help locate it. If it is about to fail, the test could demonstrate that. But a pressure test is not a nondestructive test, since it can cause a marginal system to fail sooner than it would have. I could see hydrostatic testing as a useful acceptance test on a new system. Ask yourself if there is any way that applying high pressure to an aged system could improve it? Edison (talk) 01:55, 27 February 2009 (UTC)[reply]

Macdonald 80 Shopping Center

Can anyone help me find references for Macdonald 80 Shopping Center ?, maybe in the East Bay Express, Contra Costa Times, West County Times, San Francisco Chronicle, Richmond Globe, Richmond Post, KTVU, KRON or any local affiliate of the broadcast channeles, NBC CBS FOX Univisión, etc. Richmond City Council minutes, the developer. Any help would be great!Troyster87 (talk) 02:25, 27 February 2009 (UTC)[reply]

Isn;t that a little backwards? Before deciding if the shopping center merited an article, shouldn't you have gathered your sources first? I mean, there is a very good chance that those sources don't exist at all... See WP:YFA for the proper way to build a new article, and notice that the first step, befre writing, is to gather references. Also, what is this doing at the Science Ref Desk???--Jayron32.talk.contribs 02:44, 27 February 2009 (UTC)[reply]
Engineering is a science, and this is an engineering project, and article about it, where should i have asked? and the sources exist, i have seen them. i just can't locate them, but it was reported in some of those newspapers and on websites. i am not questioning the notability, as it is a notable place, furthermore there is a precedent for this sort of article and nearly every other shopping center, district, and mall in the bay area has an article. some much smaller and far less notable!Troyster87 (talk) 00:25, 28 February 2009 (UTC)[reply]
I don't know about all that. I just know that, before I sit down to write a research-based document, I want to have the sources in front of me before I start writing. It seems a little bass-ackwards to create an article under the "I read this somewhere I think so I'll just go ahead and write all of this down." This has nothing to do with any of the other equally poorly thought out articles you may or may not find at Wikipedia. You wanted to know about your article. What you should have done before creating your article is to gather your sources first, and then used them to write your article. Just diving off the deep end and creating an article on the "maybe I can find something about this later" mentality is the cause of nearly every deleted article. People who do it the right way, by having their sources first, never get their articles deleted. If you contact the news sources you have listed, they likely have archives and concordances and things like that, and should be able to direct you to the right issues where the information is located. Just contact the newspaper and ask for the research department. Then explain that you are looking for articles about this shopping center. They can direct you to the right issues where you can find information. --Jayron32.talk.contribs 03:30, 28 February 2009 (UTC)[reply]

Psychological effects of norepinephrine

What are the psychological effects of norepinephrine? NeonMerlin 03:46, 27 February 2009 (UTC)[reply]

Did you read Norepinephrine? Or your textbook? --Shaggorama (talk) 04:14, 27 February 2009 (UTC)[reply]

I hope this Q doesn't put you to sleep...

Is there a device that can detect when a person is asleep ? My Dad seems to have narcolepsy, and falls asleep all the time during a conversation, then wakes up later and continues as if he never dropped off. The issue is with him trying to watch a movie. He is likely to fall asleep several times during the movie, but not realize when, or even that, he fell asleep. At the end he says "that movie was terrible, I couldn't follow the plot at all". It would be nice if there was a device that could detect when he falls asleep, hit the pause button, then unpause when he awakens (although he could do this last part himself, I suppose). He used to enjoy movies, and maybe he could again in this way. StuRat (talk) 04:24, 27 February 2009 (UTC)[reply]

I can't think of what it's called, but I remember once seeing a device that kind of hung off your ear and if your head fell to one side it would beep. I think it just had a motion sensor in it, simple, yet probably effective most of the time, as long as your head's not propped up. I don't know if waking him is what you're after, but this is a start at least. I'll post a link if I find it. -Pete5x5 (talk) 05:31, 27 February 2009 (UTC)[reply]
This is on the right track. Next I need a variant of this device that can send a signal to a robotic hand that can press a pause button. Any thoughts ? Could I open up the ear device and connect a wire from it to a robotic hand ? StuRat (talk) 14:26, 27 February 2009 (UTC)[reply]
Polysomnography --Mdwyer (talk) 06:34, 27 February 2009 (UTC)[reply]
Yes, an electroencephalogram (EEG). You probably should read "The Promise of Sleep" by William DeMent, ISBN 0440509017. It discusses the issues you're asking about extensively, mentions treatment options, etc. 207.241.239.70 (talk) 06:48, 27 February 2009 (UTC)[reply]
Does his doc know and is/was he taking any medication? Some meds like Zolpidem, Thalidomide, antidepressant (stopping those)etc. can have sleep disorders as side effects. His physician could then send him to a sleep center if necessary. I think this is what Pete was looking for [23]. They tested something similar on truck drivers at a shipping agency I had a project at. They ended up not using it because lots of drivers didn't like it. 76.97.245.5 (talk) 08:36, 27 February 2009 (UTC)[reply]
Narcolepsy is a rare cause of excessive daytime somnolence. There are several more likely reasons why your dad is falling asleep. If this is a problem for him, I recommend referral to a sleep specialist. Axl ¤ [Talk] 08:32, 27 February 2009 (UTC)[reply]
I thought narcolepsy WAS excessive daytime somnolence. That is, I thought it was a symptom, not a cause. It is most likely a side effect of one of his many medical problems (prostate cancer, kidney failure requiring hemodialysis, diabetes, etc.) and the meds he takes to fight them, but stopping those meds isn't an option in most cases, so we need to find a way to live with this condition, instead. StuRat (talk) 14:26, 27 February 2009 (UTC)[reply]
If you believe that your dad has a medical condition such as narcolepsy, you should advise him to seek information from qualified medical professionals, such as a doctor, and not random strangers on teh intrewebz. If he genuinely has a problem, then doctors can prescribe devices or medicine to help him. --Jayron32.talk.contribs 15:32, 27 February 2009 (UTC)[reply]
[To StuRat] Narcolepsy is a diagnosis of itself, with specific recommended treatment. It is only one of the causes of excessive daytime sleepiness. Axl ¤ [Talk] 15:40, 27 February 2009 (UTC)[reply]
Some cars have been shown (mostly a 'concept cars') with devices that detect when the driver is falling asleep at the wheel. I believe they have cameras that look at your eyes and check things like blink-rate and duration. I have yet to see one sold commercially though. This paper talks of using heart-rate changes to predict when someone is about to fall asleep while driving. Following the references from that paper might yield something useful. SteveBaker (talk) 16:32, 27 February 2009 (UTC)[reply]
Going back to the ear device, instead of getting it to activate a robotic arm you could instead get it to send an RF (radio frequency) signal to an RF repeater, changing the signal to IR, and pausing the DVD player. My dad designs and creates custom remote solutions, so I know this is possible, you'd just need to butcher an RF remote and somehow get the device to activate it. Getting the parts shouldn't be hard, but it will be moderately expensive (A good, reliable remote that's RF will run you $200, an RF repeater is about $60, and then the cost of the device (these costs in Canadian dollars)). It'd be really cool if you did it, but it would be a lot of trouble. -Pete5x5 (talk) 19:40, 27 February 2009 (UTC)[reply]
That is getting pricey, and I have no problem with wires. Is there a cheaper way to do it with wires ? StuRat (talk) 22:15, 27 February 2009 (UTC)[reply]
Well, if you have some kind of device that'll reliably detect when he falls asleep - it should be very simply to modify a 'universal' TV remote to pause the TV. Presumably there is no need to automatically unpause it since when he wakes up - he'll see that the video has paused and realise. He can then un-pause it himself. The only issue is that most DVD/VCR's will drop out of pause and simply stop playing after a couple of minutes. But assuming that's not such a huge deal - you can usually 'train' a universal remote to produce any command on any button. Carefully dismantle the remote - remove all of the buttons - you'll see the two copper 'pads' beneath that the button will connect when you press it. You'd carefully solder a thin wire to each pad and connect them up to either a small relay or an opto-isolator (both parts you can get from RadioShack for a few bucks)...whatever signal comes out of your 'sleep detector' can be used (possibly via a transistor) to drive the relay or opto-isolator to 'connect' the two pads on the remote just like you'd pressed the button yourself. No robots required! However, the means of detecting sleep may be tricky. Does his head change position as he 'nods off'? If so, something as simple as a mercury tilt-switch would work (you find them in old air conditioner thermostats for example). Something like that could be connected directly to the remote's button pads with no other electronics whatever. SteveBaker (talk) 03:26, 28 February 2009 (UTC)[reply]
Before you shovel out money on parts for a modification you should first try and see if it works at all. Automatism would have him lose awareness, but the ordinary signs of sleep would not result. A device that reacts to him literally "nodding off" would be useless if his neck muscles didn't get the information that the consciousness was taking a break. I was by no means trying to imply above that he should stop taking his medication. (Some substances can cause sleep disorders when stopped rather than while taken, actually) You should inform his doctor, because he may need to know and apparently your dad's not aware of what's going on. There's an off chance that he could switch your dad from product A to product B, or add pill C to your dad's regimen and things might improve for a bit or at least be kept from deteriorating. Sounds like, in the longer term, you should cast around for things that keep your dad occupied and allow for him to be absent for a bit, or keep his acuity up for the duration. See if he likes puzzles or playing games. --76.97.245.5 (talk) 22:30, 27 February 2009 (UTC)[reply]

A doctor can prescribe a sleep study at a hospital. to see if someone has some condition such as Sleep apnea, which is sometimes treated by a CPAP. As for falling asleep during a movie, the proof of it is starting the movie from the beginning when the person is well rested and having him note where material appears that he does not remember from having watched it. One might also sometimes point out the red stain from where the glass of merlot spilled, depending on the cause of the somnolence. The mind is good at filling in when the person sleeps through part of a movie, just as the brain fills in the blind spot in a visual image, and does not see a hole. For people who fall asleep while sitting up, the dropping of the chin on the chest cold be a way to activate a switch which pauses the movie and administers a painful shock to the rear. A Snore detector might also be useful in operating the pause control. Edison (talk) 01:57, 28 February 2009 (UTC)[reply]

A cheaper modification to my way still includes the ear thing (as I think that's one of the few, and probably least expensive, reliable methods of detecting when he's asleep. You could just take apart a cheap, Wal Mart style universal remote ($30?), and take out the part that mimic's the IR signal of other remotes. Then 'program' your remote's pause button in (probably just hold it in front of the receiver and press the pause button). Then have an IR transmitter with a wire connected to it ($1.00?) connected to the ear piece and the other end glued to the front of the DVD player, where the sensor is. The wires on the transmitters (at least the ones I have) are 12 ft. long, so that should suffice, or it can be extended. It would be annoying having a thin, black, almost invisible wire going across the room, but other than that it's inexpensive, and overall more reliable than wireless. I'd really like to see you build this contraption, it'd be very cool. I have no doubt that it can be done, you just need to be very good at taking things apart, and have a lot of electrical tape! Alternatively, I just thought that you could tape down the pause button on your remote, then have your dad put his head in the way. If he falls asleep and his head falls, the signal will no longer be blocked and the movie will pause. You'd probably want to get a cheap universal remote for this method too, so that he doesn't have to move the constantly pausing one to un-pause it when he wakes and risk putting in back in the wrong spot. You'd also need rechargeable batteries, or it would get equally costly eventually. -Pete5x5 (talk) 07:22, 28 February 2009 (UTC)[reply]

Anybody is answering it?????

What actually are the cyanides? —Preceding unsigned comment added by Achraz (talkcontribs) 05:02, 27 February 2009 (UTC)[reply]

Did you read our article on Cyanide? --Jayron32.talk.contribs 05:06, 27 February 2009 (UTC)[reply]
That's why I was always afraid to breathe the air in the CN Tower. :-) StuRat (talk) 05:09, 27 February 2009 (UTC)[reply]
Possible; his account was just made today and he's already made 3 ref desk posts (even though I don't know who you're talking about when you say Freewayguy). I think we have to assume good faith and try to help unless you KNOW this is someone evading a block. -Pete5x5 (talk) 05:22, 27 February 2009 (UTC)[reply]
In fact, the top of this page specifically says "Be polite and assume good faith, especially with users new to Wikipedia."  ;) -Pete5x5 (talk) 05:25, 27 February 2009 (UTC) [reply]
For those confused by the above conversation, note a post was removed by the poster [24] Nil Einne (talk) 09:39, 27 February 2009 (UTC)[reply]
Doesn't seem like Freewayguy to me. --Tango (talk) 12:20, 27 February 2009 (UTC)[reply]

wavefunction in quantum mechanics

we know that not all wave fanction are allowed wave fanction in quantum mechanics only those wave fanction that satisfy schrodinger equation and also they are wellbehaved.But if it is not wellbehaved then what difficultise arise?Supriyochowdhury (talk) 08:18, 27 February 2009 (UTC)[reply]

If you expect the wavefunction to be a probability amplitude, you'd hope that it is square integrable and therefore normalizable. Vespertine1215 (talk) 10:14, 27 February 2009 (UTC)[reply]

Rear Axle

Which is the most critical part in rear axle ? —Preceding unsigned comment added by 59.98.178.48 (talk) 11:31, 27 February 2009 (UTC)[reply]

This looks suspiciously like a homework question. In order to answer this question, we need all the information above the question: Is it a car, wagon, train...? It is a front wheel drive or rear wheel drive? Are there any modifications such as a limited slip differential? -- kainaw 14:02, 27 February 2009 (UTC)[reply]
If we're talking about rear-wheel drive vehicles - then the most complicated bit is undoubtedly the 'differential'. But the most 'critical' part? Well, I guess it's all pretty critical...you can't really do without any of it. On a front wheel drive vehicle, the rear axle is generally a rather simple affair - and in some there actually is no rear axle at all with the rear wheels mounted directly onto the frame with no connection between them...so in that sense, none of the rear axle is 'critical'. Perhaps you could explain in more detail what exactly it is that you want to know? SteveBaker (talk) 16:24, 27 February 2009 (UTC)[reply]

Rear Axle rework

In which section rework is allow ? —Preceding unsigned comment added by 59.98.178.48 (talk) 11:37, 27 February 2009 (UTC)[reply]

I'm afraid your question is not understandable. Specifically the word "rework" does not make sense in the context of your question. Your IP address is in India, so I am going to proceed on the assumption that English is not your native language and that you may speak Hindi. If by "rework" you mean गृहकार्य which (I hope) is "homework" in English, there is no section of the Wikipedia reference desk that will answer your homework questions for you. However, the respondents on the reference desk will be glad to help you:
  • Interpret your homework questions if you do not understand their meaning
  • Help you understand the ideas and concepts
  • Attempt to point you to resources that may help you answer them
By the way, I used this Hindi-English dictionary to find the Hindi word for homework. I apologize if I was wrong in assuming you spoke Hindi. I know India has many different languages. Other refdeskers, if I used the wrong Hindi word for homework, please correct it inline. Sifaka talk 02:07, 28 February 2009 (UTC)[reply]

Moth question

In the book Silence of the Lambs, a moth species that feeds solely on lachrymal fluid is mentioned. Does such a species of Lepidoptera (spelling?) actually exist? If so, what is its name? 65.167.146.130 (talk) 14:38, 27 February 2009 (UTC)[reply]

If I remember correctly, the moth in question was the Death's-head Hawkmoth, which feeds on honey from beehives, according to our article. I am not sure if Thomas Harris conflated the Death's-head moth with another moth, or as is more likely, simply invented the idea out of whole cloth. --Jayron32.talk.contribs 14:59, 27 February 2009 (UTC)[reply]
The Asian moth Lobocraspis griseifusa feeds on the tears of water buffalo. Some moths slip their probosces under the eyelids of sleeping birds to feed. It's interesting to note that a character in a sequel drinks the tears of children. --Sean 15:23, 27 February 2009 (UTC)[reply]
I guess the question then is "How does the moth make the water buffalo sad?" I got a million like this...--Jayron32.talk.contribs 15:43, 27 February 2009 (UTC)[reply]
You need some gnu material. :-) StuRat (talk) 18:10, 27 February 2009 (UTC) [reply]
I think Jayron's material is pretty good, especially if the audience has never herd it before. :-) 10draftsdeep (talk) 20:47, 27 February 2009 (UTC)[reply]
My aunt never liked that punny sort of humor, but my ungulate it up. --Sean 21:15, 27 February 2009 (UTC)[reply]
I wanted to contribute my own bad buffalo pun, but I don't anoa good one to use. --98.217.14.211 (talk) 22:41, 27 February 2009 (UTC)[reply]

I suspect that poking the water buffalo in the eye would cause it to water. Thanks for the answer, hopefully an article get created on this moth at some point. 65.167.146.130 (talk) 15:53, 27 February 2009 (UTC)[reply]

Shouldn't this question be at the Moth desk :-) Fribbler (talk) 15:59, 27 February 2009 (UTC) [reply]
grooooaaannnnnn :) Livewireo (talk) 21:49, 27 February 2009 (UTC)[reply]
I strenuously oppose levity on the Ref Desk, and I will not be cowed by anyone. That's no bull. We should not give anyone a bum steer. Edison (talk) 01:49, 28 February 2009 (UTC)[reply]
You're just trying to buffalo buffalo off the Science Desk... Nimur (talk) 04:51, 28 February 2009 (UTC)[reply]

Glutathione

How can you do to increase glutathione in the cells? It is true that 500 grams of vitamin C increase the glutathione in the cells by 50%? Thank you very much. —Preceding unsigned comment added by 151.65.126.7 (talk) 16:39, 27 February 2009 (UTC)[reply]

Our article glutathione mentions methods of supplementation. A google search of Vitamin C and Glutathione turned up this paper which found that mean red blood cell glutathione rose nearly 50% when subjects took doses of 500 milligrams (not grams) for 2-3 weeks. The authors also found that increasing the vitamin C dosage to 2000 milligrams (i.e. 2 grams) did not have a significantly different effect on glutathione levels compared to 500 mg. Sifaka talk 00:57, 28 February 2009 (UTC)[reply]

How to compare more than 5 products at biocompare.com

I am trying to compare the available unconjugated antibodies to human beta-actin, of rabbit origin for use in Western blots. When I perform a search using these parameters, the results are many. When I attempt to 'select all' and click 'compare', I am confronted with the message 'You are trying to compare more than 5 products. Please log in to add these to your selected products or reduce the number of products selected", or something to that effect. I am signed in, but there is no indication as to how I should "add these to my selected products" nor how this would help me achieve my aim of discerning which source of antibodies is the cheapest. Thanks in advance to anyone who might collaborate with me in elucidating this function of BioCompare. :) --129.125.137.62 (talk) 16:40, 27 February 2009 (UTC)[reply]

This is a common problem I have with the Compare function on many consumer websites. Some approaches I've tried:
1) Eliminate some items prior to use of the Compare option. This can be done using those attributes that are listed outside of Compare. In some cases, it's possible to sort by those characteristics, such as price, so you can eliminate some.
2) If you still have too many left, Compare them 5 at a time (or how every many are allowed), and then eliminate all but the best from each batch. Then Compare the best from each previous Compare.
BTW, this would have been an excellent Q for the Computer Ref Desk. StuRat (talk) 18:00, 27 February 2009 (UTC)[reply]

Homo sapiens

I know that homo sapiens are the only species within the homo genus still living. I find this curious. How common is it to only have one species within a genus in the animal kingdom? A Quest For Knowledge (talk) 17:56, 27 February 2009 (UTC)[reply]

Species and genus is an invention of the people that categorize life based on their own crietria. It is actually mostly arbitrary, and you will not easily find much common agreement between taxonomists on what constitutes distinct species within one genus, subspecies of the same species, different genuses, etc. etc. There's only one "Homo" species because someone decieded there was. You will often hear that breeding compatibility defines a unique species, but there are clear examples of where this falls down; for example what about three animals, A, B and C. Lets say that Animal A & B can breed to produce viable offspring, and B & C can breed to produce viable offspring, but that A & C cannot breed to produce viable offspring. Well, under the standard definition of species, A & B are the same species, B & C are the same species, but A & C are NOT the same species. So how do you classify them??? Depending on whose classification system you use, you will likely find THOUSANDS of single-species genuses. Look at Tuatara for an extreme example. It is an order that consists entirely of two species. --Jayron32.talk.contribs 18:23, 27 February 2009 (UTC)[reply]
Yes, but there used to be at least another dozen species under the homo genus. All are extinct except for us. To put it in another and less scientific way, there are lots of dog breeds (Shih Tzu, Pit Bull, German Shephard, etc.) but only one 'breed' of human. How common is this in the animal kingdom? A Quest For Knowledge (talk) 18:49, 27 February 2009 (UTC)[reply]
Just to clarify a little bit more, I understand that species can be classified in many ways depending on who is doing the classifying. This has come up before. I guess what I'm getting at is that our closest living relatives are the chimps and they seem to be pretty distant cousins. I'm wondering how unique humans are in not having any other close relatives still alive. I mentioned genus because it seemed the most succinct way of asking it. A Quest For Knowledge (talk) 19:10, 27 February 2009 (UTC)[reply]
Weeeeeell, to go back to pedantry and anthropocentrism... :) But seriously, have you seen the discussions on whether chimpanzees and humans should really be in separate genera? 79.66.56.21 (talk) 19:02, 27 February 2009 (UTC)[reply]
No, do you have a link you can point me to? A Quest For Knowledge (talk) 19:12, 27 February 2009 (UTC)[reply]
Check out The Third Chimpanzee by Jared Diamond. It's been a while since I read it, so I don't recall how serious Diamond was in his assertion or whether it was more or less to make the point that chimps and humans are very closely related indeed. According to the book, the Pan genus was named first, and so would have precedence over Homo, should the two groups be merged. The second chimpanzee, for those not familiar with primates, is the bonobo or "pygmy chimp". Matt Deres (talk) 21:28, 27 February 2009 (UTC)[reply]
Also, take a look at the various articles on race.--Shantavira|feed me 19:08, 27 February 2009 (UTC)[reply]
To my knowledge no reputable scientist today thinks that the races come from different species of any sort. --98.217.14.211 (talk) 01:38, 28 February 2009 (UTC)[reply]
Breeds of dog are different types of the varieties of the same species. They aren't unlike humans of different races, or even just different hair colours/heights/builds. There is more variation between breeds of dog than humans, at least visible variation, but that's only a difference of degree. --Tango (talk) 19:28, 27 February 2009 (UTC)[reply]
The question here isn't so much about the vexed question of "What is a species?" - which we've addressed at length many times before (Conclusion: It's a vague term...sorry 'bout that!). This question is "What is a Genus?". If humans are the only species in our genus - what defines that term in such a way as to prevent other species such as the chimpanzee from being a part of our genus - or conversely keeps us out of the same genus as the other great apes? I suspect the answer will boil down to not upsetting the religious nuts...but perhaps there is a better reason.
Our article Genus says:
The rules-of-thumb for delimiting a genus are outlined e.g. in Gill et al. (2005). According to these, a genus should fulfill 3 criteria to be descriptively useful:
  • monophyly – all descendants of an ancestral taxon are grouped together;
  • reasonable compactness – a genus should not be expanded needlessly; and
  • distinctness – in regards of evolutionarily relevant criteria, i.e. ecology, morphology, or biogeography; note that DNA sequences are a consequence rather than a condition of diverging evolutionarily lineages except in cases where they directly inhibit gene flow (e.g. postzygotic barriers).
Sadly, my knowledge of biology fails me at this point. However, I understand there to be a strong case for renaming the Common chimpanzee and the Bonobo 'Homo pan' and 'Homo paniscus' - thereby making them be the second and third extant species in our genus. Then we can arm-wrestle the chimps to see who gets to be type species for the genus....hmmm - maybe we'd better make that 'speed chess' - they'd probably beat us at arm-wrestling.
SteveBaker (talk) 19:50, 27 February 2009 (UTC)[reply]
The problem with that definition of Genus is that it is still open to interpretation. What makes two extinct individuals or groups part of the same ancestral taxon? What if new discoveries lead us to want to split a taxon into two new taxons? What happens to the genera below that? What kind of expansion does a genus need? What needless means to one person is different than another. What makes a criteria "evolutionarily relevent" with regards to saying "these two examples are from different genera or from the same genus?" Ultimately, we justinvent these "rules" for classification (which still are themselves open to the subjective interpretation of the individual).
Its not much less arbitrary than how I decide to organize my CD collection. Do I organize it by genre or alphabetically by artist? Do I intermingle Jefferson Starship with Jefferson Airplane or are they seperate? Is Jane's Addiction funk-metal, or alternative-rock? Ultimately, the whole concept of taxonomy is a human invention, created for the convenience of organizing our knowledge of living things. Insofar as it helps us find commonalities and difference among all life, it is a useful thing to classify; but as with ALL classification systems, no set of rules will cover every eventuality, and there will always be a level of arbitrariness to it. Ultimately, "because someone just decided it would be this way" ends up being the biggest justification for deciding how to classify one animal or another into one taxon or another. --Jayron32.talk.contribs 20:47, 27 February 2009 (UTC)[reply]
A similar case to humans might be the horseshoe crab, which, despite it's name, is only distantly related to crabs. There are 4 known species of horseshoe crab, but those may be the only species left in their genus, family, or even order. Similarly, the coelacanth, of which there are 2 known remaining species, are the only known survivors of their genus, family, order, and even sub-class. The bowfin appears to be an example where only one species is known to exist in the genus. See living fossil for more examples. StuRat (talk) 21:52, 27 February 2009 (UTC)[reply]
Articles more specific to the reduction in the number of Homos walking around at the same time: Neanderthal#Neanderthal_extinction, vs. Multiregional origin of modern humans. Personally I would not have been surprised if Sapiens killed off any others. Our species appears to have never been good with strangers, much less ones who would pose a much more severe threat than many of the animals of the time. --98.217.14.211 (talk) 01:38, 28 February 2009 (UTC)[reply]
You also have to be VERY careful if you try to use dogs as an example in any discussion of this sort. Dogs are quite unique in that they are thought to be the oldest domesticated animals and humans have been selectively breeding them for thousands of years, that's why they're called "breeds" of dog. The thing is tho, unlike natural selection, human selection relies purely on traits that we can see, which turn out to be only a superficial fraction of the genetic material that actually "makes" a species. So selective dog breeding has created an incredibly diverse number of physically distinct breeds, much more so then you would find anywhere in nature, especially in the relatively short time, however, genetically the dog "breeds" are still practically identical. Vespine (talk) 12:12, 28 February 2009 (UTC)[reply]

Bornholms Disease

Request for medical advice removed

Please note that the Wikipedia Reference Desk cannot provide medical advice. You should speak to a health care provider. - EronTalk 22:38, 27 February 2009 (UTC)[reply]

We can't but we could at least help the user find related information on wikipedia. (Going with our library analogy, your librarian would also not send you to your doctor if you asked where they have a book on condition XYZ.) The quality and relevance of that information is up to the reader to determine.:
The page you linked has a link to Coxsackie B virus on it. That has a bit more on treatment and a reference to a report on homeopathic treatment. The latter is also only symptom relief, though. A bit on using wikipedia: If you click on a blue word it will get you to a page that is related. To link to a wikipedia page here copy the page name and enclose it in two angled square brackets [[]].76.97.245.5 (talk) 23:01, 27 February 2009 (UTC)[reply]
I believe this was incorrectly removed as a request for medical advice (see discussion here: Wikipedia_talk:Reference_desk#.28Possible.29_Medical_advice_question_removed_-_Bornholms_disease) when it's simply a request to improve our article on Bornholm disease. As such, I have added your request to the talk page for that article: Talk:Bornholm disease#Article Improvements Needed. StuRat (talk) 17:55, 28 February 2009 (UTC)[reply]

February 28

Using neutrinos for communication

Glass Earth, Inc. is a science fiction short story by Steven Baxter. The technology part of it hinges on an invention to speed up global communications. Instead of sending signals via surface lines wrapping the globe, or bouncing them to a satellite and back, the company has hit on a way to control how neutrinos change their status, I think in the story it's their "flavor". They use a particle accelerator to produce collisions that send a stream of neutrinos in a straight line through the center of the earth and to the antipodes. Besides the accelerator, the only installation required is the detector at the other end. By modulating the neutrino states as if they were voltages, the information is transported.

I found that a fascinating conceit. Is the science simply wrong, as in it violates a fundamental constraint of nature, or could this in principle be done pending future advances?--Goodmorningworld (talk) 03:46, 28 February 2009 (UTC)[reply]

Creating neutrinos is relatively easy, although I'm not certain what degree of control one could have over their states. But the problem is that the same thing that makes neutrinos able to pass straight through the Earth with negligable loss make them almost impossible to detect: They almost never interact with anything. Neutrino detectors are notoriously massive, and maybe someone else can answer whether it's even theoretically possible for them to be small. Someguy1221 (talk) 04:56, 28 February 2009 (UTC)[reply]
AFAIK Cherenkov radiation offers a nice way of detecting neutrinos. You need a transparent medium (e.g. water) and lots of detection devices. You need that much of your transparent medium because it increases your chances of getting some results. Another way is to count collision products. The Homestake Experiment used a chlorine solution. Again you need lots of it to catch the few reactions. The guys at the Cowan and Reines neutrino experiment created 5×1013 neutrinos per second per square centimeter and were able to detect only 3 reactions per hour as a result. Don't know about the "relatively easy" part when it comes to creating neutrinos for a communications device. Neutrino oscillation sends modulating neutrino states to the realm of si-fi. The neutrinos we observe are thought to be mixes of 2 or more states and the mix changes. What might be possible is a digital device that sends digitally encoded messages in the amount of neutrinos you produce during a given period. Message density would be lousy and costs almost certainly prohibitive with current means. E.g. look how long it takes to power up the LHC or an atomic power plant. Just sending "SOS" in morse code could easily take months. There would also be a high error rate due to natural fluctuations.76.97.245.5 (talk) 07:22, 28 February 2009 (UTC)[reply]
"Relative" to detecting them, that is ;-) Someguy1221 (talk) 11:10, 28 February 2009 (UTC)[reply]
Yeah - it's certainly possible - in principle - to send messages through large, solid objects using some kind of modulated neutrino stream. But the bandwidth of such a link would be of the order of one bit per day - the transmitter would have to be something like a nuclear reactor that could be turned on and off - or perhaps moved from one place to another - and the receiver would be some VAST tank of dry-cleaning fluid buried deep in a mine-shaft, studded with costly electronics. 'possible' and 'practical' are not the same thing. But I don't really see the benefits. A set of satellites in orbit is much cheaper than the gargantuan equipment needed to modulate and detect neutrino flow - the equipment needed to send and receive such signals is small enough to be hand-held and the only real downside is that it takes maybe twice as long for the signal to get there...but compared to the delays inherent in statistical analysis of neutrino detection when just a couple of detections per hour is the maximum rate possible...there is no reason that I could imagine for wanting to use such a device. I have not read "Glass Earth Inc" and our one-line summary ("A policeman must sort through the memories of a murder to find out who the killer is, and in the process, learns more about himself then he ever knew.") is less than revealing about the purpose of this neutrino communication trick. Why was it important to the plot? SteveBaker (talk) 13:27, 28 February 2009 (UTC)[reply]
Oh! Now you're putting me on the spot… it's been years since I read the story and unfortunately I am not possessed of a photographic memory… But! The gubmint's on the case already Template:PDF: a project to induce rock deformation processes under the San Andreas fault (p.19) is already well under way. They mention "glass earth" (p.24) and that 1 in 1013 particles (p.4) make it all the way through this ole planet… Well that explains that fearsome excavation on Magnolia Boulevard… could I interest you in some prime real estate in the Valley?--Goodmorningworld (talk) 14:55, 28 February 2009 (UTC)[reply]
There are many other, less exotic, subatomic particles (like, say photons) which would work much better than neutrinos for the same purposes. Imagine me talking to you on the phone, but you only hear one consonant or vowel per hour. That's the sort of level of reliablity neutrinos provide. I suspect that this is a case of the author picking an exotic sounding real science term and just making up a use for it. Here in the real world, there does not seem to be any compelling reason to use neutrinos for communication; they will still be bound to the speed of light, and so would not be any better than ordinary radio (light) waves/photons, and there are many reasons why they would be WORSE. We already have reliable photon detectors out there which have been working for us for, oh, 100 years or so. If it ain't broke... --Jayron32.talk.contribs 13:31, 28 February 2009 (UTC)[reply]
The benefit is that they can get from A to B marginally quicker since going through the Earth is quicker than going round it. So, we have fantastic latency and terrible bandwidth - kind of the opposite of using satellites (although the bandwidth for neutrinos is several orders of magnitude worse than the latency for satellites). --Tango (talk) 14:41, 28 February 2009 (UTC)[reply]
Actually I read somewhere (coulda been in the Weekly World News, coulda been SciAm) that data sent over the wires does not in fact propagate at light speed despite them being electromagnetic waves, but only at one-tenth the speed effectively… guess them lil' photons bump into each other like billard balls and there is some friction involved!--Goodmorningworld (talk) 15:02, 28 February 2009 (UTC)[reply]
The whole point is that neutrinos are better than photons, albeit in some very specific ways that may not be of use in most situations. Neutrinos are essentially unstoppable and unshieldable; unlike radio or microwave photons, they're not subject to any sort of interference. Lightning and solar storms have no effect. You don't need to have a dish or antenna on the roof to pick them up; they'll get to you in a bunker a mile underground, so you're protected from blizzards, hurricanes, and bombs. Open line of sight isn't required for neutrinos.
Using visible or infrared photons is also inferior over long distances. Either you need line of sight and clear air (for laser communications), or you need waveguides (fiber optics) and lots of repeaters to cover any substantial distance. Long-distance optical fiber is expensive to place, and vulnerable to enemy action, political whims, power failures, ship anchors (quite common), and earthquakes: [25].
In all cases, the time of flight for neutrino communications is shorter. Consider the worst case scenario: two people want to communicate between points on roughly opposite sides of the globe (Perth, Australia and New York, USA, perhaps). The straight-line distance (as the neutrino flies) is about twelve thousand kilometers; going around the surface circumference is about twenty thousand. (The actual distance over the surface will be quite a bit longer, as there aren't any great-circle communications links from Perth to New York.) Communication over fiber will take longer than the distance suggests, as the speed of light is about 30% lower in optical fiber than it is in vacuum; substantial time will also be lost at relay stations where the signal is processed in and out of fibers. Radio is even worse. The distance from ground to geostationary satellite and back is more than seventy thousand kilometers. For a few applications (telepresence for telesurgery, for example) reduction in latency is worth almost any price.
The catch, of course, is that we lack any method to generate and detect powerful, modulated beams of neutrinos in anything approaching an efficient or cost-effective way. With current technology we couldn't send more than a few bits an hour, and we'd have to use billion-dollar particle accelerators and tens-of-millions-of-dollars detectors. For such a system to be viable, you need to make neutrino generation and detection much more efficient; call that the unobtainium problem for this science fiction concept. To the best of my knowledge, the generation problem is not solvable any time soon, and the detection problem is virtually intractable.
Incidentally, the world of science fiction is chock-full of uses for neutrinos. In Larry Niven's Ringworld series, the Ringworld's foundation is made of scrith — a material that stops roughly half of all neutrinos striking it with a layer about 30 meters deep. Greg Egan used neutrinos in Wang's Carpets as a gentle, non-invasive probe of alien life. Dan Simmons uses modulated neutrinos for military communication in the later parts of his Hyperion series. The assorted Star Trek spinoffs can't resist the little neutrino, and – occasionally – they do use it in a way that makes a modicum of sense. (Their favourite particle, though, is most certainly the tachyon. Since it hasn't been observed and its properties are poorly defined, it's much more useful to twenty-fourth century scriptwriters.) TenOfAllTrades(talk) 15:02, 28 February 2009 (UTC)[reply]

(unindent) It's crazy to claim that the time of flight of a neutrino makes it superior to a photon. The best neutrino detectors we have can only detect one out of every...uncountably large number...of neutrino's! So sure, the photons get there pretty quickly but all but 0.000000000...lots more zeroes...0000001% of them are completely ignored by the detector! So your transmitter has to transmit the first bit of the message as a gazillion-bazillion neutrons for about 20 minutes in order to be reasonably confident that the detector will pick up one or two of them. The tiny fraction of a second of latency you save by shooting your neutrino's through the earth are completely SWAMPED by the 20 minutes you have to wait in order to detect one of them! There is no object in the universe that's sufficiently transparent to neutrino's - yet opaque to photons that's big enough to save you 20 minutes at the speed of light!

Meanwhile, we've sent a few terabits over good old transatlantic cables, satellites, or carrier pidgeons. No matter the interference - if you could use a piece of equipment the size of a neutrino transmitter/reciever pair and send a few bits per hour we could have error-correcting codes with redundant transmission and enough sheer transmission power that we'd get through ANY amount of interference. Do you SERIOUSLY believe that there are any circumstances whatever in which photons are not at least a million times better? That's just beyond crazy!

Also neutrino's are NOT immune to interference. If your transmitter happens to be between the sun and the receiver then your receiver will be totally swamped with solar neutrino's and you'll get big-time interference from that damned great neutrino source in the sky. This might not be a practical problem for one transmitter and one receiver - but as soon as you get widespread adoption of the technology, those outages and interference between transmitters (which - bear in mind - cannot by any means be directional) would soon be a problem.

This is so far beyond reasonable...I can't believe anyone would even consider a vote in favor of the idea. SteveBaker (talk) 19:24, 28 February 2009 (UTC)[reply]

See, I saved you the trouble of brewing a cup of coffee LOL. Actually 1 in 1013 neutrinos are detectable with current technology after their trip through Earth (a lump of rock to us, an airy wisp to them). This figure comes from the DUSEL people at Homestake that I linked to above. I admit, I was having a bit of fun with them earlier, as their PDF file looks like something cooked up by a mad scientist gunning for the Ig Nobel prize, but in fact Raymond Davis Jr., who directed the Homestake Experiment, shared the Nobel prize for it in 2002. 106 is a million, 109 a billion, so we need to send ten trillion neutrinos on their way if one of them is to register on the detector. Multiply this by another factor of thousand to get a margin of safety and we're at ten quadrillion. That's only twice as much as the investment banks wrote off last year in dollars :-) How many neutrinos can we transmit per nanosecond, and how quickly can we modulate the stream? How expensive is that and how much do we need to figure for the detector? Suddenly it's beginning to look much more like an engineering and business problem than a physics problem. Which means that Yankee ingenuity will find a way, as it always has…(Probably going to need more orders of magnitude for error correction and a "wrapper" that distinguishes your message neutrinos from other broadcasters, the Sun and cosmic sources…8-) --Goodmorningworld (talk) 19:59, 28 February 2009 (UTC)[reply]
Exactly, it's an engineering and business problem. That's what all the responses have said - it's theoretically possible, just overwhelmingly impractical. --Tango (talk) 20:42, 28 February 2009 (UTC)[reply]
Steve Baker has it 100% right here (as usual). Lets simplify it. The supposed "benefit" of nutrinos is that they can travel through the Earth. The problem is, your nutrino detector is going to be made out of the same stuff the Earth is. Like atoms and molecules and stuff like that. So you broadcast some information via nutrinos. How do you catch those nutrinos to be able to read what you have broadcast? See the problem here? --Jayron32.talk.contribs 19:50, 28 February 2009 (UTC)[reply]
We already have neutrino detectors, I think it's creating the neutrino stream that will be the hard part - creating enough neutrinos quickly enough for the largest imaginable detector to pick up the signal at even a few bits an hour would be next to impossible. Building a slightly smaller detector wouldn't make that big a difference. --Tango (talk) 20:42, 28 February 2009 (UTC)[reply]
You may be a bit too pessimistic. I, however, was much too optimistic. The MINOS experiment in Minnesota collects collision events of neutrinos fired off from Chicago.

More than a trillion man-made neutrinos will pass through the MINOS detector each year. Because neutrinos interact so rarely, only about 1,500 of them each year will collide with atoms inside the detector.

— MINOS FAQ
The "trillion" sounds low, could be a dumbed-down figure to signify "very many" for the general public. The 1500 events per year arithmetically come out to a few hours per bit and perhaps give hope for still more in the future. However the 1500 per year is a statistical figure which cannot be taken as "1500 smallest units of information", the number of bits per year would be smaller and availability is capricious.
This is very disheartening, I had no idea that it was so hard to generate neutrinos at a handsome clip.
Maybe we should start thinking out of the box. Pocket supernovas, anyone?--Goodmorningworld (talk) 22:52, 28 February 2009 (UTC)[reply]
This confrontation on the Reference Desk a year ago is an interesting and relevant read. --Bowlhover (talk) 19:55, 28 February 2009 (UTC) [reply]

Observation of a mysterious psychological phenomenon

How is it that an individual is able to detect when another is staring at him or her even when the starer seems to be beyond even the peripheral vision of the detecter? Furthermore, as the starer, I can usually sense that the other individual can detect my staring insofar as I seem to know when the detecter will return the stare imminently? Is it the case that I was mistaken in the first question and the starer is simply NOT beyond the periphery of the detecter, or is there something more to this apparent example of ESP?Lashyn (talk) 04:54, 28 February 2009 (UTC)[reply]

It's likely not ESP. There are several things at work here probably:
  1. When someone is very close to you and staring at you, like right behind your back, there are other cues, such as the sound of them breathing or their body heat or smell, which may lie below your conscious perception, but which you clearly are able to detect. Thus, that "creepy" feeling when someone is reading over your shoulder. You can't see them, but you "know" they are there because you can sense them with your other senses.
  2. You're eyes are scanning much more than you realize. When you look at something, your eyes don't focus exclusively on that thing, but rather they spend most of the time looking at what you are concentrating on, and the rest of the time scanning your field of vision. If you watch someone's eyes very closely, you will see they are almost never "still", but constantly shifting and moving and refocusing. Thus, while you aren't looking at the guy staring at you, you still "see" him. Also, our minds are atuned to making eye contact; think how disconcerting it is to talk to someone who DOESN'T make it. Our mind automatically notices when someone is making eye contact with us, even from a great distance. These two things make evolutionary sense, as the first thing means that we can reflexly react to a threat even if we aren't focusing on it. The second makes sense because knowing when someone is watching you can be quite important if you need to assess his friend/foe status. Someone looking at you should always be a "person of interest"...
  3. A sort of deja vu when you realize that someone IS staring at you. When you notice someone staring at you, you reflexively assume he HAS been staring at you for a while, then you convince yourself that you noticed it earlier. You really didn't, but like all forms of deja vu, your mind creates the memory and so it feels real.
Does this all sound like reasonable explanations besides ESP? --Jayron32.talk.contribs 05:20, 28 February 2009 (UTC)[reply]
We have an article for this: The Psychic Staring Effect, but it's not very comprehensive, probably because there doesn't seem to be a lot of convincing scientific data about the thing, despite numerous research efforts. Still, it may give you some pointers on where to look for more information. Personally, I'm extremely skeptical. -- Captain Disdain (talk) 10:16, 28 February 2009 (UTC)[reply]
I too am extremely skeptical. I suspect that the 'observer selection' effect is happening. When someone sneaks up and stares at you from behind - but you DON'T notice them - you aren't creeped-out and you never know that you failed to spot them. It's ONLY in those cases when you did eventually spot them that you (perhaps retrospectively) felt that you were 'feeling their presence'. There is strong evidence that our conscious minds operate with a significant delay behind 'real time' events and that our subconsciousnesses 'edit' the perception of the world to make everything self-consistent after the fact. So it could be that we see someone who WAS looking at us - this gives our conscious mind the 'creeped-out' feeling immediately - and then a second or so later, the knowledge that we just saw them hits our conscious brain.
But if it were somehow to be shown to be true aside from all of that - then there are even more subtle cues you could be picking up on - every object in a room reflects light onto every other object - and every object blocks ambient light from every other object. When something as large as a person moves within a reasonably close distance to you, your surroundings change in subtle ways - shadows shift as light that was reflected from the walls behind you is blocked, the color of the person's clothing alters the color of the light being reflected off of things close to you. Similar things happen in the audio domain - ambient sounds such as the whir of the fan in your computer are attenuated by this new, large, soft object in your proximity. This is more than enough for your visual/audio system to realise that there is something large in close proximity that you aren't otherwise aware of. The 'creepy feeling' would be a very useful evolved response to a potential danger. "Warning - Potential sabre-toothed tiger sneaking up behind you - don't move but get ready to move!".
There is certainly no reason to suspect this 'psychic' crap might be true. The idea that this rather simple phenomenon could require most of physics to be rewritten when there are any number of plausible explanations is an exceedingly stupid one. SteveBaker (talk) 13:13, 28 February 2009 (UTC)[reply]
My psychic staring perception is absolutely awful. Maybe I'm paranoid (actually, no maybe about it), but I always think people are staring at me, and so I am very self-conscious. I quite often look around to find that people actually aren't staring at me. This whole psychic staring effect to me seems like a load of confirmation bias. --Mark PEA (talk) 14:19, 28 February 2009 (UTC)[reply]
As a matter of fact, I and my many fellow sufferers from Human Spontaneous Involuntary Invisibility, or HSII, are afflicted with the opposite condition. There's always people cutting in front of us in line as if we weren't there, and when we get to the counter at the Dept. of Motor Vehicles the lady gets up and puts up the Lunch Break sign because she doesn't see us.--Goodmorningworld (talk) 15:22, 28 February 2009 (UTC)[reply]
Heh, I get that even when I'm the only customer. Typically, it's in a cafe where you have to order at the counter. I'm in plain view (and I have a fairly large build, so I'm hardly easy to miss), but sometimes I've stood there for a couple of minutes, just to see how long it takes before one of the up to 6 people behind the counter notices there's someone there. And then one of them finally says "Oh, I didn't see you there. Are you right?" That tempts me to turn and walk out, but that would mean I've wasted my time, so I say "No, not yet" or some equally smart remark, and order what I came for. -- JackofOz (talk) 21:50, 28 February 2009 (UTC)[reply]
Rupert Sheldrake's book "The Sense of Being Stared At" covers this. --TammyMoet (talk) 18:01, 28 February 2009 (UTC)[reply]
The Sense of Being Stared At#Tests of the staring effect covers the flaws of his experiments. --Mark PEA (talk) 19:18, 28 February 2009 (UTC)[reply]

I deny the truth of this claimed effect. It has been tested repeatedly by psychologists back to Titchenor, and only chance effects were seen in many opf the experiments. It seems to be purely anecdotal. If the starer's face is visible in peripheral vision, it is very salient, draws your attention, and AHA! You've caught them. If it is out of sight and you happen to turn and catch them, it confirms the myth. But you are utterly unaware of the times when a person out of your visual field stared at you and you failed to catch them. Like other ESP experiments, it is easy to produce positive results. Edison (talk) 21:04, 28 February 2009 (UTC)[reply]

contradiction between lorentz transformation and length contraction

the lorentz transformation equation for x' in the coordinate system K' as seen from the other coordinate system K is . But this means that x' should be farther from the origin as seen from K which means that the length will increase. This is contradictory to the length contradiction in special relativity. Please explain if i am wrong--harish (talk) 09:05, 28 February 2009 (UTC)[reply]

The length of an object, say , is obtained by simultaneously measuring the position of two extremes, say point 1 and point 2 with coordinates and and subtracting them. So . The condition of simultaneously measuring both ends tells us that . If now we apply the Lorentz transformation to both time and space coordinates at both extreme points, we get four equations:

,
,
, and
.

But we've seen that which allows us to obtain

which simplifys as

and

The expression for then becomes

Now, assuming that the object is at rest in the coordinate system K (that's an important point), we must have and we don't care about the simultaneity of those measurements since the object is at rest. Defining , we get

.

Conclusion: the size of the object measured in a coordinate system K' where it is in motion is smaller then its size measured in a coordinate system K where it is at rest. Dauto (talk) 16:24, 28 February 2009 (UTC)[reply]

current even not grounded

Hello,

It seems simpler.

If one is touching a current carrying wire no matter current is A.C. or D.C. should one get a current through its body.I think it should. Consider two points on my finger touching wire they will have some potential difference and ofcourse current will flow .But ya if one was touching ground potential difference will high and got high current so but if only wire is touch P.D. will be there and so current.

Is that concept is right and if not whats wrong with that and how can be improved. —Preceding unsigned comment added by 122.163.42.205 (talk) 12:58, 28 February 2009 (UTC)[reply]

If the current in the wires is not enough to overcome the electrical impedance or Electrical resistance of your body, then the current will not flow to any meaningful amount. (the equations imply perfect linearity; that is ANY voltage should always produce a current in any medium; however for practical purposes, extremely high resistances will produce such small currents that they are undetectable, and therefore essentially nil). If you have two wires with a small voltage difference between them, and hold one wire in each hand, likely no measureable current will flow. If the voltage is high enough, then current WILL flow, which can be a dangerous thing, as there's stuff in your body that does not react kindly to having electricty pass through it. Even if it causes no damage, it can create some uncomfortable or painful effects. Now, as to the difference between passing through the two wires OR passing from one wire to the ground; it depends upon which voltage difference is greater. Since the impedence between your outstretched hands and your hands and your feet is likely to be similar, the deciding factor on which path the electrons take will be whichever path will allow the electrons to lose the most energy. If the path to the ground does this, it will take that path rather than the path to the other wire. --Jayron32.talk.contribs 13:24, 28 February 2009 (UTC)[reply]


If you touch even one wire with one finger, even while not touching a grounded object, painful or dangerous current can flow, if the wire is at a high enough voltage like a transmission line. It might be due to capacitive current. Edison (talk) 20:48, 28 February 2009 (UTC)[reply]

If you're not familiar with capacitive current, I will briefly summarize it. It is possible for current to flow, even though there is no complete circuit, for some amount of time. During this time, electric charge is stored on a capacitor (which may be your body). This current could be sufficiently high to be harmful. Also, remember that you can have a complete-circuit to ground if you are not totally isolated from the environment (a lot of materials act as conductors when a sufficiently high voltage is applied). Nimur (talk) 22:41, 28 February 2009 (UTC)[reply]

Diagnosing asthma in children

Hi all the article on asthma does not say why peak expiratory flow rate tests are not used to diagnose asthma in children. I have read elsewhere that they are used in children over 5 and in another source, Clinical Medicine by Kumar and Clark, that excercise tests are mainly used to diagnose asthma in children. Please help - thanks in advance. —Preceding unsigned comment added by 139.222.241.116 (talk) 15:39, 28 February 2009 (UTC)[reply]

Peak expiratory flow rate measurement is strongly influenced by effort. It is difficult to get young children to exert a standard amount of effort for what they may see as a pointless exercise. Also, asthma is "reactive" airway disease, and as a rule the lungs of a child with asthma function normally at rest (between exacerbations). Exercise is a common trigger, but is not uniformly so. It's a heterogeneous disease. --Scray (talk) 17:36, 28 February 2009 (UTC)[reply]

Many thanks for the reply. But your post made me think - The PEFR test is used commonly in diagnosing asthma in adults but don't the lungs of adults who have asthma function normally at rest as well? So how is the test beneficial? —Preceding unsigned comment added by 139.222.240.115 (talk) 18:36, 28 February 2009 (UTC)[reply]

Why all the various notations for wavelength and frequency?

I hate using (nu) because it looks like a v

Also, there is lamda, and f, and h

Please someone correct me if I'm wrong

(nu) = frequency

f is also frequency

lamda is λ is wavelength

h is also wavelength?

I am learning chemistry and physics at the same time, and I already asked at Yahoo answers. I am OCD, and this multi-naming is really truly frustrating me. I am fixing to start the chapters on waves, and optics (all in physics) but also am doing chemistry review and there is some alternate terminology but the reason is explained well, but I just can't understand it unless someone puts it in laymen's terms. I have tried to read the relevant wikipedia articles, and the gist is something like "chemistry uses lamda, h, and (nu) to highlight the relationship that electromagnetic radiation propogates at a constant phase speed of c = 3*10^8 m/s" whereas "physics takes a holistic approach and analyzes this stuff from looking at everyday wave phenomena like a buoy in the ocean--the waves can be imagined as the buoy oscillating up and down in simple harmonic motion, and its not a physical phenomenon but almost like a mathematical phenomenon governed by a math formula x-doubledot plus cosA(x+phi) = 0"

This my amalgamation of trying to make sense of various online sources with what my book says. My instincts were to ask this on the math refdesk, because I like the style of mathematicians' explanations on stuff like this. I received a very good answer a long time ago about the dx or dt at the end of an integral explanation, and I truly understood the explanation. I don't really understand what I wrote, but I wish to understand why use the various symbols. I really do want to know the reasoning behind keeping around both sets. I personally prefer f for frequency and don't have preference on what to use on the rest.

I am going to print out my responses, because I really need a custom explanation. I like math symbols that make sense, and delving into the science fields of chemistry and physics, they don't have the "purity" that I'm accustomed to from my endeavors in calculus.

If someone else can just write me an explanation that makes sense to me, and won't conflict with my science books, then I can finally just memorize the formulas. Until then, I'm hesitant of memorizing the wrong set of formulas. I am also a perfectionist, as evidenced by how apologetic I am for asking this simple question. Please AGF me, and help me make sense of these two seemingly incompatible reasonings for keeping around both sets of symbols. Thanks very much.

LeeJaedong (talk) 16:42, 28 February 2009 (UTC)[reply]

h is Planck's constant. The others are right. It doesn't really matter what you call things, you use whatever symbols aren't already being used for something else, although there are certainly symbols that are generally used for certain things - you should always specify what you mean by each symbol the first time you use it, though. f is often used for a miscellaneous function, so isn't always available for frequency. You may need to alter your handwriting slightly to distinguish all the different symbols. My handwriting changed a lot during my first year of uni - I now cross 7's and z's, I loop ells, I put ticks at the top of 1's, v's, w's and o's, etc., etc.. I write nu's with a straight line down and right, and then a curved line up and out to the right and then back in to the left, whereas v's have a straight line down and right and a straight line up and right, with a horizontal tick at the end - they are easy to distinguish (my u's look more like by v's than my nu's do - that's what the horizontal tick is for, u's don't have them). --Tango (talk) 16:57, 28 February 2009 (UTC)[reply]

I also have a pet peeve against the use of the because it looks like a v. But you must be aware that other people like using it and there's nothing you can do about it. There is no other reason for the the multiple notations beyond the fact that different people have different tastes. Get used to it. Just remember that the impostant thing is the physical concept, not the symbol used to describe it. Dauto (talk) 17:05, 28 February 2009 (UTC)[reply]

Characteristic time scale for rotational motion.

I am interested in knowing the time scale on which a molecule rotates in free space. I have estimates for the three principal moments of inertia. I know that this time would depend on the particular J state involved, so perhaps we can assume that the molecule is in the ground rotational state. I have looked at Rotational spectroscopy and Moment of inertia but neither seem to address this issue. Thanks in advance Man It's So Loud In Here (talk) 16:55, 28 February 2009 (UTC)[reply]

Use the formula and the eigenvalue relations . Dauto (talk) 17:38, 28 February 2009 (UTC)[reply]
Is this material covered in a WP article, for those who would like to understand more? --Scray (talk) 17:46, 28 February 2009 (UTC)[reply]
From infrared spectroscopy, you will notice that most molecular vibrations and rotations occur at infrared frequencies, while atomic vibrations and rotations often occur at optical frequencies. Larger molecules will have resonances at lower frequencies (due to larger mass), but their spectra may be dominated by functional groups, (small, tightly-coupled sub-molecule atom-groups). Do you need help converting infrared frequency to time scales? These should be on the order of 10x10-15 seconds, or ~ millionths of nanoseconds. Nimur (talk) 22:46, 28 February 2009 (UTC)[reply]

Hormones and neurotransmitters

As I understand, the same chemical can be a hormone and a neurotransmitter, but when is it a hormone and when is it a neurotransmitter? Do I understand correctly that both chemicals influence other cells, but if it is produced by a nerve cell, we call it neurotransmitter, and when it is produced by any other cell, we call it a hormone? Or is there more to it? Lova Falk (talk) 17:14, 28 February 2009 (UTC)[reply]

A hormone is released in the bloodstream. A neurotransmitter is released into the synapse cleft. Dauto (talk) 17:25, 28 February 2009 (UTC)[reply]
Ah! It is as simple as this. Thank you! Lova Falk (talk) 17:37, 28 February 2009 (UTC)[reply]
PS Maybe not quite as simple. My textbook says: "There are axosecretory synapses, in which an axon terminal synapses with a tiny blood vessel called a capillary and secretes its transmitter directly into the blood." Do the authors of this book just express themselves in a sloppy way or are there exceptions to the rule? Lova Falk (talk) 17:49, 28 February 2009 (UTC)[reply]
The difference between hormone and neutransmitter is in scope of action. When the trasmitter is released in the bloodstream (using an axosecretory synapses, for instance) it may act on all the bocy cells that heppen to have detectors for that substance (provided that enough of the transmitter was released to begin with).When the transmitter is released in a synapses cleft (acting on another neuron, for istance) the action may be restricted to just one cell at a time and will act on a much shorter timeframe. Dauto (talk) 18:49, 28 February 2009 (UTC)[reply]

soda ash clairification

Is soda ash used in swimming pool treatment the samae thing as soda ash used in well water treatment. We lost our supplier for the soda ash we used in our water system and can see there is what is called soda ash in both swimming pool sites and water sites. Thank you Marilyn —Preceding unsigned comment added by 63.46.128.40 (talk) 18:26, 28 February 2009 (UTC)[reply]

Soda ash is the old-fashioned name for Sodium carbonate - and sodium-carbonate is sodium-carbonate is sodium-carbonate. So if what you have is pure soda ash with no other ingredients - and what you need is pure soda ash with no other ingredients - then it should be just fine. Since both swimming pools and drinking water need similar properties - I (personally) would feel perfectly OK with stuffing the pool stuff into my well. SteveBaker (talk) 18:57, 28 February 2009 (UTC)[reply]
Difficult to know, because nonstandard nomenclature is, well... nonstandard. Do you have any old buckets which state definitively whether it's sodium carbonate? Additionally, please note that different formulations can make a lot of difference - for one easy example, it would be a horrible idea to use 80% Na2CO3 in the same amount that you would use 10%. (Not to mention that you wouldn't know what's in the remaining 20% or 90% composition.)
Even with a chemistry degree, I would be extremely hesitant to use something meant for one purpose for another in this manner. arimareiji (talk) 19:03, 28 February 2009 (UTC)[reply]
You REALLY must be careful, because like other terms such as Lime (material) or Potash, there are actually many compounds that fall under the "soda ash" label. Soda ash (unqualified) is usually used to mean sodium carbonate, but "caustic soda ash" is sodium hydroxide and there are other variations. The problem is that, in the 19th century world, "Soda ash" meant "a melange of sodium containing compounds" while "potash" meant the same for potassium and "lime" meant the same for calcium. These compounds could be seperated or treated to form new compounds, but the entire class is still known as "soda ash"; just different kinds of soda ash. The convention has just hung around for 150 years or so. As noted, what is called "soda ash" for one application may be very different than what is "soda ash" for another. You would need to know the assay of the soda ash in the water treatment application, and obtain something which was the same stuff. Just being called "soda ash" makes no guarantee that it will be... --Jayron32.talk.contribs 19:14, 28 February 2009 (UTC)[reply]
What Jayron said. One other thing which might help explain why we're advising such strong caution... drinking genuinely strongly-alkaline water is a really good way to wind up in the hospital or dead, health food gurus' claims notwithstanding. Underdoing the amount (and alkalinity) could be dangerous if it means bacteria don't get zapped, but overdoing it can easily be just as dangerous. If you ever saw a skull and crossbones on a sign in a really old Western that warned "ALKALI WATER" - well, alkalosis can kill you a lot faster than dehydration. arimareiji (talk) 19:40, 28 February 2009 (UTC)[reply]

Why Are People So Stupid?

Why do people take a health class, learn about the germ theory of disease, and yet still think that going outside in the cold is what causes their illnesses. I thought we were out of the dark ages, but I guess not. Why do you think people think this? 169.229.75.128 (talk) 19:24, 28 February 2009 (UTC)[reply]

Because there is a correlation between low temperatures and people getting ill, and people often misunderstand the natures of causation and correlation. See Common cold#Exposure to cold weather for details. --Tango (talk) 19:30, 28 February 2009 (UTC)[reply]
That, and people have forgotten about the importance of handwashing in disease prevention. It's nasty to think about, but 1) cold weather -> runny nose 2) runny nose gets wiped by hand 3) hand touches object (i.e. door handles) 4) other hand acquires infectious mucus from object 5) repeat steps 1 and 2. arimareiji (talk) 19:44, 28 February 2009 (UTC)[reply]
But the cold weather doesn't CAUSE the runny nose. 169.229.75.128 (talk) 19:55, 28 February 2009 (UTC)[reply]
My nose and the weather outside beg to differ. ;-) But more importantly, so does wisegeek.arimareiji (talk) 20:12, 28 February 2009 (UTC)[reply]
I concur. We now have two anecdotes, thus data, QED. :) --Tango (talk) 20:33, 28 February 2009 (UTC)[reply]
Germs may be the cause of a particular disease, but dried out sinuses due to low humidity inside in the winter might lessen the bodies resistance, as could stress or other effects of extreme cold. Cold weather might increase to chances of exposure to a germ resulting in the full-blown infeection. Edison (talk) 20:45, 28 February 2009 (UTC)[reply]
Cold weather does too cause runny nose. When you inhale, the cold air cools the internal surfaces of you nose cavity. Then when you exale warm and humid air, the vapor condenses on these cold surfaces and the now liquid water runs down your nose, hence a runny nose. Dauto (talk) 20:48, 28 February 2009 (UTC)[reply]


The problem is that the average person in the street is continuously bombarded with false information. We're told that herbal remedies will cure medical conditions by 'boosting your immune system' (these statements have not been verified by the FDA, this food supplement is not designed to diagnose or treat any medical condition) and that you can lose (up to) 30lbs (results not typical) just by taking this pill without exercising - and eating all you want!! (results obtained along with diet and exercise). We find 'dianetics' filed in amongst 'diabetics' in book stores - we have homeopathic remedies (ie pure water) in amongst actual real treatments in pharmacies. We have adverts on TV telling you that "it's now well accepted that male enhancement really works" - and that "we couldn't say this was true if we didn't have TWO!!! US patents for our product!!!" (sadly, yes you could - and a patent proves nothing). We have 'energy drinks' in the 'health food' aisles in supermarkets and yoghurt that makes you shit at "more regular" intervals (Why is that important?). I especially liked the product that contains as it's "active ingredient" something described as "a tiny relative of the mushroom" (they mean it has fungus in it). Even fully trained doctors are prescribing antibiotics for viral diseases such as the common cold because people who've sat around for an hour with a whiny sick kid don't want to leave the office without a prescription for SOMETHING in their hands. No wonder people are confused! Who should they believe? SteveBaker (talk) 20:49, 28 February 2009 (UTC)[reply]

We don't need no education? But seriously, the United States has mandatory free public education and in many places it's all-year long. If this kind of exposure to scientific fact is not able to eliminate misinformation, I can only conclude that in some statistical sense, much of the population will never be able to distinguish fact-based reality from fiction. Calling them "stupid" is just a pejorative label of their alternative viewpoint about reality. Nimur (talk) 22:50, 28 February 2009 (UTC)[reply]

Stone-age axe-head factory.

I recently listened to the audio book of Bill Bryson's A Short History of Nearly Everything. Near the end he speaks of a place in Africa (Rift Valley?) where the are remains of a massive stone-age axe-head factory. He describes how hundreds of people must have been employed in the production of stone tools for some time at this one location. Because I only have the audio book, I don't know how he is spelling the name of this place, but it sounds like Olega Sally, although that's not how it's spelt so I can't find out any further info on it. Can anyone give me the actual name of this place please? Thanks.